You are on page 1of 459

1.

Which of the following is owned by the RBI and is operated by


CCIL on behalf of the RBI?
A. RTGS
B. SSS
C. NDS-OM
D. All of these
E. None of these

Answer & Explanation

C. NDS-OM
Explanation:
Negotiated Dealing System- Order Matching (NDS-OM) is owned by the
RBI and is operated by CCIL on behalf of the RBI. NDS-OM is an
electronic, screen based, anonymous, order driven trading system for
dealing in Government securities which was introduced in 2005.

2. What are the types of trades that are currently settled through
CCIL?
A. Forex inter-bank Cash
B. Tom
C. Spot
D. Forward USD/INR
E. All of these

Answer & Explanation

E. All of these
Explanation:
CCIL settles all Forex inter-bank Cash, Tom, Spot and Forward
USD/INR transactions.

3. Which of the folllowing is CCIL’s Settlement Member?


A. American Express Banking Corp.
B. UBS Switzerland AG

www.bankingpdf.com
C. BNP Paribas
D. Antwerp Diamond Bank NV.
E. Bank of America N.T. & S.A.

Answer & Explanation

B. UBS Switzerland AG
Explanation:
CCIL provides CLS Settlement through a CLS Settlement Bank namely
UBS Switzerland AG.

4. Continuous Linked Settlement (CLS) Bank offers settlement in


______currencies
A. 16
B. 17
C. 24
D. 18
E. 14

Answer & Explanation

D. 18
Explanation:
Continuous Linked Settlement (CLS) Bank offers settlement in 18
currencies

5. __________ is a web-based application for submission of


Collateral Notices and Membership related data electronically.
A. e-Notice System
B. Settlement Guarantee Fund
C. Exposure Limit
D. Default Fund
E. All of these

Answer & Explanation

www.bankingpdf.com
A. e-Notice System
Explanation:
eNotice System is a web-based application for submission of Collateral
Notices and Membership related data electronically.

6. ________ are unsecured bonds that cannot be converted to


company equity or stock.
A. Fully Convertible Debentures
B. Partly Convertible Debentures
C. Convertible Debentures
D. Non Convertible Debentures
E. All of these

Answer & Explanation

D. Non Convertible Debentures


Explanation:
Convertible Debentures are unsecured bonds that cannot be converted
to company equity or stock.

7. Which of the following is/are used in the loan approval


processes?
A. Credit Information Report (CIR)
B. CIBIL TransUnion Score
C. Both(1) and (2)
D. STRIPS
E. None of these

Answer & Explanation

C. Both(A) and (B)


Explanation:
The Credit Information Report (CIR) and CIBIL TransUnion Score are
used in the loan approval process.

www.bankingpdf.com
8. How many digits are there in CIBIL score?
A. 4
B. 3
C. 6
D. 9
E. None of these

Answer & Explanation

B. 3
Explanation:
The CIBIL Score is a 3-digit number ranging from 300-900

9. What is the full form of SEPA?


A. Simple Euro Payments Area
B. Simple Euro Payments Attempt
C. Single Euro Payments Area
D. Single Euro Payments Attempt
E. Standard Euro Payments Area

Answer & Explanation

C. Single Euro Payments Area


Explanation:
The Single Euro Payments Area (SEPA) is a payment-integration
initiative of the European Union for simplification of bank transfers
denominated in euro.

10. DVP III is a settlement process in which settlements are


done on _______ after achieving multilateral netting.
A. Delivery Versus Payment
B. Payment Versus Payment
C. Payment Versus Delivery
D. Payment and Settlement
E. All of these

www.bankingpdf.com
Answer & Explanation

A. Delivery Versus Payment


Explanation:
DVP III is a settlement process in which settlements are done on
Delivery versus Payment after achieving multilateral netting.

Next 10 questions

1. What is the full form of RNBC?


A. Reserve Non-Banking Company
B. Reverse Non-Banking Company
C. Residential Non-Banking Company
D. Rural Non-Banking Company
E. Residuary Non-Banking Company

Answer & Explanation

E. Residuary Non-Banking Company


Explanation:
Residuary Non-Banking Companies(RNBCs) is one of the categories of
NBFCs whose principal business is acceptance of deposits and investing
in approved securities.

2. RNBCs should offer a rate of interest of not less than ____ per
annum on term deposits
A. 5%
B. 4%
C. 6%
D. 7%
E. None of these

www.bankingpdf.com
Answer & Explanation

A. 5%
Explanation:
RNBCs should offer a rate of interest of not less than 5% per annum on
term deposits

3. RNBCs should offer a rate of interest of ______ on daily


deposits.
A. 5.5%
B. 3.4%
C. 4.6%
D. 3.7%
E. 3.5%

Answer & Explanation

E. 3.5%
Explanation:
RNBCs should offer a rate of interest of 3.5% on daily deposits.

4. RNBCs cannot accept deposits for a period less than ____


months and more than ____ months.
A. 12, 60
B. 12, 72
C. 24, 72
D. 12, 84
E. 24, 84

Answer & Explanation

D. 12, 84
Explanation:
RNBCs cannot accept deposits for a period less than 12 months and
more than 84 months.

www.bankingpdf.com
5. Which of the following is an electronic trading platform,
operated by the Reserve Bank of India, used to facilitate the
exchange of government securities and other money market
instruments?
A. Payment and Settlements System
B. Sachet
C. Negotiated Dealing System
D. All of these
E. None of these

Answer & Explanation

C. Negotiated Dealing System


Explanation:
Negotiated Dealing System(NDS) is an electronic trading platform,
operated by the Reserve Bank of India, used to facilitate the exchange
of government securities and other money market instruments.

6. Which of the following was set up in April, 2001 to provide


guaranteed clearing and settlement functions for transactions
in Money, G-Secs, Foreign Exchange and Derivative markets?
A. SHCIL
B. SEBI
C. CIBIL
D. CCIL
E. All of these

Answer & Explanation

D. CCIL
Explanation:
The Clearing Corporation of India Ltd. (CCIL) was set up in April, 2001
to provide guaranteed clearing and settlement functions for
transactions in Money, G-Secs, Foreign Exchange and Derivative
markets.

www.bankingpdf.com
7. What types of trades are settled through CCIL?
A. Treasury Bills
B. Central Government Securities
C. State Government Securities
D. STRIPS
E. All of these

Answer & Explanation

E. All of these
Explanation:
Treasury Bills
Central Government Securities
State Government Securities
STRIPS

8. ________ refers to the period during which no settlements are


permitted in a security.
A. Short Period
B. Long Period
C. Closed Period
D. Shut Period
E. None of these

Answer & Explanation

D. Shut Period
Explanation:
Shut period refers to the period during which no settlements are
permitted in a security.

9. CBLO Stands for _________


A. Collective Borrowing and Lending Obligation
B. Collateralized Borrowing and Lending Organization
C. Collective Borrowers and Lenders Obligation

www.bankingpdf.com
D. Collateralized Borrowing and Lending Obligation
E. Collective Borrowers and Lenders Organization

Answer & Explanation

D. Collateralized Borrowing and Lending Obligation


Explanation:
Collateralized Borrowing and Lending Obligation(CBLO) facilitates in a
collateralized environment, borrowing and lending of funds to market
participants who are admitted as members in CBLO Segment.

10. What is the mode of settlement through CCIL?


A. Delivery Versus Payment
B. Payment Versus Payment
C. Payment Versus Delivery
D. Payment and Settlement
E. All of these

Answer & Explanation

B. Payment Versus Payment


Explanation:
CCIL adopts the Payment versus payment mode of Settlement.

20-30 questions :

1. Which of the following is the dimension of new 500 rupee note?


A. 157 × 73 mm
B. 66 × 150 mm
C. 66 × 166 mm
D. 147 × 73 mm
E. None of these

Answer & Explanation

www.bankingpdf.com
B. 66 × 150 mm
Explanation:
The dimension of new Indian 500-rupee banknote is 66 × 150 mm
Width – 150mm & Height – 66mm

2. Which of the following is the dimension of 2000 rupee note?


A. 157 × 73 mm
B. 66 × 150 mm
C. 66 × 166 mm
D. 147 × 73 mm
E. None of these

Answer & Explanation

C. 66 × 166 mm
Explanation:
The dimension of Indian 2000-rupee banknote is 66 × 166 mm
Width – 166mm & Height – 66mm

3. Which of the following is the main color of 2000 rupee note?


A. Stone Grey
B. Green
C. Orange – Violet
D. Magenta
E. Red – Orange

Answer & Explanation

D. Magenta
Explanation:
The main color of 2000 rupee note is Magenta

4. Which of the following is the main color of new 500 rupee note?
A. Stone Grey
B. Green

www.bankingpdf.com
C. Orange – Violet
D. Magenta
E. Red – Orange

Answer & Explanation

A. Stone Grey
Explanation:
The main color of 2000 rupee note is Stone Grey

5. Which of the following features on the reverse of 2000 rupee


note?
A. Mangalyaan
B. Himalaya Mountains
C. Red Fort
D. Parliament of India
E. Tractor

Answer & Explanation

A. Mangalyaan
Explanation:
The reverse side features Mangalyaan, India’s first interplanetary space
mission

6. Which of the following features on the reverse of 500 rupee


note?
A. Mangalyaan
B. Himalaya Mountains
C. Red Fort
D. Parliament of India
E. Tractor

Answer & Explanation

www.bankingpdf.com
C. Red Fort
Explanation:
The reverse side features a motif of the Indian heritage site of Red Fort

7. When the note is tilted, color of the thread changes from


_______
A. red to green
B. green to blue
C. blue to green
D. green to grey
E. None of the Above

Answer & Explanation

B. green to blue
Explanation:
New rupee note features windowed security thread with inscriptions
‘भारत’, RBI and 500 on banknotes with colour shift. Colour of the thread
changes from green to blue when the note is tilted.

8. Which of the following features on the reverse of both 500 and


2000 rupee notes?
A. Mangalyaan
B. Himalaya Mountains
C. Red Fort
D. PMJDY
E. Swachh Bharat Abhiyan

Answer & Explanation

E. Swachh Bharat Abhiyan


Explanation:
The logo and a tag line of Swachh Bharat Abhiyan features on the
reverse of both 500 and 2000 rupee notes

www.bankingpdf.com
9. The obverse side of 500 and 2000 rupee note features a portrait
of Mahatma Gandhi as well as the ________
A. Mangalyaan
B. Parliament
C. Red Fort
D. Ashoka Pillar Emblem
E. Swachh Bharat Abhiyan

Answer & Explanation

D. Ashoka Pillar Emblem


Explanation:
The obverse side of 500 and 2000 rupee note features a portrait of
Mahatma Gandhi as well as the Ashoka Pillar Emblem.

10. Year of printing of the note present on the ______ side of


the bank note(reverse)
A. bottom
B. right
C. left
D. All of these
E. None of these

Answer & Explanation

C. left
Explanation:
Year of printing of the note on the left (reverse).

30-40 questions

www.bankingpdf.com
1. What are the services available under IMPS?
A. Funds Transfer and Remittances
B. Merchant Payments
C. NUUP
D. Both (A) and (B)
E. All of the Above

Answer & Explanation

E. All of the Above


Explanation:
The services available under IMPS are
Funds Transfer and Remittances
Merchant Payments
NUUP
QSAM

2. What is the full form of NUUP?


A. National Unified USSD Protocol
B. National Unitied USSD Platform
C. National Unitied USSD Protocol
D. National Unified USP Platform
E. National Unified USSD Platform

Answer & Explanation

E. National Unified USSD Platform


Explanation:
NUUP – National Unified USSD Platform

3. Which of the following is a USSD based mobile banking service


from NPCI that brings together all the Banks and Telecom
Service Providers?.
A. WAP
B. QSAM

www.bankingpdf.com
C. NUUP
D. Both (A) and (B)
E. Both (B) and (C)

Answer & Explanation

C. NUUP
Explanation:
NUUP (National Unified USSD Platform) is a USSD based mobile
banking service from NPCI that brings together all the Banks and
Telecom Service Providers.

4. In NUUP, a customer can access banking services by just


pressing ________ from his/her mobile phones.
A. *999#
B. #*99#
C. *99#
D. 1*99#
E. None of the Above

Answer & Explanation

C. *99#
Explanation:
In NUUP, a customer can access banking services by just pressing
*99# from his/her mobile phones. This service works across all GSM
mobile handsets.

5. What is the full form of QSAM?


A. Quality Service on Aadhaar Mapper
B. Query Sending on Aadhaar Mapping
C. Query Service on Aadhaar Mapper
D. Quality Service on Aadhaar Mapping
E. Query Sending on Aadhaar Mapper

www.bankingpdf.com
Answer & Explanation

C. Query Service on Aadhaar Mapper


Explanation:
Query Service on Aadhaar Mapper

6. _________ service helps user in knowing their Aadhaar


Seeding status with their bank account.
A. WAP
B. QSAM
C. NUUP
D. Both (A) and (B)
E. Both (B) and (C)

Answer & Explanation

B. QSAM
Explanation:
QSAM (Query Service on Aadhaar Mapper) – This service helps user in
knowing their Aadhaar Seeding status with their bank account

7. QSAM service can be availed by dialling ________


A. *999#
B. *99*99#
C. *99#
D. 1*99#
E. None of the Above

Answer & Explanation

B. *99*99#
Explanation:
This service can be availed by dialling *99*99#. User will know
whether his/her AADHAAR number is seeded/linked to any bank

www.bankingpdf.com
account number or not. If yes, then with which bank and when it was
last updated.

8. How many digits are there in Mobile Money Identifier(MMID)?


A. 7 digits
B. 9 digits
C. 10 digits
D. 11 digits
E. None of the Above

Answer & Explanation

A. 7 digits
Explanation:
Mobile Money Identifier is a 7 digit number, issued by banks. MMID is
one of the input which when clubbed with mobile number facilitates
fund transfer. Combination of Mobile no. & MMID is uniquely linked
with an Account number and helps in identifying the beneficiary details.
Different MMID’s can be linked to same Mobile Number.

9. A customer can link how many account to the same mobile


number?
A. Only One
B. Only Two
C. Only Three
D. More than One
E. None of the Above

Answer & Explanation

D. More than One


Explanation:
customer can link more than one account to the same mobile number.
However each A/C no will have different MMID.

www.bankingpdf.com
10. An unbanked customer can initiate IMPS transaction using
the services of _______.
A. PPI
B. PCI
C. PFI
D. PDI
E. None of the Above

Answer & Explanation

A. PPI
Explanation:
An unbanked customer can initiate IMPS transaction using the services
of Pre-Paid Payments instrument issuer (PPI).

40-50 Questions

1. Which of the following is the Slogan of UPI(Unified Payment


Interface)?
A. Less Amount More Digital
B. Less Cash More Transaction
C. Less Cashflow More Digital
D. Less Circulation More Digital
E. Less Cash More Digital

Answer & Explanation

E. Less Cash More Digital


Explanation:
Less cash more digital is the Slogan of UPI(Unified Payment Interface)

2. Which of the following is the parent organisation of UPI?


A. RBI
B. GOI
C. NPCI

www.bankingpdf.com
D. All of the Above
E. None of the Above.

Answer & Explanation

C. NPCI
Explanation:
National Payments Corporation of India(NPCI) is the parent
organisation of UPI(Unified Payment Interface)

3. Customers using the UPI system only share _____


A. Bank Account Number
B. IFSC Code
C. Virtual Address
D. All of the Above
E. None of the Above.

Answer & Explanation

C. Virtual Address
Explanation:
Customers using the system do not have to disclose any sensitive
information such as bank account numbers or IFSC codes during a
financial transaction. It is safe as the customers only share a virtual
address and provide no other sensitive information.

4. What is the upper transaction limit of UPI system?


A. 1 lakh
B. 2 lakh
C. 5 lakh
D. 10 lakh
E. No limit

Answer & Explanation

www.bankingpdf.com
A. 1 lakh
Explanation:
The upper transaction limit of UPI is Rs.1 lakh.

5. Which of the following is the format of Virtual Address in UPI?


A. [name] @ [bank name] B. [Phone number] @ [bank name] C.
[name] @ [bank branch name] D. Both (A) and (B)
E. Both (B) and (C)

Answer & Explanation

D. Both (A) and (B)


Explanation:
Instead of account details, the receiver can share a virtual address,
which can be your name, or your phone number and the sender can
transfer money. For example, if your name is abc, the your virtual
address could be abc@axisbank or abc@icicibank; if your phone
number is 1234567890, then your virtual address could
be1234567890@axisbank or 1234567890@icicibank and so on.

6. In UPI, we can Send money to any bank account through


mobile without entering ____________
A. debit/credit card details
B. net banking/wallet password
C. receiver’s account number.
D. All of the Above
E. None of the Above.

Answer & Explanation

D. All of the Above


Explanation:
Send money up to Rs 1 lakh to any bank account through the mobile
without entering debit/credit card details, net banking/wallet password
or even knowing the receiver’s account number.

www.bankingpdf.com
7. Which of the following banks are on-boarded as issuers?
A. IDBI Bank
B. RBL Bank
C. Karnataka Bank
D. Both (A) and (B)
E. Both (B) and (C)

Answer & Explanation

D. Both (A) and (B)


Explanation:
IDBI Bank and RBL Bank are on-boarded as issuers. It enables their
customers to download any UPI enabled Apps mentioned above and
link their account.

8. Unified Payment System (UPI), a common platform which links


your _________ to a virtual payment address, effectively
making money transfer as easy as an SMS.
A. Bank Account Number
B. IFSC Code
C. Virtual Address
D. All of the Above
E. None of the Above.

Answer & Explanation

A. Bank Account Number


Explanation:
Unified Payment System (UPI), a common platform which links your
bank account number to a virtual payment address, effectively making
money transfer as easy as an SMS.

9. UPI will eliminate the need of entering ________ details


A. Bank Account Number
B. IFSC Code

www.bankingpdf.com
C. CVV code
D. All of the Above
E. None of the Above.

Answer & Explanation

D. All of the Above


Explanation:
UPI can be used for shopping at the supermarket, or online shopping
as well. It will eliminate the need of entering card details such as
number, expiry date, CVV code and OTP.

10. What are the important instruments under Indian Stamp


Act, 1899?
A. bill of exchange
B. power of Attorney
C. policy of insurance
D. None of the Above
E. All of the Above

Answer & Explanation

E. All of the Above


Explanation:
Important Instruments are agreements, conveyances, exchange, gift,
Certificate of sale, deed of partition, Power of Attorney to sell
immovable property when given for consideration, deed of settlement
and transfer of lease by way of assignment, bill of exchange, bill of
lading, debenture, letter of credit, policy of insurance, proxy, receipt
and transfer of shares.

www.bankingpdf.com
50-60 Questions

1. Which of the following is/are feature(s) of e-stamping?


A. Cost Savings
B. Security
C. User-friendly
D. All of the Above
E. None of the Above

Answer & Explanation

D. All of the Above


Explanation:
Features:
Easy accessibility and faster processing
Security
Cost savings
User friendly

2. Which of the following can be used to check the authenticity of


the Certificate?
A. PAN Number
B. Unique Identification Number
C. A/C Number
D. All of the Above
E. None of the Above

Answer & Explanation

B. Unique Identification Number


Explanation:
UIN is a Unique system generated number mentioned on the e-Stamp
Certificate. Anybody, having the Unique Identification Number, can
check the authenticity of the Certificate

www.bankingpdf.com
3. What are the different modes of paying stamp duty in e-
Stamping system?
A. RTGS
B. NEFT
C. Pay order
D. All of the Above
E. None of the Above

Answer & Explanation

D. All of the Above


Explanation:
Client can pay stamp duty amount through the following modes
Cash
Cheque
Demand Draft
Pay Order
RTGS
NEFT
Account to Account transfer.

4. In which States/UT s e-Stamping is currently operational?


A. Gujarat
B. Karnataka
C. Punjab
D. All of the Above
E. None of the Above

Answer & Explanation

D. All of the Above


Explanation:
e-Stamping is currently operational in the States/UT s of Gujarat,
Karnataka, NCT Delhi, Assam, Tamil Nadu, Rajasthan, Himachal
Pradesh, Uttarakhand, UT of Dadra & Nagar Haveli, UT of Daman &

www.bankingpdf.com
Diu, Puducherry, Uttar Pradesh, Chhattisgarh, Jharkhand, Jammu &
Kashmir, Punjab and Chandigarh.

5. Which of the following is a computer based application and a


secured way of paying Non-Judicial stamp duty to the
Government?
A. Commercial Papers
B. E-stamp
C. Treasury bill
D. All of the Above
E. None of the Above

Answer & Explanation

B. E-stamp
Explanation:
e-Stamping is a computer based application and a secured way of
paying Non-Judicial stamp duty to the Government.

6. Stamp Duty collected by the States can be broadly divided into


_______ categories
A. One
B. Two
C. Three
D. All of the Above
E. None of the Above

Answer & Explanation

B. Two
Explanation:
Stamp Duty collected by the States can be broadly divided into two
categories, viz., Stamp Duty paid under the Indian Stamp Act, 1899
and Stamps used in payment of fees under the Court-fees Act 1870.

www.bankingpdf.com
7. Which of the following details can be used to verify the
authenticity of an e-Stamp?
A. Certificate Number (UIN)
B. Stamp Duty Type (Description of Document)
C. 6 character alphanumeric string
D. All of the Above
E. None of the Above

Answer & Explanation

D. All of the Above


Explanation:
An e-Stamp can be verified online by clicking on verify e-Stamp
certificate and entering the required details i.e
State
Certificate Number (UIN)
Stamp Duty Type (Description of Document)
Certificate Issue Date
6 character alphanumeric string

8. Which of the following payment should be initiated only after


consulting nearest e-Stamping center?
A. RTGS
B. NEFT
C. Pay order
D. Both (A) and (B)
E. Both (B) and (C)

Answer & Explanation

D. Both (A) and (B)


Explanation:
RTGS / NEFT payment should be initiated only after consulting nearest
e-Stamping center.

www.bankingpdf.com
9. Stamps used under the Indian Stamp Act, 1899 & The Bombay
Stamp Supply And Sale Rules, 1934, can be broadly divided into
_______ categories.
A. One
B. Two
C. Three
D. All of the Above
E. None of the Above

Answer & Explanation

B. Two
Explanation:
Stamps used under the Indian Stamp Act, 1899 & The Bombay Stamp
Supply And Sale Rules, 1934, can be broadly divided into two
categories.
Impressed stamps
Adhesive stamps.

10. SHCIL will accept Stamp Duty Payment in ______ only


A. US$
B. Euro
C. INR
D. None of the Above
E. All of the Above

Answer & Explanation

C. INR
Explanation:
SHCIL will accept Stamp Duty Payment in Indian Rupees (INR) only.

60-70 Questions :

www.bankingpdf.com
1. What is the fullform of eSBTR?
A. Electronic Secured Basic Transaction Receipt
B. Electronic Stamp Bank Treasury Receipt
C. Electronic Stamp Bank Transaction Receipt
D. Electronic Secured Bank Treasury Receipt
E. Electronic Secured Basic Treasury Receipt

Answer & Explanation

D. Electronic Secured Bank Treasury Receipt


Explanation:
e-SBTR, is an Electronic – Secured Bank and Treasury Receipt, issued
by the Officer of the participating bank, on secured Government
stationery, on payment of Stamp- duty in Virtual Treasury.

2. Which of the following is the country’s largest custodian and


depository participant?
A. CIBIL
B. SEBI
C. NABARD
D. SHCIL
E. None of the Above

Answer & Explanation

D. SHCIL
Explanation:
Stock Holding Corporation of India Ltd (SHCIL), India’s largest
custodian and depository participant.

3. The Headquarters of SHCIL is located in ___________


A. Hyderabad
B. Mumbai
C. Chennai

www.bankingpdf.com
D. New Delhi
E. None of the Above

Answer & Explanation

B. Mumbai
Explanation:
The Headquarters of SHCIL is located in Mumbai, Maharashtra.

4. Which of the following is the only Central Record Keeping


Agency (CRA) appointed by the Government of India?
A. CIBIL
B. SEBI
C. NABARD
D. SHCIL
E. None of the Above

Answer & Explanation

D. SHCIL
Explanation:
SHCIL is appointed as Central Record-keeping Agency (CRA) and
associated with stamp duty collection and not valuation.

5. Stock Holding Corporation of India Limited is owned by?


A. SU-UTI
B. NIA
C. GIC
D. LIC
E. All of the Above

Answer & Explanation

E. All of the Above


Explanation:
Stock Holding Corporation of India Limited(SHCIL) is owned by the

www.bankingpdf.com
India’s leading Banks and Financial Institutions such as, SU-UTI, IFCI
Ltd., LIC, GIC, NIA, NIC, UIC, and TOICL.

6. SHCIL was established in 1986 under the Government of India


as __________
A. Private Ltd
B. Public Ltd
C. Limited Liability Partnership
D. Unlimited company
E. None of the Above

Answer & Explanation

B. Public Ltd
Explanation:
Stock Holding Corporation of India Limited(SHCIL) was established in
1986 under the Government of India as public limited company.

7. Minimum value of transaction for e-SBTR should be _______


A. Rs.1000
B. Rs.2000
C. Rs.3000
D. Rs.5000
E. No limit

Answer & Explanation

D. Rs.5000
Explanation:
Minimum value of transaction for e-SBTR should be Rs.5,000.

8. Maximum value of transaction for e-SBTR should be _________


A. Rs.1000
B. Rs.2000
C. Rs.3000

www.bankingpdf.com
D. Rs.5000
E. No limit

Answer & Explanation

E. No limit
Explanation:
There is no maximum limit.

9. Authorised Collection Center(ACC) is an agent appointed by


SHCIL?
A. SU-UTI
B. GOI
C. GIC
D. SHCIL
E. All of the Above

Answer & Explanation

D. SHCIL
Explanation:
ACC means Authorised Collection Center (ACC). Its an agent appointed
by SHCIL. ACC is the intermediary between the CRA and Stamp Duty
payer.

10. Which of the following can become ACCs?


A. Scheduled Banks
B. Insurance Majors
C. Post Offices
D. Both (A) and (B)
E. Both (A) and (C)

Answer & Explanation

www.bankingpdf.com
E. Both (A) and (C)
Explanation:
Scheduled Banks and Post Offices can become ACCs.

70-80 Questions :

1. ________ is a temporary pass through account held by a third


party during the process of a transaction between two parties.
A. Savings Bank Account
B. Current Account
C. Deposit Account
D. Escrow Account
E. None of the Above

Answer & Explanation

D. Escrow Account
Explanation:
Escrow Account: Account in which funds are accumulated for specific
disbursements.
It is a temporary pass through account held by a third party during the
process of a transaction between two parties.

2. Which of the following is not available to depositors of NBFCs?


A. Deposit Insurance facility
B. Acquisition of shares
C. Loans and Advances
D. All of the Above
E. None of the Above

Answer & Explanation

A. Deposit Insurance facility


Explanation:
Deposit insurance facility of Deposit Insurance and Credit Guarantee

www.bankingpdf.com
Corporation is not available to depositors of NBFCs, unlike in case of
banks.

3. NBFCs include ______


A. loan company
B. investment company
C. asset finance company
D. All of the Above
E. None of the Above

Answer & Explanation

D. All of the Above


Explanation:
NBFCs include a loan company, an investment company, asset finance
company (i.e. a company conducting the business of equipment leasing
or hire purchase finance) and Residuary Non-Banking Companies.

4. Infrastructure Finance Company (IFC) is a non-banking finance


company has a minimum Net Owned Funds of _______
A. Rs.100 Crore
B. Rs.200 Crore
C. Rs.500 Crore
D. Rs.300 Crore
E. None of the Above

Answer & Explanation

D. Rs.300 Crore
Explanation:
Infrastructure Finance Company (IFC) is a non-banking finance
company has a minimum Net Owned Funds of Rs.300 Crore.

5. NBFCs whose asset size is of _____ or more as per last audited


balance sheet are considered as systemically important NBFCs.

www.bankingpdf.com
A. Rs.100 Crore
B. Rs.200 Crore
C. Rs.500 Crore
D. All of the Above
E. None of the Above

Answer & Explanation

C. Rs.500 Crore
Explanation:
NBFCs whose asset size is of Rs.500 Crore or more as per last audited
balance sheet are considered as systemically important NBFCs.

6. Which of the following does not guarantee the repayment of


deposits accepted by NBFCs?
A. RBI
B. GOI
C. SEBI
D. NABARD
E. None of the Above

Answer & Explanation

A. RBI
Explanation:
RBI does not guarantee the repayment of deposits accepted by NBFCs.

7. RNBC stands for _______


A. Residuary Non Banking Companies
B. Reserve Non Banking Companies
C. Reserve New Banking Companies
D. Residuary New Banking Companies
E. None of the Above

Answer & Explanation

www.bankingpdf.com
A. Residuary Non Banking Companies
Explanation:
RNBC – Residuary non banking companies. Residuary Non-Banking
Company is a class of NBFC which is a company and has as its principal
business the receiving of deposits, under any scheme or arrangement
or in any other manner and not being investment, asset financing, loan
company.

8. RNBCs cannot accept deposits for a period less than ______


months and more than ____ months
A. 12, 84
B. 12, 60
C. 24, 60
D. 24, 84
E. None of the Above

Answer & Explanation

A. 12, 84
Explanation:
RNBCs cannot accept deposits for a period less than 12 months and
more than 84 months.

9. According to accepting public deposits, NBFCs can be classified


into ____ broad categories.
A. Two
B. Three
C. Four
D. All of the Above
E. None of the Above

Answer & Explanation

A. Two
Explanation:

www.bankingpdf.com
According to accepting public deposits, NBFCs can be classified into two
broad categories, viz.,
(i) NBFCs accepting public deposit (NBFCs-D)
(ii) NBFCs not accepting/holding public deposit (NBFCs-ND).

10. IFC is a non-banking finance company with Capital to Risk


(Weighted) Assets Ratio (CRAR) of _____
A. 15%
B. 20%
C. 25%
D. 30%
E. None of the Above

Answer & Explanation

A. 15%
Explanation:
IFC is a non-banking finance company with Capital to Risk (Weighted)
Assets Ratio (CRAR) of 15%.

80-90 Questions :

1. A fund that tracks an index but can be traded like a stock is


known as _______
A. Exchange Traded Funds(ETF)
B. Mutual Funds
C. Energy Funds
D. Investment Fund
E. None of the Above

Answer & Explanation

A. Exchange Traded Funds(ETF)


Explanation:
An ETF(Exchange Traded Fund) is a marketable security that tracks an

www.bankingpdf.com
index, a commodity, bonds, or a basket of assets like an index
fund. Unlike mutual funds, an ETF trades like a common stock on a
stock exchange.

2. ________ are financial markets for the buying and selling of


long-term debt or equity-backed securities.
A. Bullion Market
B. Capital Market
C. Money Market
D. All of the Above
E. None of the Above

Answer & Explanation

B. Capital Market
Explanation:
Capital market is a market where buyers and sellers engage in trade of
financial securities like bonds, stocks, etc

3. A market established for the buying and selling of precious


metals such as Gold and Silver is termed as ________
A. Bullion Market
B. Capital Market
C. Money Market
D. All of the Above
E. None of the Above

Answer & Explanation

A. Bullion Market
Explanation:
A market established for the buying and selling of precious metals such
as Gold and Silver is termed as Bullion Market.

www.bankingpdf.com
4. Which of the following is a currency established as money by
government regulation or law but it is not backed by any
commodity, such as gold, silver etc., but only by the faith of the
bearer?
A. Commodity Money
B. Representative Money
C. Fiat Money
D. fiduciary currency
E. None of the Above

Answer & Explanation

C. Fiat Money
Explanation:
Fiat money is currency that a government has declared to be legal
tender, but it is not backed by a physical commodity.

5. Which of the following is a form of money that has an intrinsic


value, like gold coins?
A. Commodity Money
B. Representative Money
C. Fiat Money
D. fiduciary currency
E. None of the Above

Answer & Explanation

A. Commodity Money
Explanation:
Commodity Money is a form of currency in which the value of the
currency comes from the material of which it is made.

6. Paper Money issued on the credit of a bank or government is


called ______
A. Commodity Money

www.bankingpdf.com
B. Representative Money
C. Fiat Money
D. Fiduciary currency
E. None of the Above

Answer & Explanation

D. Fiduciary currency
Explanation:
Fiduciary Money that depends for its value on confidence that it is an
accepted medium of exchange. It originated as a paper certificate that
was a promise to pay a certain amount of gold or silver to the bearer.

7. Any type of money that has face value greater than its value as
material substance is known as _______
A. Commodity Money
B. Representative Money
C. Fiat Money
D. fiduciary currency
E. None of the Above

Answer & Explanation

B. Representative Money
Explanation:
A claim on a commodity, for example gold certificates or silver
certificates. Any type of money that has face value greater than its
value as material substance is called Representative Money.

8. Loan and Advances of a bank come under which of the following


category?
A. Assets
B. Liabilities
C. Deposits

www.bankingpdf.com
D. All of the Above
E. None of the Above

Answer & Explanation

A. Assets
Explanation:
Anything tangible or intangible that can be owned or controlled to
produce value and that is held to have positive economic value is
considered an asset. Anything tangible or intangible that can be owned
or controlled to produce value and that is held to have positive
economic value is considered an asset.

9. NABARD was established on the recommendations of the


______________
A. Public Accounts Committee
B. Shivaraman Committee
C. Narasimham Committee
D. All of the Above
E. None of the Above

Answer & Explanation

B. Shivaraman Committee
Explanation:
NABARD was established on the recommendations of Shivaraman
Committee, (by Act 61, 1981 of Parliament) on 12 July 1982 to
implement the National Bank for Agriculture and Rural Development
Act 1981.

10. RBI sold its stake in NABARD to the Government of India,


which now holds _____ stake
A. 65%
B. 70%
C. 95%

www.bankingpdf.com
D. 99%
E. None of the Above

Answer & Explanation

D. 99%
Explanation:
RBI sold its stake in NABARD to the Government of India, which now
holds 99% stake.

90-100 Questions :

1. Currency swap is an instrument to manage _______


A. Interest Rate Risk
B. Currency Risk
C. Cash flows in different countries
D. All of the Above
E. None of the Above

Answer & Explanation

C. Cash flows in different countries


Explanation:
A currency swap (or a cross currency swap) is a foreign exchange
derivative between two institutions to exchange the principal and/or
interest payments of a loan in one currency for equivalent amounts, in
net present value terms, in another currency.

2. “Hawala transactions” in India are prohibited under the


provision of which of the following acts?
A. Banking Regulation Act
B. Financial Action Task Force Act
C. Foreign Exchange Management Act
D. All of the Above
E. None of the above

www.bankingpdf.com
Answer & Explanation

C. Foreign Exchange Management Act


Explanation:
“Hawala transactions” in India are prohibited under the provision of
Foreign Exchange Management Act.

3. Which of the following refers to an account that a bank holds in


a foreign currency in another bank?
A. Nostro Account
B. Vostro Account
C. LORO Account
D. Mirror Account
E. None of the Above

Answer & Explanation

A. Nostro Account
Explanation:
Nostro account refers to an account that a bank holds in a foreign
currency in another bank.

4. Local currency account maintained by a local bank for a foreign


(correspondent) bank is termed as _________
A. Nostro Account
B. Vostro Account
C. LORO Account
D. Mirror Account
E. None of the Above

Answer & Explanation

B. Vostro Account
Explanation:

www.bankingpdf.com
Local currency account maintained by a local bank for a foreign
(correspondent) bank is termed as Vostro Account.

5. ____is a Current Account Maintained by one Domestic Bank on


behalf of other domestic bank in foreign bank in foreign
currency
A. BSBDA
B. CA
C. LORO Account
D. Mirror Account
E. None of the Above

Answer & Explanation

C. LORO Account
Explanation:
LORO Account is a Current Account Maintained by one Domestic Bank
on behalf of other domestic bank in foreign bank in foreign currency.

6. How many Basic Savings Bank Deposit Account(BSBDA) can be


opened by an individual?
A. One
B. Two
C. Three
D. Four
E. None of the Above

Answer & Explanation

A. One
Explanation:
An individual is eligible to have only one ‘Basic Savings Bank Deposit
Account’ in one bank.

www.bankingpdf.com
7. What are the other deposit accounts can be opened by an
individual in where one holds ‘Basic Savings Bank Deposit
Account’?
A. Fixed Deposit
B. Recurring Deposit
C. Another BSBDA
D. Both A and B
E. None of the Above

Answer & Explanation

D. Both A and B
Explanation:
One can have Term/Fixed Deposit, Recurring Deposit etc., accounts in
the bank where one holds ‘Basic Savings Bank Deposit Account.

8. Maximum balance in the BSBDA small account should not


exceed ______ rupees at any time
A. 50000
B. 100000
C. 200000
D. 300000
E. None of the Above

Answer & Explanation

A. 50000
Explanation:
Maximum balance in the BSBDA small account should not exceed fifty
thousand rupees at any time.

9. The total of debits by way of cash withdrawals and transfers


will not exceed ten thousand rupees in _________
A. One month
B. Two month

www.bankingpdf.com
C. Three month
D. All of the Above
E. None of the Above

Answer & Explanation

A. One month
Explanation:
The total of debits by way of cash withdrawals and transfers will not
exceed ten thousand rupees in a month for BSBDA small accounts.

10. BSBDA Small accounts are valid for a period of _________


A. 3 months
B. 4 months
C. 12 months
D. 6 months
E. None of the Above

Answer & Explanation

C. 12 months
Explanation:
Small accounts are valid for a period of 12 months initially which may
be extended by another 12 months if the person provides proof of
having applied for an Officially Valid Document.

100-110 Questions :

1. SEBI is a/an ________


A. Constitutional body
B. Advisory body
C. non-statutory body
D. Statutory body
E. None of the Above

Answer & Explanation

www.bankingpdf.com
D. Statutory body
Explanation:
Initially SEBI was a non statutory body without any statutory power.
However, in 1995, the SEBI was given additional statutory power by
the Government of India through an amendment to the Securities and
Exchange Board of India Act, 1992.

2. Which of the following policies is known as Annual Policy


Statement?
A. Annual budget of central government
B. Credit and Monetary Policy of RBI
C. Foreign trade policy of DGFT
D. Regulations issued by SEBI
E. None of the Above

Answer & Explanation

B. Credit and Monetary Policy of RBI


Explanation:
Credit and Monetary Policy of RBI is known as Annual Policy
Statement.

3. Which of the following is the Regulator of the credit rating


agencies in India ?
A. RBI
B. SEBI
C. SIDBI
D. GOI
E. None of the Above

Answer & Explanation

B. SEBI
Explanation:
The capital market regulator regulates rating agencies in most regions.

www.bankingpdf.com
In India, the capital markets regulator, the Securities and Exchange
Board of India (SEBI), regulates the rating agencies in the country.

4. Securities Appellate Tribunal is a/an __________ established


under the provisions of Section 15K of the Securities and
Exchange Board of India Act, 1992
A. Constitutional body
B. Advisory body
C. non-statutory body
D. Statutory body
E. None of the Above

Answer & Explanation

D. Statutory body
Explanation:
Securities Appellate Tribunal is a statutory body established under the
provisions of Section 15K of the Securities and Exchange Board of
India Act, 1992 to hear and dispose of appeals against orders passed
by the Securities and Exchange Board of India.

5. Which of the following is a function of SEBI?


A. to approve by−laws of stock exchanges.
B. inspect the books of accounts of financial intermediaries.
C. to require the stock exchange to amend their by−laws.
D. compel certain companies to list their shares in one or more stock
exchanges.
E. All of the Above

Answer & Explanation

E. All of the Above


Explanation:
Function of SEBI:
To approve by−laws of stock exchanges.

www.bankingpdf.com
Inspect the books of accounts of financial intermediaries.
To require the stock exchange to amend their by−laws.
Compel certain companies to list their shares in one or more stock
exchanges.

6. The Monetary Authority in India, viz Reserve Bank of India is


bound to maintain a reserve against the notes issued, Whatever
may be the mount. This system is called as __________
A. Minimum Reserve System
B. Proportional Reserve System
C. Maximum Fiduciary Issue system
D. Simple deposit system
E. None of the Above

Answer & Explanation

A. Minimum Reserve System


Explanation:
The RBI follows a minimum reserve system in the note issue. Initially,
it used to keep 40 per cent of gold reserves in its total assets.

7. Which of the following services is NOT provided by the post


offices in India ?
A. Savings Bank Scheme
B. Retailing of Mutual Funds
C. Sale of stamp Papers (Judicial)
D. Issuance of Demand Drafts
E. Life Insurance cover

Answer & Explanation

D. Issuance of Demand Drafts


Explanation:
Issuance of Demand Drafts is NOT provided by the post offices in
India.

www.bankingpdf.com
8. Credit rating _______
A. is used to rate the borrowers while giving advances
B. is used to work out performance of the employees
C. is used to calculate the number of excellent audit rated branches
D. is not used in any bank
E. is necessary before giving promotion to employees

Answer & Explanation

A. is used to rate the borrowers while giving advances


Explanation:
Credit rating is an analysis of the credit risks associated with a financial
instrument or a financial entity. An assessment of the creditworthiness
of a borrower with respect to a particular debt or financial obligation.

9. The section 4 of the Negotiable Instruments Act, 1881 deals


with ________
A. Cheque
B. Bills of Exchange
C. Promissory Note
D. All of the Above
E. None of the Above

Answer & Explanation

C. Promissory Note
Explanation:
The section 4 of the Negotiable Instruments Act, 1881 deals with
Promissory Note.

10. Which of the following section of Negotiable Instruments


Act, 1881 deals with Cheques?
A. Section 4
B. Section 5
C. Section 6

www.bankingpdf.com
D. Section 7
E. None of the Above

Answer & Explanation

C. Section 6
Explanation:
The section 6 of the Negotiable Instruments Act, 1881 deals with
Cheques.

110-120 Questions:

1. Small finance banks are a type of ______ banks in our country.


A. niche
B. investment
C. industrial
D. All of the Above
E. None of the Above

Answer & Explanation

A. niche
Explanation:
Small finance banks are a type of niche banks in India.Niche Bank is a
specialised bank which deals only with certain types of customers or
services.

2. Which of the following service(s) is/are available in small


finance banks?
A. Providing loans
B. Acceptance of deposits
C. Setting up of Subsidiaries
D. Both (A) and (B)
E. None of the Above

Answer & Explanation

www.bankingpdf.com
D. Both (A) and (B)
Explanation:
Banks with a small finance bank license can provide basic banking
service of acceptance of deposits and lending.

3. According to RBI Regulations, which of the following are


allowed to set up small finance banks?
A. NBFC
B. MFI
C. LAB
D. All of the Above
E. None of the Above

Answer & Explanation

D. All of the Above


Explanation:
The firms must have a capital of Rs.100 crore.Existing non-banking
financial companies (NBFC), micro-finance institutions (MFI) and local
area banks (LAB) are allowed to set up small finance banks.

4. Reserve Bank of India gave provisional licences to _______


entities to launch small finance banks.
A. 10
B. 11
C. 12
D. 13
E. None of the Above

Answer & Explanation

A. 10
Explanation:
Reserve Bank of India gave provisional licences to 10 entities out of 72
applicants to launch small finance banks.

www.bankingpdf.com
The selected applicants are: Au Financiers (Jaipur), Capital Local Area
Bank (Jalandhar), Disha Microfin (Ahmedabad), Equitas Holdings
(Chennai), ESAF Microfinance and Investments (Chennai), Janalakshmi
Financial Services (Bengaluru), RGVN (Northeast) Microfinance
(Guwahati), Suryoday Micro Finance (Navi Mumbai), Ujjivan Financial
Services (Bengaluru) and Utkarsh Micro Finance (Varanasi).

5. Minimum paid-up capital of small finance banks is _______


A. 100 crore
B. 200 Crore
C. 500 crore
D. 1000 crore
E. None of the Above

Answer & Explanation

A. 100 crore
Explanation:
Minimum paid-up capital of small finance banks is Rs.100 crore.

6. The small finance banks will be licensed under ______ of the


Banking Regulation Act, 1949.
A. Section 21
B. Section 22
C. Section 23
D. Section 24
E. None of the Above

Answer & Explanation

B. Section 22
Explanation:
The small finance banks will be licensed under Section 22 of the
Banking Regulation Act, 1949.

www.bankingpdf.com
7. An external advisory committee headed by _______ evaluated
the license applications for small finance banks.
A. NR Madhawan Menon
B. Usha Thorat
C. Nachiket Mor
D. Y.H. Melagam
E. None of the Above

Answer & Explanation

B. Usha Thorat
Explanation:
An external advisory committee headed by Usha Thorat evaluated the
license applications for small finance banks.

8. ______ percentage of Foreign share holding will be allowed in


small finance banks as per the rules for FDI in private banks in
India
A. 64%
B. 74%
C. 61%
D. 50%
E. None of the Above

Answer & Explanation

B. 74%
Explanation:
Foreign share holding will be allowed in these banks as per the rules
for FDI in private banks in India. Foreign shareholding capped at 74%
of paid capital.

9. Which of the following can be provided by small finance banks


A. Mutual Funds
B. Insurance Products

www.bankingpdf.com
C. Third Party financial products
D. All of the Above
E. None of the Above

Answer & Explanation

D. All of the Above


Explanation:
Small finance banks can distribute mutual funds, insurance products
and other simple third-party financial products.

10. Maximum loan size to a single person cannot exceed


________ of total capital funds.
A. 30%
B. 20%
C. 11%
D. 10%
E. None of the Above

Answer & Explanation

D. 10%
Explanation:
Maximum loan size to a single person cannot exceed 10% of total
capital funds; cannot exceed 15% in the case of a group.
Small finance bank can
lend 75% of their total adjusted net bank credit to priority sector
Maximum loan size would be 10% of capital funds to single borrower,
15% to a group
Minimum 50% of loans should be up to 25 lakhs.

www.bankingpdf.com
120-130 Questions :

1. The banks will be licensed as payments banks under ______ of


the Banking Regulation Act, 1949.
A. Section 21
B. Section 22
C. Section 23
D. Section 24
E. None of the Above

Answer & Explanation

B. Section 22
Explanation:
The payment banks will be licensed as payments banks under Section
22 of the Banking Regulation Act, 1949 and will be registered as public
limited company under the Companies Act, 2013.

2. Payment banks can accept a restricted deposit which is


currently limited to _______ per customer account.
A. Rs.1 lakh
B. Rs.3 lakh
C. Rs.2 lakh
D. Rs.5 lakh
E. None of the Above

Answer & Explanation

A. Rs.1 lakh
Explanation:
Payment banks can accept a restricted deposit which is currently
limited to Rs. 1 lakh per customer account.

3. Payment Banks can not issue ________


A. Credit Card

www.bankingpdf.com
B. Debit Card
C. loans
D. Both (B) and (C)
E. Both (A) and (C)

Answer & Explanation

E. Both (A) and (C)


Explanation:
Payment banks cannot issue loans and credit cards while at the same
time current account and savings accounts can be operated by such
banks.

4. Any acquisition by the Payment Banks that are more than ____
will require approval of the RBI.
A. 5%
B. 10%
C. 15%
D. 20%
E. None of the Above

Answer & Explanation

A. 5%
Explanation:
Any acquisition of more than 5% will require approval of the RBI.

5. Which of the following services can be offered by Payment


Banks?
A. Debit Cards
B. Online Banking
C. Mobile Banking
D. All of the Above
E. None of the Above

www.bankingpdf.com
Answer & Explanation

D. All of the Above


Explanation:
Payments banks can issue services like ATM cards, debit cards online
banking and mobile banking.

6. _______ percentage of Payment Bank’s branches must be in


the unbanked rural area.
A. 25%
B. 10%
C. 15%
D. 20%
E. None of the Above

Answer & Explanation

A. 25%
Explanation:
25% of Payment Bank’s branches must be in the unbanked rural area.

7. Committee on Comprehensive Financial Services for Small


Businesses and Low Income Households, headed by ________
recommended the formation of a new category of bank called
payments bank.
A. NR Madhawan Menon
B. Bimal Jalan
C. Nachiket Mor
D. Y.H. Melagam
E. None of the Above

Answer & Explanation

C. Nachiket Mor
Explanation:

www.bankingpdf.com
Committee on Comprehensive Financial Services for Small Businesses
and Low Income Households, headed by Nachiket Mor recommended
the formation of a new category of bank called payments bank.

8. Reserve Bank of India gave “in-principle” licences to _______


entities to launch payments banks.
A. 10
B. 11
C. 12
D. 13
E. None of the Above

Answer & Explanation

B. 11
Explanation:
Reserve Bank of India gave “in-principle” licences to 11 entities to
launch payments banks. They are:-
Aditya Birla Nuvo
Airtel M Commerce Services
Cholamandalam Distribution Services
Department of Posts
FINO PayTech
National Securities Depository
Reliance Industries
Dilip Shanghvi, Sun Pharmaceuticals
Vijay Shekhar Sharma, Paytm
Tech Mahindra
Vodafone M-Pesa.

9. Which of the following surrendered their “in-principle” to


launch payments banks?
A. Cholamandalam Distribution Services
B. Dilip Shanghvi, Sun Pharmaceuticals

www.bankingpdf.com
C. Tech Mahindra
D. All of the Above
E. None of the Above

Answer & Explanation

D. All of the Above


Explanation:
Three of the 11 entities have surrendered their licenses. They are
“Chalomandalam Distribution Services”, “Dilip Shanghvi, Sun
Pharmaceuticals” and “Tech Mahindra”.

10. The “in-principle” license is valid for _____months within


which the entities must fulfil the requirements.
A. 12 Months
B. 14 Months
C. 18 Months
D. 24 Months
E. None of the Above

Answer & Explanation

C. 18 Months
Explanation:
The “in-principle” license is valid for 18 months within which the
entities must fulfil the requirements.

www.bankingpdf.com
130-140 Questions :

1. As per the licensing norms of RBI, a new bank must have a


_______ capital
A. 200 Crore
B. 100 Crore
C. 1000 Crore
D. 500 Crore
E. None of the Above

Answer & Explanation

D. 500 Crore
Explanation:
The RBI licensing norms stipulate that a new bank must have Rs.500
crore capital.

2. Who among the following inaugurates Bandhan Bank as full-


fledged scheduled commercial bank?
A. Narendra Modi
B. Rajnath Singh
C. Santosh Gangwar
D. Arun Jaitley
E. None of the Above

Answer & Explanation

D. Arun Jaitley
Explanation:
Union Finance Minister Arun Jaitley officially inaugurated the bank in
Kolkata. Initially it opened with 501 branches across India.

3. Which is the first bank to be set up in eastern part of India after


Independence?
A. UCO Bank

www.bankingpdf.com
B. Allahabad Bank
C. United Bank of India
D. Bandhan Bank
E. None of the Above

Answer & Explanation

D. Bandhan Bank
Explanation:
Kolkata-headquartered Bandhan Bank is the first bank to be set up in
eastern part of India after Independence.

4. Bandhan Bank is headquartered in _______


A. Mumbai
B. Kolkata
C. Chennai
D. Bangalore
E. None of the Above

Answer & Explanation

B. Kolkata
Explanation:
Bandhan Bank is headquartered in Kolkata.

5. Bandhan Bank starts with a capital base of ______ crore


A. 2500 Crore
B. 1070 Crore
C. 2570 Crore
D. 2470 Crore
E. None of the Above

Answer & Explanation

C. 2570 Crore
Explanation:

www.bankingpdf.com
Bandhan Bank starts with a capital base of Rs. 2,570 crore and this will
be raised to Rs. 3,052 crore.

6. Which of the following is/are the loan product(s) of Bandhan


Bank?
A. Agri loan
B. MSME loan
C. Retail loan
D. Both (A) and (C)
E. All of the Above

Answer & Explanation

E. All of the Above


Explanation:
Products of Bandhan Bank includes Agri loan, MSME loan, Retail loan
and Micro loan.

7. Which of the following is the slogan of Bandhan Bank?


A. Hum Hai na
B. Aapka Bhala, Sabki Bhalai
C. Aao Sochein Bada
D. Both (A) and (C)
E. None of the Above

Answer & Explanation

B. Aapka Bhala, Sabki Bhalai


Explanation:
Aapka Bhala, Sabki Bhalai is the slogan of Bandhan Bank.

8. IDFC Bank is headquartered in _______


A. Mumbai
B. Kolkata
C. Chennai

www.bankingpdf.com
D. Bangalore
E. None of the Above

Answer & Explanation

A. Mumbai
Explanation:
IDFC Bank is headquartered in Mumbai.

9. Who among the following launched IDFC Bank?


A. Narendra Modi
B. Rajnath Singh
C. Santosh Gangwar
D. Arun Jaitley
E. None of the Above

Answer & Explanation

A. Narendra Modi
Explanation:
Prime Minister Narendra Modi formally launched IDFC Bank on 19
October 2015.

10. Which of the following is/are the product(s) of IDFC Bank?


A. Commercial Banking
B. Bharat Banking
C. Retail Banking
D. Business Banking
E. All of the Above

Answer & Explanation

E. All of the Above


Explanation:
Products of IDFC Bank are as follows
Retail banking,

www.bankingpdf.com
Commercial & Wholesale Banking,
Business Banking
Bharat Banking

140-150 Questions :

1. Which of the following is the clearing agency for Government


securities?
A. RBI
B. GOI
C. CCIL
D. SEBI
E. None of the Above

Answer & Explanation

C. CCIL
Explanation:
The CCIL is the clearing agency for Government securities. It acts as a
Central Counter Party (CCP) for all transactions in Government
securities by interposing itself between two counter parties.

2. _______ is used for electronic dealing and reporting of


transactions in government securities
A. Delivery versus Payment (DvP)
B. Negotiated Dealing System (NDS)
C. Risk Mitigation
D. All of the Above
E. None of the Above

Answer & Explanation

www.bankingpdf.com
B. Negotiated Dealing System (NDS)
Explanation:
The Negotiated Dealing System (NDS) for electronic dealing and
reporting of transactions in government securities was introduced in
February 2002.It facilitates the members to submit electronically, bids
or applications for primary issuance of Government Securities when
auctions are conducted.

3. Which of the following is the mode of settlement of securities


wherein the transfer of securities and funds happen
simultaneously?
A. Delivery versus Payment (DvP)
B. Duration
C. Convexity
D. All of the Above
E. None of the Above

Answer & Explanation

A. Delivery versus Payment (DvP)


Explanation:
Delivery versus Payment (DvP) is the mode of settlement of securities
wherein the transfer of securities and funds happen simultaneously.
This ensures that unless the funds are paid, the securities are not
delivered and vice versa. DvP settlement eliminates the settlement risk
in transactions.

4. The monetary ceiling of amount for settlement through Lok


Adalat is _________
A. 5 lakh
B. 10 lakh
C. 15 lakh
D. 20 lakh
E. None of the Above

www.bankingpdf.com
Answer & Explanation

D. 20 lakh
Explanation:
The monetary ceiling of amount for settlement through Lok Adalat is
INR 20 Lakh

5. An EEFC account can be held only in the form of a ________


A. Current Account
B. Savings Bank Account
C. loan Account
D. All of the Above
E. None of the Above

Answer & Explanation

A. Current Account
Explanation:
An EEFC(Exchange Earners’ Foreign Currency Account) account can be
held only in the form of a current account. No interest is payable on
EEFC accounts.

6. Reserve Bank of India has made it mandatory on the banks to


discontinue the practice of stapling the currency note packets
under _______ of the Banking Regulation Act, 1949
A. Section 35A
B. Section 25A
C. Section 15A
D. Section 30A
E. None of the Above

Answer & Explanation

A. Section 35A
Explanation:

www.bankingpdf.com
The Reserve Bank of India has made it mandatory on the banks to
discontinue the practice of stapling the currency note packets. It has
issued a directive to banks under Section 35A of the Banking
Regulation Act, 1949.

7. “Islamic Banking” the term used for _______


A. Covering Banking to Islamic nation
B. prohibition of the collection and payment of interest
C. Sharing of Profit and loss
D. Both (A) and (B)
E. Both (B) and (C)

Answer & Explanation

E. Both (B) and (C)


Explanation:
Two basic principles behind Islamic banking are the sharing of profit
and loss and, significantly, the prohibition of the collection and
payment of interest by lenders and investors. Collecting interest is not
permitted under Islamic law.

8. The first Indian RRB that has achieved Core Banking


Solution(CBS) is ________
A. Rushikulya Gramya Bank (RGB)
B. Pallavan Bank
C. Bangiya Gramin Vikash Bank
D. Allahabad UP Gramin Bank
E. None of the Above

Answer & Explanation

A. Rushikulya Gramya Bank (RGB)


Explanation:
The Rushikulya Gramya Bank (RGB), a regional rural bank (RRB)
operating in south Orissa, has become the first RRB in the state to

www.bankingpdf.com
have placed all its branches on the Core Banking Solutions (CBS)
platform.

9. Government of India closed ______ and established National


Company Law Tribunal and National Company Law Appellate
Tribunal.
A. IIFCL
B. BIFR
C. SICA
D. Both (A) and (C)
E. None of the Above

Answer & Explanation

B. BIFR
Explanation:
The Sick Industrial Companies (Special Provisions) Repeal Act, 2003
replaced SICA and sought to dissolve the BIFR and the Appellate
Authority for Industrial and Financial Reconstruction (AAIFR), replacing
them by the NCLT and NCLAT. The Board for Industrial and Financial
Reconstruction(BIFR), an agency of GOI was established under The
Sick Industrial Companies (Special Provisions) Act, 1985 (SICA).

10. RBI can inspect the branches of Banks under _______ of the
Banking Regulation Act, 1949
A. Section 35
B. Section 25
C. Section 15
D. Section 30
E. None of the Above

Answer & Explanation

A. Section 35
Explanation:

www.bankingpdf.com
RBI can inspect the branches of Banks under section 35 of the Banking
Regulation Act, 1949

150-160 Questions :

1. Which allows banks and other financial institution to auction


residential or commercial properties to recover loans?
A. Industrial Disputes Act 1947
B. Foreign Exchange Management Act, 1999
C. Banking Regulation Act, 1949
D. SARFAESI Act, 2002
E. None of the Above

Answer & Explanation

D. SARFAESI Act, 2002


Explanation:
The Securitisation and Reconstruction of Financial Assets and
Enforcement of Security Interest Act, 2002 (also known as the
SARFAESI Act) is an Indian law. It allows banks and other financial
institution to auction residential or commercial properties to recover
loans.

2. _______ is an integrated bill payment system which will offer


inter operable bill payment service to customers online as well
as through a network of agents on the ground.
A. BBPS
B. IMPS
C. AEPS
D. APBS
E. None of the Above

Answer & Explanation

www.bankingpdf.com
A. BBPS
Explanation:
The Bharat Bill Payment System (BBPS) is a RBI mandated system
which will offer integrated and interoperable bill payment services to
customers across geographies with certainty, reliability and safety of
transactions.

3. Which of the following will function as the Bharat Bill Payment


Central Unit (BBPCU), single authorized entity operating the
BBPS?
A. RBI
B. NPCI
C. GOI
D. All of the Above
E. None of the Above

Answer & Explanation

B. NPCI
Explanation:
National Payments Corporation of India (NPCI) will function as the
authorized Bharat Bill Payment Central Unit (BBPCU), which will be
responsible for setting business standards, rules and procedures for
technical and business requirements for all the participants. NPCI, as
the BBPCU, will also undertake clearing and settlement activities
related to transactions routed through BBPS.

4. Which of the following is not a type of Billers?


A. Online Mode
B. Offline Mode (A)
C. Offline Mode (B)
D. All of the Above
E. None of the Above

Answer & Explanation

www.bankingpdf.com
D. All of the Above
Explanation:
Type of Billers:
Online Mode
Offline Mode (A)
Offline Mode (B)

5. Which of the following will be the authorised operational units?


A. BBPCU
B. BBPOU
C. Biller
D. All of the Above
E. None of the Above

Answer & Explanation

B. BBPOU
Explanation:
Bharat Bill Payment Operating Units (BBPOUs) will be the authorised
operational units.

6. To become BBPOUs, Banks and non-bank entities are


mandatorily required to apply for approval / authorisation to
Reserve Bank of India under ________
A. Payment and Settlement Systems (PSS) Act 2007.
B. Foreign Exchange Management Act, 1999
C. Banking Regulation Act, 1949
D. SARFAESI Act, 2002
E. None of the Above

Answer & Explanation

A. Payment and Settlement Systems (PSS) Act 2007.


Explanation:
To become BBPOUs, Banks and non-bank entities are mandatorily

www.bankingpdf.com
required to apply for approval / authorisation to Reserve Bank of India
under Payment and Settlement Systems (PSS) Act 2007. Bharat Bill
Payment Operating Units (BBPOUs) will be the authorised operational
units.

7. In which of the following mode, BBPOU will confirm the bill


payment basis the bill information dump provided by the
respective biller?
A. Online Mode
B. Offline Mode (A)
C. Offline Mode (B)
D. All of the Above
E. None of the Above

Answer & Explanation

B. Offline Mode (A).


Explanation:
In Offline Mode (A) BBPOU will confirm the bill payment basis the bill
information dump provided by the respective biller.

8. In which of the following mode, the BBPOU receives all the bill
payment requests for that particular biller without any
validation?
A. Online Mode
B. Offline Mode (A)
C. Offline Mode (B)
D. All of the Above
E. None of the Above

Answer & Explanation

C. Offline Mode (B)


Explanation:

www.bankingpdf.com
In Offline Mode (B) receives all the bill payment requests for that
particular biller without any validation.

9. As per the Reserve Bank of India, BBPS will initially accept


utility bill payments such as _________
A. Telephone Bill
B. Electricity Bill
C. DTH services
D. All of the Above
E. None of the Above

Answer & Explanation

D. All of the Above


Explanation:
As per the Reserve Bank of India, BBPS will initially accept utility bill
payments such as electricity, water, gas, telephone and direct-to-home
services and will later include other repetitive payments including
school and university fees and municipal taxes.

10. ________ have been awarded the license by the Central


Bank to operate Bharat Bill Payment System (BBPS) recently.
A. Common Service Centres (CSC)
B. Financial Institutions
C. Only (A)
D. Both (A) and (B)
E. None of the Above

Answer & Explanation

A. Common Service Centres (CSC)


Explanation:
over 1.5 Lakh common service centres (CSC) or Digital Seva Kendres
have been awarded the license by the Central Bank to operate Bharat
Bill Payment System (BBPS).

www.bankingpdf.com
160-170 Questions :

1. A trad-able form of loan is normally termed as ________


A. Certificate of Deposit (CD)
B. Inter Bank term Money
C. Bill Re-discounting
D. Debt Instrument
E. None of the Above

Answer & Explanation

D. Debt Instrument
Explanation:
A trad-able form of loan is normally termed as a Debt Instrument.
They are usually obligations of issuer of such instrument as regards
certain future cash flow representing Interest & Principal, which the
issuer would pay to the legal owner of the Instrument.

2. Which of the following is not a type of Debt Instrument?


A. Issuer class
B. Security
C. Coupon bearing
D. All of the Above
E. None of the Above

Answer & Explanation

E. None of the Above


Explanation:
Debt Instruments are of various types.The distinguishing factors of the
Debt Instruments are as follows:
Issuer class
Coupon bearing / Discounted
Interest Terms

www.bankingpdf.com
Repayment Terms
Security / Collateral / Guarantee

3. Debentures are divided into different categories on the basis of


which of the following?
A. Convertibility
B. Security
C. Term
D. Both (A) and (B)
E. None of the Above

Answer & Explanation

D. Both (A) and (B)


Explanation:
Debentures are divided into different categories on the basis of:
convertibility of the instrument and Security.
Debentures can be classified on the basis of convertibility into:
Non Convertible Debentures (NCD)
Partly Convertible Debentures (PCD)
Fully convertible Debentures (FCD)
Optionally Convertible Debentures (OCD)
Debentures can be classified on the basis of security into:
Secured Debentures
Unsecured Debentures

4. Which of the following instruments retain the debt character


and can not be converted in to equity shares?
A. Non Convertible Debentures (NCD)
B. Partly Convertible Debentures (PCD)
C. Fully convertible Debentures (FCD)
D. Optionally Convertible Debentures (OCD)
E. None of the Above

Answer & Explanation

www.bankingpdf.com
A. Non Convertible Debentures (NCD)
Explanation:
Non Convertible Debentures instruments retain the debt character and
can not be converted in to equity shares.

5. In which of the following instruments the investor has the


option to either convert these debentures into shares at price
decided by the issuer/agreed upon at the time of issue.
A. Non Convertible Debentures (NCD)
B. Partly Convertible Debentures (PCD)
C. Fully convertible Debentures (FCD)
D. Optionally Convertible Debentures (OCD)
E. None of the Above

Answer & Explanation

D. Optionally Convertible Debentures (OCD)


Explanation:
The investor has the option to either convert these debentures into
shares at price decided by the issuer/agreed upon at the time of issue.

6. In which of the following conversion the investors enjoy the


same status as ordinary shareholders of the company?
A. Secured Debentures
B. Partly Convertible Debentures (PCD)
C. Fully convertible Debentures (FCD)
D. Optionally Convertible Debentures (OCD)
E. Unsecured Debentures

Answer & Explanation

C. Fully convertible Debentures (FCD)


Explanation:
These are fully convertible into Equity shares at the issuer’s notice. The
ratio of conversion is decided by the issuer. Upon conversion the

www.bankingpdf.com
investors enjoy the same status as ordinary shareholders of the
company.

7. Under ______ debentures, if the issuer fails on payment of


either the principal or interest amount, his assets can be sold to
repay the liability to the investors
A. Secured Debentures
B. Partly Convertible Debentures (PCD)
C. Fully convertible Debentures (FCD)
D. Optionally Convertible Debentures (OCD)
E. Unsecured Debentures

Answer & Explanation

A. Secured Debentures
Explanation:
These instruments are secured by a charge on the fixed assets of the
issuer company. So if the issuer fails on payment of either the principal
or interest amount, his assets can be sold to repay the liability to the
investors.

8. A part of debt instruments that are converted into Equity shares


in the future at notice of the issuer is called _________
A. Secured Debentures
B. Partly Convertible Debentures (PCD)
C. Fully convertible Debentures (FCD)
D. Optionally Convertible Debentures (OCD)
E. Unsecured Debentures

Answer & Explanation

B. Partly Convertible Debentures (PCD)


Explanation:
A part of these instruments are converted into Equity shares in the

www.bankingpdf.com
future at notice of the issuer. The issuer decides the ratio for
conversion. This is normally decided at the time of subscription.

9. Who among the following are institutional investors in the


Indian Debt Market?
A. Banks
B. Mutual funds
C. Foreign investors (FIIs)
D. Trusts
E. All of the Above

Answer & Explanation

E. All of the Above


Explanation:
Institutional investors operating in the Indian Debt Market are :
Banks
Insurance companies
Provident funds
Mutual funds
Trusts
Corporate treasuries

10. Who among the following regulates the Indian Corporate


Debt Market?
A. RBI
B. NABARD
C. SIDBI
D. SEBI
E. None of the Above

Answer & Explanation

D. SEBI
Explanation:

www.bankingpdf.com
Regulator for the Indian Corporate Debt Market is the Securities and
Exchange Board of India (SEBI). SEBI controls bond market and
corporate debt market in cases where entities raise money from public
through public issues.

170-180 Questions :

1. Who among the following appoints the “Authorized Dealers” for


foreign exchange transactions?
A. GOI
B. RBI
C. PSBs
D. FEDAI
E. FIMMDA

Answer & Explanation

D. FEDAI
Explanation:
Foreign Exchange Dealer’s Association of India (FEDAI) was set up in
1958 as an Association of banks dealing in foreign exchange in India
(typically called Authorised Dealers – ADs) as a self regulatory body
and is incorporated under Section 25 of The Companies Act, 1956. It’s
major activities include framing of rules governing the conduct of inter-
bank foreign exchange business among banks vis-à-vis public and
liaison with RBI for reforms and development of forex market.

2. Which of the following is an Association of Commercial Banks,


Financial Institutions and Primary Dealers?
A. GOI
B. RBI
C. PSBs

www.bankingpdf.com
D. FEDAI
E. FIMMDA

Answer & Explanation

E. FIMMDA
Explanation:
FIMMDA stands for The Fixed Income Money Market and Derivatives
Association of India (FIMMDA). It is an Association of Commercial
Banks, Financial Institutions and Primary Dealers. FIMMDA is a
voluntary market body for the bond, Money And Derivatives Markets.

3. Banking services provided to Special Economic Zones(SEZ)


units have been exempted from _________
A. Sales Tax
B. Securities Transaction Tax – STT.
C. Central Excise Duty
D. Service Tax
E. None of the Above

Answer & Explanation

D. Service Tax
Explanation:
Banking services offered to units set up in special economic zones have
been exempted from paying service tax.

4. The relationship between maturity and interest rates is known


as ________
A. Yield Curve
B. Zero Coupon Yield Curve
C. Negative Yield Curve
D. Neutral Yield Curve
E. None of the Above

www.bankingpdf.com
Answer & Explanation

B. Zero Coupon Yield Curve


Explanation:
The Zero Coupon Yield Curve aka the Spot Curve is a relationship
between maturity and interest rates which differs from a normal yield
curve

5. The relationship between time and yield on a homogenous risk


class of securities is called ________
A. Yield Curve
B. Zero Coupon Yield Curve
C. Spot Curve
D. Both (A) and (B)
E. None of the Above

Answer & Explanation

A. Yield Curve
Explanation:
The relationship between time and yield on a homogeneous risk class
of securities is called the Yield Curve.

6. Long-term debt securities issued by the GOI or any of the State


Government’s or undertakings owned by them or by
development financial institutions are called as ________
A. Shares
B. Debentures
C. Bonds
D. Both (A) and (C)
E. None of the Above

Answer & Explanation

www.bankingpdf.com
C. Bonds
Explanation:
Long-term debt securities issued by the Government of India or any of
the State Government’s or undertakings owned by them or by
development financial institutions are called as bonds.

7. A long-term security yielding a fixed rate of interest, issued by


a company is called ________
A. Shares
B. Debentures
C. Bonds
D. Both (A) and (C)
E. None of the Above

Answer & Explanation

B. Debentures
Explanation:
A debenture is a type of debt instrument that is not secured by
physical assets or collateral. Instruments issued by companies are
called debentures. The difference between the two is actually a
function of where they are registered and pay stamp duty and how
they trade.

8. Issuance of stamp duty on bonds is under ________


A. The Companies Act, 1956
B. The Public Debt Act 1944.
C. Indian Stamp Act 1899 (Central Act)
D. Reserve Bank of India Act, 1934
E. Both (A) and (C)

Answer & Explanation

C. Indian Stamp Act 1899 (Central Act)


Explanation:

www.bankingpdf.com
Issuance of stamp duty on bonds is under Indian Stamp Act 1899
(Central Act). A bond is transferable by endorsement and delivery
without payment of any transfer stamp duty.

9. Which of the following is subject to payment of Stamp Duty


under Indian Stamp Act, 1899 (Central Act)?
A. Commercial Paper
B. Treasury Bills
C. Certificate of Deposit
D. Both (A) and (C)
E. None of the Above

Answer & Explanation

C. Certificate of Deposit
Explanation:
CD is subject to payment of Stamp Duty under Indian Stamp Act, 1899
(Central Act).

10. Which of the following is/are not eligible to issue


Certificates of deposit(CD)?
A. RRBs
B. Co-operative banks
C. Scheduled banks
D. Both (A) and (B)
E. None of the Above

Answer & Explanation

D. Both (A) and (B)


Explanation:
All scheduled banks (except RRBs and Co-operative banks) are eligible
to issue CDs

www.bankingpdf.com
180-190 Questions :

1. An Indian rupee denominated bond issued outside India is


called ________
A. Yankee bond
B. Bulldog bond
C. Uridashi bond
D. Samurai bond
E. Masala bond

Answer & Explanation

E. Masala bond
Explanation:
Masala bonds an Indian rupee denominated bond issued outside India.

2. A Japanese yen-denominated bond issued by a non-Japanese


entity in the Japanese market is known as _______
A. Yankee bond
B. Bulldog bond
C. Uridashi bond
D. Samurai bond
E. Maple bond

Answer & Explanation

D. Samurai bond
Explanation:
Samurai bond, a Japanese yen-denominated bond issued by a non-
Japanese entity in the Japanese market.

3. “Maple bond” issued by which of the following countries?


A. USA
B. Japan
C. China

www.bankingpdf.com
D. Canada
E. Korea

Answer & Explanation

D. Canada
Explanation:
Maple bond, a Canadian dollar-denominated bond issued by a non-
Canadian entity in the Canadian market.

4. A foreign currency denominated bond issued by foreign


company in Singapore is known as ______
A. Yankee bond
B. Bulldog bond
C. Uridashi bond
D. Husao bond
E. Lion City bond

Answer & Explanation

E. Lion City bond


Explanation:
Lion City bond foreign currency denominated bond issued by foreign
company in Singapore.

5. A non-yen-denominated bond sold to Japanese retail investors


is called _________
A. Yankee bond
B. Bulldog bond
C. Uridashi bond
D. Husao bond
E. Lion City bond

Answer & Explanation

www.bankingpdf.com
C. Uridashi bond
Explanation:
Uridashi bond, a non-yen-denominated bond sold to Japanese retail
investors.

6. _______ is an Australian dollar-denominated bond issued by a


non-Australian entity in the Australian market.
A. Yankee bond
B. Bulldog bond
C. Kangaroo bond
D. Husao bond
E. Panda bond

Answer & Explanation

C. Kangaroo bond
Explanation:
Kangaroo bond is an Australian dollar-denominated bond issued by a
non-Australian entity in the Australian market.

7. A Chinese renminbi-denominated bond issued by a Chinese


entity in Hong Kong is called ________
A. Yankee bond
B. Dim sum bond
C. Kangaroo bond
D. Husao bond
E. Panda bond

Answer & Explanation

B. Dim sum bond


Explanation:
A Chinese renminbi-denominated bond issued by a Chinese entity in
Hong Kong is called Dim sum bond.

www.bankingpdf.com
8. _______ is a Chinese renminbi-denominated bond issued by a
non-China entity in the People’s Republic of China market
A. Yankee bond
B. Dim sum bond
C. Kangaroo bond
D. Husao bond
E. Panda bond

Answer & Explanation

E. Panda bond
Explanation:
A Panda bond is a Chinese renminbi-denominated bond from a non-
Chinese issuer, sold in the People’s Republic of China.

9. In Which of the following countries “Shogun bond” is issued?


A. USA
B. Japan
C. China
D. Canada
E. Korea

Answer & Explanation

B. Japan
Explanation:
A type of foreign-currency denominated bond that is issued in Japan by
foreign entities.

10. “Matryoshka bond” is issued by?


A. USA
B. Japan
C. Russia
D. Canada
E. Korea

www.bankingpdf.com
Answer & Explanation

C. Russia
Explanation:
A Russian rouble-denominated bond issued in the Russian Federation
by non-Russian entities. The name derives from the famous Russian
wooden dolls, Matrioshka, popular among foreign visitors to Russia.

190-200 Questions :

1. What is the full form of RDA?


A. Rupee Draft Arrangement
B. Rupee Demand Arrangement
C. Rupee Draw Arrangement
D. Rupee Drawing Arrangement
E. None of the Above

Answer & Explanation

D. Rupee Drawing Arrangement


Explanation:
Rupee Drawing Arrangement (RDA) is a channel to receive cross-
border remittances from overseas jurisdictions. Under this
arrangement, the Authorised Category I banks enter into tie-ups with
the non-resident Exchange Houses in the FATF compliant countries to
open and maintain their Vostro Account.
Vostro Account means Local currency account maintained by a local
bank for a foreign (correspondent) bank. For the foreign bank it is a
Nostro account.

2. FATF stands for ________


A. Financial Action Task Force
B. Financial Asset Task Force

www.bankingpdf.com
C. Financial Asset Trade Force
D. Financial Action Trade Force
E. None of the Above

Answer & Explanation

A. Financial Action Task Force


Explanation:
The Financial Action Task Force (on Money Laundering) (FATF), also
known by its French name, Groupe d’action financière (GAFI), is an
intergovernmental organization founded in 1989 on the initiative of the
G7 to develop policies to combat money laundering.

3. FATF, an Intergovernmental Organization is headquartered in


__________
A. Newyork, USA
B. Geneva, Switzerland
C. Italy, Rome
D. Paris, France
E. None of the Above

Answer & Explanation

D. Paris, France
Explanation:
The Financial Action Task Force (on Money Laundering) (FATF), also
known by its French name, Groupe d’action financière (GAFI), is an
intergovernmental organization founded in 1989
The FATF Secretariat is housed at the headquarters of the OECD in
Paris.

4. How many members are there in FATF?


A. 42
B. 43
C. 40

www.bankingpdf.com
D. 37
E. 39

Answer & Explanation

D. 37
Explanation:
There are currently 37 members of the FATF; 35 jurisdictions and 2
regional organisations (the Gulf Cooperation Council and the European
Commission). These 37 Members are at the core of global efforts to
combat money laundering and terrorist financing.

5. Which of the following is an observer of FATF?


A. Ireland
B. Israel
C. Saudi Arabia
D. Both (B) and (C)
E. All of the Above

Answer & Explanation

D. Both (B) and (C)


Explanation:
FATF Observer Countries:
Israel
Saudi Arabia

6. _________ is a way of transferring personal remittances from


abroad to beneficiaries in India
A. RDA
B. MTSS
C. Both (A) and (B)
D. All of the Above
E. None of the Above

www.bankingpdf.com
Answer & Explanation

B. MTSS
Explanation:
Money Transfer Service Scheme (MTSS) is a way of transferring
personal remittances from abroad to beneficiaries in India.
Only inward personal remittances into India such as remittances
towards family maintenance and remittances favouring foreign tourists
visiting India are permissible. Under the scheme there is a tie-up
between reputed money transfer companies abroad known as Overseas
Principals and agents in India known as Indian Agents who would
disburse funds to beneficiaries in India at ongoing exchange rates.

7. The Overseas Principal should obtain necessary authorisation


from the Reserve Bank of India under the provisions of which
Act to commence/ operate a payment system?
A. Reserve Bank of India Act–1934
B. Banking Companies (Acquisition and Transfer of Undertakings) Act–
1970
C. Banking Regulation (Companies) Rules, 1949
D. Payment and Settlement Systems Act (PSS Act), 2007
E. None of the Above

Answer & Explanation

D. Payment and Settlement Systems Act (PSS Act), 2007


Explanation:
The Overseas Principal should obtain necessary authorisation from the
Department of Payment and Settlement Systems, Reserve Bank of
India under the provisions of the Payment and Settlement Systems Act
(PSS Act), 2007 to commence/ operate a payment system.

8. What is the maximum limit on the amount of money that can be


sent under MTSS?
A. USD 2,500

www.bankingpdf.com
B. USD 1,500
C. USD 1,000
D. USD 2,200
E. None of the Above

Answer & Explanation

A. USD 2,500
Explanation:
A cap of USD 2,500 has been placed on individual remittances under
the Money Transfer Service Scheme (MTSS)

9. How many remittances can be received by a single individual


beneficiary under the scheme during a year?
A. 40
B. 45
C. 40
D. 35
E. 30

Answer & Explanation

E. 30
Explanation:
Thirty remittances can be received by a single individual beneficiary
under the scheme during a calendar year.

10. What is the maximum limit on cash payment that is to be


made to the beneficiary under MTSS in India?
A. Rs. 25,000
B. Rs. 15,000
C. Rs. 10,000
D. Rs. 40,000
E. Rs. 50,000

www.bankingpdf.com
Answer & Explanation

E. Rs. 50,000
Explanation:
Amounts up to INR 50,000/- may be paid in cash to a beneficiary in
India. These can also be loaded on to a pre-paid card issued by banks.
Any amount exceeding this limit shall be paid by means of account
payee cheque/ demand draft/ payment order, etc., or credited directly
to the beneficiary’s bank account. However, in exceptional
circumstances, where the beneficiary is a foreign tourist, higher
amounts may be disbursed in cash.

200-210 Questions :

1. _________ is an account maintained in foreign currency with


an Authorised Dealer Bank.
A. Exchange Earners’Foreign Currency Account
B. Exchange Eligible Foreign Currency Account
C. Earners’ Eligible Foreign Currency Account
D. Earners’ Exchange Foreign Currency Account
E. Equity Exchange Foreign Currency Account

Answer & Explanation

A. Exchange Earners’Foreign Currency Account


Explanation:
Exchange Earners’ Foreign Currency Account (EEFC) is an account
maintained in foreign currency with an Authorised Dealer Bank
(Category – I Bank) in foreign exchange.

2. An EEFC account can be held only in the form of ________


A. Savings Bank Account
B. Deposit Account

www.bankingpdf.com
C. Current Account
D. Both (B) and (C)
E. None of the Above

Answer & Explanation

C. Current Account
Explanation:
An EEFC account can be held only in the form of a current account. No
interest is payable on EEFC accounts.

3. Who is/are eligible to open an EEFC Account?


A. Individuals
B. Companies
C. Residents of India
D. Both (B) and (C)
E. All of the Above

Answer & Explanation

E. All of the Above


Explanation:
All categories of foreign exchange earners, such as individuals,
companies, etc., who are resident in India, may open EEFC accounts.

4. Which of the following percentage of foreign exchange earnings


can be credited to the EEFC account?
A. 80%
B. 90%
C. 75%
D. 95%
E. 100%

Answer & Explanation

www.bankingpdf.com
E. 100%
Explanation:
under EEFC account, a facility provided to the foreign exchange
earners, including exporters, to credit 100 per cent of their foreign
exchange earnings to the account. so that the account holders do not
have to convert foreign exchange into Rupees and vice versa, thereby
minimizing the transaction costs.

5. What is the minimum maturity period for Rupee Denominated


Bonds?
A. 2 years
B. 3 years
C. 4 years
D. 5 years
E. None of the Above

Answer & Explanation

B. 3 years
Explanation:
The Rupee denominated bonds can only be issued in a country and can
only be subscribed by a resident of a country with minimum maturity
period of 3 years.

6. Which of the following can issue Rupee denominated bonds?


A. Real Estate Investment Trusts (REITs)
B. Infrastructure Investment Trusts (InvITs)
C. Limited Liability Partnerships
D. Both (A) and (B)
E. None of the Above

Answer & Explanation

D. Both (A) and (B)


Explanation:

www.bankingpdf.com
Any corporate (entity registered as a company under the Companies
Act, 1956/ 2013) or body corporate (entity specially created out of a
specific act of the Parliament) is eligible to issue Rupee denominated
bonds overseas. Real Estate Investment Trusts (REITs) and
Infrastructure Investment Trusts (InvITs) coming under the regulatory
jurisdiction of the Securities and Exchange Board of India (SEBI) are
also eligible. Indian banks are, however, not permitted to issue such
bonds. Other resident entities like Limited Liability Partnerships and
Partnership firms, etc. are also not eligible to issue these bonds.

7. What is the maximum amount that can be raised through


issuance of Rupee denominated bonds under automatic route?
A. INR 50 billion
B. INR 60 billion
C. INR 55 billion
D. INR 65 billion
E. No limit

Answer & Explanation

A. INR 50 billion
Explanation:
The maximum amount that any eligible borrower can raise through
issuance of these bonds under automatic route is INR 50 billion or its
equivalent during a financial year.

8. Under Liberalised Remittance Scheme, PAN card need not be


insisted upon for remittance made towards permissible current
account transactions up to __________
A. USD 25,000
B. USD 20,000
C. USD 15,000
D. USD 10,000
E. None of the Above

www.bankingpdf.com
Answer & Explanation

A. USD 25,000
Explanation:
PAN card need not be insisted upon for remittance made towards
permissible current account transactions up to USD 25,000 per
financial year.

9. Which of the following account can be repatriable?


A. NRE Account
B. NRO Account
C. FCNR(B) Account
D. Both (A) and(C)
E. All of the Above

Answer & Explanation

D. Both (A) and(C)


Explanation:
Non-Resident (External) Rupee Account Scheme[NRE Account] and
Foreign Currency (Non-Resident) Account (Banks) Scheme [FCNR (B)
Account]can be repatriable
while Non-Resident Ordinary Rupee Account Scheme[NRO Account]is
not repatriable except for all current income.Balances in an NRO
account of NRIs/ PIOs are remittable up to USD 1 (one) million per
financial year (April-March) along with their other eligible assets.

10. Which of the following does not have savings account?


A. NRE Account
B. NRO Account
C. FCNR(B) Account
D. Both (A) and(C)
E. All of the Above

Answer & Explanation

www.bankingpdf.com
C. FCNR(B) Account
Explanation:
Non-Resident (External) Rupee Account Scheme[NRE Account]:
Savings, Current, Recurring, Fixed Deposit
Foreign Currency (Non-Resident) Account (Banks) Scheme [FCNR (B)
Account]: Term Deposit only
Non-Resident Ordinary Rupee Account Scheme [NRO Account]:
Savings, Current, Recurring, Fixed Deposit.

210-220 Questions :

1. The IMGC is the sole provider of mortgage guarantor for


housing finance in India. The alphabet “I” in the abbreviation
IMGC stands for________
A. India
B. Institute
C. Internal
D. Informative
E. International

Answer & Explanation

A. India
Explanation:
Mortgage Guarantee (also known as mortgage insurance in some parts
of the world) is a financial product which compensates lending
institutions or housing finance companies for losses that may arise
when a home owner defaults on a mortgage loan.
IMGC is the first company of its kind to introduce this kind of product
in the Housing sector in India. The intention is to mitigate risk taken by
lenders, banks and housing finance companies, over time making it
easier for people to get access to home loans of higher values (or with

www.bankingpdf.com
lower down-payment amounts) earlier in life than would have been
possible without IMGC’s involvement.

2. The national agency mandated to control, analyse and


disseminate Suspicious Transaction Reports(STR), Cash
Transaction Reports and counterfeit currency reports to various
investigative agencies of the country after receiving it from
banks and other financial intermediaries is ________
A. RBI
B. Financial Intelligence Unit(FIU)
C. Intelligence Bureau(IB)
D. Central Bureau of Investigation(CBI)
E. Central Intelligence Unit(CIU)

Answer & Explanation

B. Financial Intelligence Unit(FIU)


Explanation:
The FIU is the national agency mandated to collect, analyse and
disseminate Suspicious Transaction Reports (STRs), Cash Transaction
Reports and Counterfeit Currency Reports to various investigative
agencies of the country after receiving it from banks and other financial
intermediaries under a stipulated mechanism.

3. According to RBI guidelines the term ‘bulk deposit’ is used for


single rupee term deposit of ________
A. Rs. 1 crore and above
B. Rs, 25 lakh and above
C. Rs, 75 lakh and above
D. Rs, 50 lakh and above
E. Rs, 15 lakh and above

Answer & Explanation

www.bankingpdf.com
A. Rs. 1 crore and above
Explanation:
Bulk Deposit means single Rupee term deposits of 1 Crore and above.

4. A collection of non-bank financial intermediaries that provide


services similar to traditional commercial banks is known as
_________
A. Off-shore Banking
B. Investment Banking System
C. Shadow Banking System
D. Conventional Banking
E. Merchant Banking System

Answer & Explanation

C. Shadow Banking System


Explanation:
A shadow banking system refers to the financial intermediaries
involved in facilitating the creation of credit across the global financial
system but whose members are not subject to regulatory oversight.
The shadow banking system also refers to unregulated activities by
regulated institutions.

5. According to Reserve Bank of India Regulations, “Universal


Banks” are permitted to pick up stake in Payment Banks up to
_________
A. 35%
B. 25%
C. 20%
D. 40%
E. 30%

Answer & Explanation

www.bankingpdf.com
E. 30%
Explanation:
RBI regulations allow universal banks to invest up 30 per cent in
payments banks.

6. The “Stand Up India” is aimed at promoting entrepreneurship


among women and scheduled castes and tribes. The bank loans
under the scheme shall be referenced to _______
A. SIDBI
B. NABARD
C. RBI
D. MUDRA
E. DRDA

Answer & Explanation

A. SIDBI
Explanation:
The “Stand up India Scheme” is being launched now to promote
entrepreneurship among Scheduled Caste/Schedule Tribe and Women
for loans in the range of Rs. 10 Lakhs to Rs. 100 Lakhs. The bank loans
under the scheme shall be referenced to SIDBI.

7. An NBFC-MFI is defined as a non-deposit taking NBFC (other


than a company licensed under Section 25 of the Indian
Companies Act, 1956) with Minimum Net Owned Funds of
________
A. Rs.5 crore
B. Rs.4 crore
C. Rs.3 crore
D. Rs.2 crore
E. None of the Above

Answer & Explanation

www.bankingpdf.com
A. Rs.5 crore
Explanation:
Non Banking Financial Company – Micro Finance Institutions (NBFC-
MFIs) licensed under Section 25 of the Indian Companies Act, 1956)
with Minimum Net Owned Funds of Rs.5 crore.

8. The Minimum Net Owned Fund for NBFC-MFIs registered in the


North Eastern Region of the country is _________
A. Rs.5 crore
B. Rs.4 crore
C. Rs.3 crore
D. Rs.2 crore
E. None of the Above

Answer & Explanation

Rs.2 crore
Explanation:
Minimum Net Owned Fund for NBFC-MFIs registered in the North
Eastern Region of the country is Rs. 2 crore.

9. Which are defined as total assets other than cash and bank
balances and money market instruments?
A. Net Assets
B. Qualifying Assets
C. Commercial Papers
D. Both(A) and (C)
E. All of the Above

Answer & Explanation

A. Net Assets
Explanation:
“Net assets” are defined as total assets other than cash and bank
balances and money market instruments.

www.bankingpdf.com
10. In SNRR Account, SNRR stands for _________
A. Statutory Non Resident Rupee
B. Single Non Resident Rupee
C. Statutory Non Reflex Rupee
D. Single Non Resident Rupee
E. Special Non Resident Rupee

Answer & Explanation

E. Special Non Resident Rupee


Explanation:
Any person resident outside India, having a business interest in India,
can open a Special Non-Resident Rupee Account (SNRR account) with
an authorised dealer for the purpose of putting through bonafide
transactions in rupees.

220-230 Questions :

1. Which of the following is/are correct about the “Operational


Risk” as used in the field of banking?
A. Risk of loss due to inadequate or failed internal process
B. Risk of loss due to natural calamities
C. Loss occurred due to non-compliance of legal procedures
D. All of the Above
E. None of the Above

Answer & Explanation

A. Risk of loss due to inadequate or failed internal process


Explanation:
The Basel Committee defines the operational risk as the “risk of loss
resulting from inadequate or failed internal processes, people and
systems or from external events”.

www.bankingpdf.com
2. What is/are the characteristics of ‘Hot Money’?
A. The term is used for fresh currency notes issued by RBI
B. It is the fund which inflows in the market to take advantage of high
interest rates
C. It is the fund which is thrown in the market to create imbalance in
the stock markets
D. All of the Above
E. None of the Above

Answer & Explanation

B. It is the fund which inflows in the market to take advantage


of high interest rates
Explanation:
Hot money is currency that moves regularly, and quickly, between
financial markets so investors ensure they are getting the highest
short-term interest rates available.

3. When a corporate entity wishes to raise money from the market


it can do that by issuing
A. Treasury Bills
B. Kisan Vikas Patra
C. National Savings Certificate
D. Commercial Papers
E. None of the Above

Answer & Explanation

D. Commercial Papers
Explanation:
Commercial paper is an unsecured, short-term debt instrument issued
by a corporation, typically for the financing of accounts receivable,
inventories and meeting short-term liabilities.

www.bankingpdf.com
4. When an agent asks a customer to invest in a Mutual Fund
product without telling him/her about the risks involved in the
investment, the process is termed as ______
A. Mis-selling
B. Undertaking
C. Misappropriation of Funds
D. cross selling
E. None of the Above

Answer & Explanation

A. Mis-selling
Explanation:
The act of selling something that is not suitable for the person who
buys it.

5. In the Capital market, the term arbitrage is used with reference


to ______
A. Purchase of securities to cover the sale
B. Sale of securities to reduce the loss on purchase
C. Simultaneous purchase and sale of securities to make profits from
price
D. Variation in different markets
E. None of the Above

Answer & Explanation

C. Simultaneous purchase and sale of securities to make profits


from price
Explanation:
Arbitrage is the simultaneous purchase and sale of an asset to profit
from a difference in the price. It is a trade that profits by exploiting the
price differences of identical or similar financial instruments on
different markets or in different forms. Arbitrage exists as a result of
market inefficiencies.

www.bankingpdf.com
6. Sub-prime lending refers to ________
A. lending done by banks at rates below PLR
B. Funds raised by the banks at sub-libor rates
C. Group of banks which are not rated as prime banks as per Banker’s
Almanac
D. lending done by financing institutions including banks to customers
not meeting with normally required credit appraisal standards
E. All of the Above

Answer & Explanation

D. lending done by financing institutions including banks to


customers not meeting with normally required credit appraisal
standards
Explanation:
In finance, subprime lending (also referred to as near-prime, non-
prime, and second-chance lending) means making loans to people who
may have difficulty maintaining the repayment schedule, sometimes
reflecting setbacks, such as unemployment, divorce, medical
emergencies, etc.

7. Currency Swap is an instrument to manage _______


A. currency risk
B. interest rate risk
C. currency and interest rate risk
D. cash flows in different currency
E. All of the above

Answer & Explanation

D. cash flows in different currency


Explanation:
A currency swap (or a cross currency swap) is a foreign exchange
derivative between two institutions to exchange the principal and/or

www.bankingpdf.com
interest payments of a loan in one currency for equivalent amounts, in
net present value terms, in another currency.

8. Head quarters of National Council of Applied Economic


Research(NCAER) is located in ______
A. Mumbai
B. New Delhi
C. Hyderabad
D. Chennai
E. None of the Above

Answer & Explanation

B. New Delhi
Explanation:
National Council of Applied Economic Research or NCAER is a New
Delhi based non-profit think tank of economics.

9. Trade Gap means


A. Gap between total GDP and total consumption
B. Gap between total imports and total exports
C. Gap between available liquidity and expected demand in next five
months
D. Gap between budgeted revenue collection and actual collection of
the same
E. None of the above

Answer & Explanation

B. Gap between total imports and total exports


Explanation:
The amount by which the value of a country’s visible imports exceeds
that of visible exports; an unfavourable balance of trade.

www.bankingpdf.com
10. Which of the following agencies in India is responsible for
computation of national income?
A. NCAER
B. CSO
C. NSS
D. RBI
E. None of the Above

Answer & Explanation

B. CSO
Explanation:
The Central Statistical Office which is one of the two wings of the
National Statistical Organisation (NSO) is responsible for coordination
of statistical activities in the country and for evolving and maintaining
statistical standards. Its activities include compilation of National
Accounts.

230-240 Questions :

1. Which of the following is the most active segment of the money


market in India?
A. Commercial Paper(CP)
B. Certificate of Deposit(CD)
C. Call Money
D. All of the above
E. None of the Above

Answer & Explanation

C. Call Money
Explanation:

www.bankingpdf.com
Call money is short-term finance repayable on demand, with a maturity
period of one to fourteen days or overnight to fortnight.

2. SEPA Stands for ______


A. Scottish Environment Protection Agency
B. Single Euro Payments Area
C. Scottish Environment Protection Area
D. Single Euro Payments Agency
E. None of the Above

Answer & Explanation

B. Single Euro Payments Area


Explanation:
The Single Euro Payments Area (SEPA) is a payment-integration
initiative of the European Union for simplification of bank transfers
denominated in euro.

3. The paid-up capital of non-scheduled bank is less than _____


A. Rs. 10 lakh
B. Rs. 20 lakh
C. Rs. 15 lakh
D. Rs. 5 lakh
E. None of the Above

Answer & Explanation

D. Rs. 5 lakh
Explanation:
Banks not under in the second schedule of RBI Act, 1934 are called
non-scheduled banks.
Banks with a reserve capital of less than 5 lakh rupees qualify as non-
scheduled banks. Unlike scheduled banks, they are not entitled to
borrow from the RBI for normal banking purposes, except, in
emergency or “abnormal circumstances.

www.bankingpdf.com
Jammu & Kashmir Bank is an example of a non-scheduled commercial
bank.

4. The paid up capital and collected funds of scheduled bank must


not be less than _________
A. Rs. 10 lakh
B. Rs. 20 lakh
C. Rs. 15 lakh
D. Rs. 5 lakh
E. None of the Above

Answer & Explanation

D. Rs. 5 lakh
Explanation:
Banks under in the second schedule of RBI Act, 1934 are called
scheduled bank.To qualify as a scheduled bank, the paid up capital and
collected funds of the bank must not be less than Rs.5 lakh. Scheduled
banks are eligible for loans from the Reserve Bank of India at bank
rate, and are given membership to clearing houses.

5. Co-operative banks are regulated by the Reserve Bank of India


under _________
A. Banking Regulation Act, 1949
B. Banking Laws (Application to Co-operative Societies) Act, 1965
C. Both (A) and (B)
D. Negotiable Instrument Act–1881
E. None of the Above

Answer & Explanation

C. Both (A) and (B)


Explanation:
Co-operative banks are regulated by the Reserve Bank of India under

www.bankingpdf.com
the Banking Regulation Act, 1949 and Banking Laws (Application to Co-
operative Societies) Act, 1965

6. All banks registered under the ___________ are considered co-


operative banks.
A. Banking Regulation Act 1949
B. The Bankers’Books Evidence Act–1891
C. Cooperative Societies Act, 1912
D. Both (B) and (C)
E. None of the Above

Answer & Explanation

C. Cooperative Societies Act, 1912


Explanation:
Co-operative banks operate in both urban and non-urban areas. All
banks registered under the Cooperative Societies Act, 1912 are
considered co-operative banks. These are banks run by an elected
managing committee with provisions of members’ rights and a set of
“communally developed and approved bylaws and amendments.co-
operative banks work on a “no profit, no loss” basis.

7. _______ refers to the system in which two are more banks are
brought under common control by a device other than the
holding company?
A. Chain Banking
B. Group Banking
C. Mixed Banking
D. Branch Banking
E. None of the Above

Answer & Explanation

A. Chain Banking
Explanation:

www.bankingpdf.com
Chain Banking refers to the system in which two are more banks are
brought under common control by a device other than the holding
company.

8. A negotiable instrument delivered to a person conditionally or


for safe custody, but not for the purpose of negotiation is called
_________
A. Protest
B. Escrow
C. Noting
D. All of the Above
E. None of the Above

Answer & Explanation

B. Escrow
Explanation:
An escrow account is a temporary pass through account held by a third
party during the process of a transaction between two parties.

9. __________ is the process by which the ownership of the


credit instrument is transferred from one person to another.
A. Assignment
B. Endorsement
C. Negotiation
D. All of the Above
E. None of the Above

Answer & Explanation

C. Negotiation
Explanation:
In documentary credit, usually the beneficiary’s bank which agrees to
pay the beneficiary by purchasing a negotiable instrument (importer’s
or buyer’s draft). Also called accrediting party.

www.bankingpdf.com
10. A charge where there is neither the transfer of ownership
nor the possession is called _______
A. Pledge
B. Hypothecation
C. Mortgage
D. Lien
E. None of the Above

Answer & Explanation

B. Hypothecation
Explanation:
A charge against property for an amount of debt where neither
ownership nor possession is passed to the creditor.

240-250 Questions :

1. The rules framed in the Clayton’s case have been incorporated


in _________
A. Banking Regulation Act, 1949
B. Reserve Bank of India Act, 1934
C. Negotiable Instruments Act, 1881
D. Indian Contract Act, 1872
E. None of the Above

Answer & Explanation

D. Indian Contract Act, 1872


Explanation:
The rules framed in the Clayton’s case was incorporated in Section 59
of the Indian Contract Act, 1872.

www.bankingpdf.com
2. The Definition of ‘Banking’ is given in _______
A. Negotiable Instrument Act, 1881
B. RBI Act, 1934
C. The Banking Regulation Act, 1949
D. Indian Contract Act, 1872
E. None of the Above

Answer & Explanation

C. The Banking Regulation Act, 1949


Explanation:
As per Section 5(b) of the Banking Regulation Act, 1949 , “banking”
means the accepting, for the purpose of lending or investment, of
deposits of money from the public, repayable on demand or otherwise,
and withdraw-able by cheque, draft, order or otherwise.

3. Banks are required to maintain SLR under_____


A. Section 24 of the Banking Regulation Act
B. Section 35 of the Negotiable Instrument Act, 1881
C. Section 24 of RBI Act
D. Section 40 of Indian Contract Act, 1872
E. None of the Above

Answer & Explanation

A. Section 24 of the Banking Regulation Act


Explanation:
SLR is governed by the provisions of Section 24 of the Banking
Regulation Act. There is no minimum stipulation on SLR (earlier there
used to be a minimum stipulated SLR of 25% – but this was removed
with an amendment to the Banking Regulation Act in 2007).
However, SLR can not exceed 40%.

4. Which of the following was/were established under the


Recovery of Debts Due to Banks and Financial Institutions Act

www.bankingpdf.com
(RDDBFI Act), 1993?
A. DRTs
B. DRATs
C. ATs
D. Both (A) and (B)
E. None of the Above

Answer & Explanation

D. Both (A) and (B)


Explanation:
The Debts Recovery Tribunals (DRTs) and Debts Recovery Appellate
Tribunal (DRATs) were established under the Recovery of Debts Due to
Banks and Financial Institutions Act (RDDBFI Act), 1993 with the
specific objective of providing expeditious adjudication and recovery of
debts due to Banks and Financial Institution.

5. Which of the following is governed by the provisions of Section


42 of the Reserve Bank of India Act, 1934?
A. Bank rate
B. Reverse Repo Rate
C. SLR
D. CRR
E. None of the Above

Answer & Explanation

D. CRR
Explanation:
CRR is governed by the provisions of Section 42 of the Reserve Bank of
India Act, 1934.
There is no minimum & maximum level of CRR. We could go upto zero
CRR (negative values are of course absurd).

www.bankingpdf.com
6. A Shareholder has been defined by ________
A. Banking Regulation Act, 1949
B. Reserve Bank of India Act, 1934
C. Negotiable Instruments Act, 1881
D. Indian Contract Act, 1872
E. The companies act, 1956

Answer & Explanation

E. The companies act, 1956


Explanation:
The Companies Act, 1956
“Small Shareholder” means a shareholder holding shares of nominal
value of twenty thousand rupees or less in a public company to which
section 252 of the Act applies.

7. Spurious coins detected at the counter are sent to _____


A. RBI
B. GOI
C. IBRD
D. Mint
E. None of the Above

Answer & Explanation

D. Mint
Explanation:
when Spurious coins tendered over the counter, it should be cut and
handed over to the tenderer. If the tenderer disputes it, the coins
should be sent to the mint at his cost, for examination.

8. On a Cheque instead of two parallel lines only bank’s name is


written is known as _________
A. Payable to bearer
B. General Crossing

www.bankingpdf.com
C. Special Crossing
D. Both (B) and (C)
E. None of the Above

Answer & Explanation

C. Special Crossing
Explanation:
When a particular bank’s name is written in between the two parallel
lines the cheque is said to be specially crossed.The effect of special
crossing is that the bank makes payment only to the banker whose
name is written in the crossing. Specially crossed cheques are more
safe than a generally crossed cheques.

9. Minimum period of reinvestment deposits is _______


A. 2 months
B. 3 months
C. 6 months
D. 9 months
E. E. None of the Above

Answer & Explanation

C. 6 months
Explanation:
A Reinvestment Deposit Plan basically allows you to reinvest the
interest earned on your deposit. Period of deposit 6 months to 10
years.

10. What is the minimum limit in reinvestment deposits?


A. 100
B. 500
C. 1000
D. 5000

www.bankingpdf.com
Answer & Explanation

C. 1000
Explanation:
E. None of the Above
The minimum deposit amount is Rs. 1,000/-. Initially deposits should
be made only in multiples of Rs. 100/-. Renewals can be made in any
amount.

250-260 Questions :

1. NCAER is located in _______


A. Mumbai
B. New Delhi
C. Noida
D. Chennai
E. Hyderabad

Answer & Explanation

B. New Delhi
Explanation:
National Council of Applied Economic Research or NCAER is a New
Delhi based non-profit think tank of economics.

2. The Central Statistical Office (CSO) is located in _____


A. Mumbai
B. New Delhi
C. Noida
D. Chennai
E. Hyderabad

Answer & Explanation

www.bankingpdf.com
B. New Delhi
Explanation:
The Central Statistics Organisation is responsible for coordination of
statistical activities in the country, and evolving and maintaining
statistical standards. Its activities include National Income Accounting,
etc.

3. National Sample Survey Organization(NSSO) is an organization


under the ________
A. Ministry of Finance
B. Ministry of Statistics
C. Ministry of Commerce
D. All (B) and (C)
E. None of the Above

Answer & Explanation

B. Ministry of Statistics
Explanation:
The National Sample Survey Organisation (NSSO), now known as
National Sample Survey Office, is an organization under the Ministry of
Statistics of the Government of India. It is the largest organisation in
India conducting regular socio-economic surveys. It was established in
1950.

4. Headquarter of Agriculture of Finance Corporation(AFC) is


situated at_______
A. Mumbai
B. New Delhi
C. Noida
D. Gurugram
E. Hyderabad

Answer & Explanation

www.bankingpdf.com
A. Mumbai
Explanation:
Headquarter of AFC is situated at Mumbai. The Company has three
Regional Offices at Kolkata, New Delhi and Bangalore besides three
Branch Offices at Lucknow, Hyderabad and Pune and Field Offices at
Kalahandi, Bargarh (Orissa) and Godda (Jharkhand).

5. The headquarters of Investment Information & Credit Rating


Agency(ICRA) Limited is located in _________
A. Mumbai
B. New Delhi
C. Noida
D. Gurugram
E. Hyderabad

Answer & Explanation

D. Gurugram
Explanation:
ICRA is second largest Indian rating comapany in term of customer
base. It was a joint-venture between Moody’s and various Indian
commercial banks and financial services companies. Headquarters is in
Gurgaon (Haryana),India

6. The headquarter of CARE is ______


A. Mumbai
B. New Delhi
C. Noida
D. Gurugram
E. Hyderabad

Answer & Explanation

www.bankingpdf.com
A. Mumbai
Explanation:
Credit Analysis & Research Limited(CARE) is located in Mumbai.

7. SME Rating Agency of India Limited (SMERA), a third party


rating agency is situated in _______
A. Mumbai
B. New Delhi
C. Noida
D. Gurugram
E. Hyderabad

Answer & Explanation

A. Mumbai
Explanation:
SME Rating Agency of India Limited (SMERA) is a third party rating
agency exclusively set up for micro, small and medium enterprises
(MSME) in India for ratings on creditworthiness.

8. Credit Information Bureau India Limited (CIBIL) is India’s first


Credit Information Company (CIC) headquartered at ________
A. Mumbai
B. New Delhi
C. Noida
D. Gurugram
E. Hyderabad

Answer & Explanation

A. Mumbai
Explanation:
Credit Information Bureau (India) Limited (CIBIL) is India’s first Credit
Information Company (CIC) founded in August 2000. CIBIL collects

www.bankingpdf.com
and maintains records of an individual’s payments pertaining to loans
and credit cards.

9. STANDARD & POOR’S (S&P) is located in _______


A. New York
B. Washington D.C
C. Los Angeles
D. San Francisco
E. Chicago

Answer & Explanation

A. New York
Explanation:
Standard & Poor’s Financial Services LLC is an American financial
services company. It is a division of S&P Global that publishes financial
research and analysis on stocks, bonds and commodities.

10. The headquarter of ONICRA is ______


A. Mumbai
B. New Delhi
C. Noida
D. Gurugram
E. Hyderabad

Answer & Explanation

D. Gurugram
Explanation:
Onicra-leading Credit and Performance Rating agencies in India
providing ratings, risk assessment and analytical solutions.

www.bankingpdf.com
260-270 Questions :

1. What is the full form of CERSAI?


A. Combined Registry of Securitisation Asset Reconstruction and
Security Interest
B. Central Registry of Securitisation Agency Reconstruction and
Security Interest
C. Combined Registry of Securitisation Agency Reconstruction and
Security Interest
D. Central Registry of Securitisation Asset Reconstruction and Security
Interest
E. None of these

Answer & Explanation

D. Central Registry of Securitisation Asset Reconstruction and


Security Interest
Explanation:
Central Registry of Securitisation Asset Reconstruction and Security
Interest (CERSAI) is a central online security interest registry of India.
It was primarily created to check frauds in lending against equitable
mortgages, in which people would take multiple loans on the same
asset from different banks.

2. The headquarters of CERSAI located in ________


A. Mumbai
B. New Delhi
C. Hyderabad
D. Gurugram
E. None of these

Answer & Explanation

B. New Delhi
Explanation:

www.bankingpdf.com
The headquarters of Central Registry of Securitisation Asset
Reconstruction and Security Interest(CERSAI) located in New Delhi.

3. CERSAI was registered as a government-licensed company,


under section 25 of _________
A. Securitisation and Reconstruction of Financial Assets and
Enforcement of Security Interest Act, 2002.
B. Companies Act, 1956
C. Banking Regulation Act, 1949
D. Both (A) and (B)
E. None of these

Answer & Explanation

B. Companies Act, 1956


Explanation:
Central Registry of Securitisation Asset Reconstruction and Security
Interest of India is a company licensed under section 25 of the
Companies Act, 1956

4. CERSAI incorporated with majority share holding of ________


A. Government of India
B. National Housing Bank
C. Public Sector Banks
D. Both (B) and (C)
E. (A), (C) and (B)

Answer & Explanation

A. Government of India
Explanation:
51% of the equity is owned by the government, and the rest is owned
equally by National Housing Bank and 10 other public sector banks.

www.bankingpdf.com
5. Which of the following is/are share holders of CERSAI?
A. GOI
B. RBI
C. PSBs
D. NHB
E. (A), (C) and (D)

Answer & Explanation

E. (A), (C) and (D)


Explanation:
The Company is a Government Company with a shareholding of 51%
by the Central Government and select Public Sector Banks and the
National Housing Bank are also shareholders of the Company.

6. Financial institutions must register details of security interests


created by them with CERSAI within howmany days of its
creation?
A. 30
B. 60
C. 15
D. 90
E. None of these

Answer & Explanation

A. 30
Explanation:
Financial institutions must register details of security interests created
by them with CERSAI within 30 days of its creation.

7. Which of the following is/are provided by the platform of


CERSAI?
A. Registrations of transactions of securitisation
B. Asset Reconstruction

www.bankingpdf.com
C. Security Interest
D. All of the Above
E. None of these

Answer & Explanation

D. All of the Above


Explanation:
The Company is providing the platform for filing registrations of
transactions of securitisation, asset reconstruction and security interest
by the banks and financial institutions.

8. Which of the following defines the objectives of CERSAI?


A. Securitisation and Reconstruction of Financial Assets and
Enforcement of Security Interest Act, 2002.
B. Companies Act, 1956
C. Banking Regulation Act, 1949
D. Both (A) and (B)
E. None of these

Answer & Explanation

A. Securitisation and Reconstruction of Financial Assets and


Enforcement of Security Interest Act, 2002.
Explanation:
The object of the company is to maintain and operate a Registration
System for the purpose of registration of transactions of securitisation,
asset reconstruction of financial assets and creation of security interest
over property, as contemplated under Chapter IV of the Securitisation
and Reconstruction of Financial Assets and Enforcement of Security
Interest Act, 2002. (SARFAESI Act).

9. According to _________, Any person can also search and


inspect the records maintained by the Registry on payment of
fees.

www.bankingpdf.com
A. Securitisation and Reconstruction of Financial Assets and
Enforcement of Security Interest Act, 2002.
B. Companies Act, 1956
C. Both (A) and (B)
D. Securitisation and Reconstruction of Financial Assets and
Enforcement of Security Interest (Central Registry) Rules, 2011
E. None of these

Answer & Explanation

D. Securitisation and Reconstruction of Financial Assets and


Enforcement of Security Interest (Central Registry) Rules, 2011
Explanation:
Any person can also search and inspect the records maintained by the
Registry on payment of fees prescribed under the Securitisation and
Reconstruction of Financial Assets and Enforcement of Security Interest
(Central Registry) Rules, 2011.
For the general public, especially for home buyers, it enables them to
check the registry’s records to ensure that any property they are
planning to purchase, is free of any loan/security interest created by a
lender.

10. Which of the following is/are functions of CERSAI?


A. check frauds in lending against equitable mortgages
B. maintains a central registry of equitable mortgages
C. Provides loan
D. Both (A) and (B)
E. None of these

Answer & Explanation

D. Both (A) and (B)


Explanation:
CERSAI was primarily created to check frauds in lending against
equitable mortgages, in which people would take multiple loans on the

www.bankingpdf.com
same asset from different banks. CERSAI to start registration of
security interests created on movable and intangible assets such as
accounts receivables, book debt, and hypothecation as well as to start
registration of all other types of mortgages used in India.

270-280 Questions :

1. Which company is providing free 4G sims to certain smartphone


users like Samsung, LG, etc.?
A) Airtel
B) Vodafone
C) Reliance
D) Aircel
E) Tata

Answer & Explanation

C) Reliance
Explanation:
Reliance is providing its 4G sims free of cost to some companies
smartphone users on which they will have free calls and net for 3
months

2. Which bank has become the first private sector bank to sell
Indian gold coins?
A) Dhanlaxmi Bank
B) Federal Bank
C) ICICI Bank
D) IndusInd Bank
E) Karur Vysya Bank

www.bankingpdf.com
Answer & Explanation

B) Federal Bank
Explanation:
Kerala based private sector lender, Federal Bank has tied up with
MMTC for distribution of Indian Gold Coins (IGC), the first-Ever
national gold offering by the government, which was launched by
Prime Minister Narendra Modi In November 2015.

3. With which card company has Axis bank launched its


contactless debit cards?
A) MasterCard
B) RuPay
C) EuroPay
D) Visa
E) None of these

Answer & Explanation

D) Visa
Explanation:
Axis Bank, on Wednesday announced the launch of its Contactless
Secure+ Debit Card for regular savings account customers, to provide
access to a larger customer base with contactless technology. The
card, launched on the Visa payWave platform, will enable Axis Bank’s
customers to conveniently ‘Just Wave to Pay’ at NFC-enabled point of
sale (POS) terminals and also use the card as a normal debit card at
any POS terminal.

4. Which bank recently tied up with Chillr for cashless


transactions?
A) Federal Bank
B) DCB Bank
C) IDFC Bank

www.bankingpdf.com
D) Karnataka Bank
E) IndusInd Bank

Answer & Explanation

A) Federal Bank
Explanation:
Federal Bank announced its partnership with with Chillr, a mobile
banking application, to provide cashless transactions.
Chillr, India’s first and only multi-bank mobile application, allows users
to connect seamlessly with multiple bank accounts and manage all
their banking needs on a single platform, making money transfer
between bank accounts easier, faster and safer than sending a text
message.

5. After Mumbai, Doha bank is now going to open its branches in


which city?
A) Palakkad
B) Kochi
C) Idukki
D) Kasaragod
E) None of these

Answer & Explanation

B) Kochi
Explanation:
Qatar-based Doha Bank will open a state-of-the-art branch in Kochi,
the only branch of a Gulf Cooperation Council (GCC)-based bank in
India.

6. Who has been named as the new RBI Governor?


A) Subir Gokarn
B) Arundhati Bhattacharya
C) R. Gandhi

www.bankingpdf.com
D) Urjit R Patel
E) Arvind Subramanian

Answer & Explanation

D) Urjit R Patel
Explanation:
The choice of Urjit Patel as RBI governor was made by PM Narendra
Modi himself.

7. IndusInd Bank has tied up with which company to use its Cloud
commerce solutions to transform customer engagements and
enhance its cross-sell platforms?
A) Accenture
B) Infosys
C) Wipro
D) TCS
E) IBM

Answer & Explanation

E) IBM
Explanation:
As a part of a three year strategic agreement, IBM will provide a cloud
and predictive analytics based multi-channel campaign management
solution that enables IndusInd Bank to strengthen its online banking
presence while improving the product holding per customer.

8. For customers of how many banks, the UPI mobile application


is going to launch in the first phase?
A) 21
B) 19
C) 15
D) 25
E) None of these

www.bankingpdf.com
Answer & Explanation

A) 21
Explanation:
According to NPCI, the UPI application of 19 banks can be downloaded
from Google Play Store in next two to three working days.
The list of banks providing the app on Google Play Store are: Andhra
Bank, Axis Bank, Bank of Maharashtra, Bhartiya Mahila Bank, Canara
Bank, Catholic Syrian Bank, DCB Bank, Federal Bank, ICICI Bank, TJSB
Sahakari Bank, Oriental Bank of Commerce, Karnataka Bank, UCO
Bank, Union Bank of India, United Bank of India, Punjab National Bank,
South Indian Ban k
The customers of IDBI Bank and RBL Bank can download any UPI
enabled apps and link their account.

9. Which two banks have been given the tag “too big too fail” for
the year 2016?
A) ICICI and Axis Bank
B) Axis Bank and Punjab National Bank
C) Punjab National bank and SBI
D) SBI and ICICI
E) SBI and Axis Bank

Answer & Explanation

D) SBI and ICICI


Explanation:
The Reserve Bank of India retained the tag of “too big to fail” banks for
SBI and ICICI second year in a row. This means that the banking
regulator considers failure of these banks to be dangerous for the
economy, which renders them ‘too big to fail’.
RBI had designated both these banks as domestic systemically
important banks (D-SIBs) last year in August for the first time.

www.bankingpdf.com
Both these banks are required to make additional capital requirements
after they were named as D-SIBs.

10. SBI has been named as domestic systemically important


banks (D-SIBs) this year. What additional capital it will have to
maintain as a percent of loans and investments?
A) 0.2%
B) 0.4%
C) 0.6%
D) 0.8%
E) None of these

Answer & Explanation

C) 0.6%
Explanation:
Based on the methodology announced by the central bank, SBI will
need to maintain additional capital equivalent to 0.6 per cent of its
loans and investments while ICICI will need to maintain 0.2 per cent
more.

280-290 Questions :

1. Which is a full service credit rating agency exclusively set up


for micro, small and medium enterprises?
A. CRISIL
B. ICRA
C. CARE
D. ONICRA
E. SMERA

www.bankingpdf.com
Answer & Explanation

E. SMERA
Explanation:
SMERA Ratings Limited (formerly SME Rating Agency of India Ltd.) is a
full service credit rating agency exclusively set up for micro, small and
medium enterprises.

2. SMERA is a joint initiative of _________


A. SIDBI & Leading PSBs
B. Dun & Bradstreet Information Services India Private Limited (D&B)
& Leading Private Banks
C. RBI & SEBI
D. Both (A) and (B)
E. Both (A) and (C)

Answer & Explanation

D. Both (A) and (B)


Explanation:
SMERA Ratings Limited (formerly SME Rating Agency of India Ltd.) is a
joint initiative of Small Industries Development Bank of India (SIDBI),
Dun & Bradstreet Information Services India Private Limited (D&B) and
leading public and private sector banks in India.

3. The Headquarters of SMERA is located in _______


A. Mumbai
B. New Delhi
C. Hyderabad
D. Gurugram
E. None of these

Answer & Explanation

www.bankingpdf.com
A. Mumbai
Explanation:
The Headquarters of SMERA is located in Mumbai, Maharashtra.

4. SMERA Ratings Ltd (SMERA) is registered by ______


A. SEBI
B. SIDBI
C. NABARD
D. RBI
E. None of these

Answer & Explanation

A. SEBI
Explanation:
SMERA is registered with the Securities and Exchange Board of India
(SEBI) as a Credit Rating Agency.

5. SMERA is the ______ rating agency in the country?


A. First
B. Sixth
C. Seventh
D. Eighth
E. None of these

Answer & Explanation

B. Sixth
Explanation:
SMERA is registered with the Securities and Exchange Board of India
(SEBI) as a Credit Rating Agency (6th in India).

6. Credit Rating Agency, SMERA accredited by ________


A. SEBI
B. SIDBI

www.bankingpdf.com
C. NABARD
D. RBI
E. None of these

Answer & Explanation

D. RBI
Explanation:
The Company has received accreditation from the Reserve Bank of
India (RBI) as an External Credit Assessment Institution (ECAI) under
BASEL – II norms for undertaking bank loan ratings.

7. Which of the following instruments rated/graded by SMERA?


A. Commercial Papers
B. Security Receipts
C. Fixed Deposits
D. Only (B) and (C)
E. All of these

Answer & Explanation

E. All of these
Explanation:
SMERA can rate/grade various instruments such as: IPO, NCDs,
Commercial Papers, Bonds, Security Receipts, Fixed Deposits etc

8. SMERA is also em-panelled as an approved rating agency


by_______
A. SIDBI
B. NABARD
C. NSIC
D. Both (B) and (C)
E. None of these

Answer & Explanation

www.bankingpdf.com
C. NSIC
Explanation:
SMERA is also em-panelled as an approved rating agency by the
National Small Industries Corporation Ltd. (NSIC) under the
“Performance & Credit Rating Scheme for Small Industries”, approved
by the Ministry of Small Scale Industries, Government of India.

9. Which of the following has awarded SIDBI with “Outstanding


Development Project Award” for setting up SMERA?
A. Standard & Poor
B. IBRD
C. ADB
D. ADFIAP
E. None of these

Answer & Explanation

D. ADFIAP
Explanation:
The Association of Development Financing Institutions in Asia and the
Pacific (ADFIAP) has awarded SIDBI with “Outstanding Development
Project Award” for setting up SMERA in 2007.

10. SMERA has been registered under _______


A. Securities and Exchange Board of India Act, 1992
B. Reserve Bank of India Act, 1934
C. Banking Regulation Act, 1949
D. Securities and Exchange Board of India (Credit Rating Agencies)
Regulations, 1999
E. None of these

Answer & Explanation

D. Securities and Exchange Board of India (Credit Rating


Agencies) Regulations, 1999

www.bankingpdf.com
Explanation:
SMERA has been registered under Securities and Exchange Board of
India (Credit Rating Agencies) Regulations, 1999.

290-300 Questions :

1. Which of the following is India’s first Credit Information


Company?
A. CRISIL
B. CIBIL
C. SMERA
D. CERSAI
E. CARE

Answer & Explanation

B. CIBIL
Explanation:
TransUnion CIBIL Limited is India’s first Credit Information Company,
also commonly referred as a Credit Bureau. It collect and maintain
records of individuals’ and non-individuals’ (commercial entities)
payments pertaining to loans and credit cards. These records are
submitted to credit bureau by banks and other lenders on a monthly
basis; using this information a Credit Information Report (CIR) and
Credit Score is developed, enabling lenders to evaluate and approve
loan applications.

2. Which of the following provides license to a credit bureau?


A. SEBI
B. SIDBI
C. NABARD

www.bankingpdf.com
D. RBI
E. GOI

Answer & Explanation

D. RBI
Explanation:
A Credit Bureau is licensed by the RBI.

3. How many divisions are there in CIBIL?


A. one
B. Two
C. Three
D. Four
E. None of the Above

Answer & Explanation

B. Two
Explanation:
It has two divisions : Consumer Bureau and Commercial Bureau

4. A Credit Bureau is governed by _________


A. Securities and Exchange Board of India Act, 1992
B. Reserve Bank of India Act, 1934
C. Banking Regulation Act, 1949
D. Securities and Exchange Board of India (Credit Rating Agencies)
Regulations, 1999
E. Credit Information Companies (Regulation) Act of 2005

Answer & Explanation

E. Credit Information Companies (Regulation) Act of 2005


Explanation:
A Credit Bureau is governed by the Credit Information Companies
(Regulation) Act of 2005.

www.bankingpdf.com
5. Which of the following plays an important role in the loan
approval process?
A. Credit Information Report (CIR)
B. CIBIL TransUnion Score
C. Only A
D. Both (A) and (B)
E. None of the Above

Answer & Explanation

D. Both (A) and (B)


Explanation:
The Credit Information Report (CIR) and CIBIL TransUnion Score are
very important in the loan approval process.

6. Which of the following is/are technical partners of CIBIL?


A. TransUnion International
B. Dun and Bradstreet
C. SIDBI & PSBs
D. Both (A) and (B)
E. Both (A) and (C)

Answer & Explanation

D. Both (A) and (B)


Explanation:
TransUnion International and Dun and Bradstreet are technical
partners of CIBIL.

7. Which of the following holds 4% stake in CIBIL?


A. Bank of India
B. TransUnion International
C. Aditya Birla Trustee Company Private Ltd
D. ICICI Bank Ltd
E. India Infoline Finance Limited

www.bankingpdf.com
Answer & Explanation

C. Aditya Birla Trustee Company Private Ltd


Explanation:
Aditya Birla Trustee Company Private Ltd – 4%

8. Which of the following holds minority stake in CIBIL?


A. Bank of India
B. TransUnion International
C. Aditya Birla Trustee Company Private Ltd
D. ICICI Bank Ltd
E. India Infoline Finance Limited

Answer & Explanation

E. India Infoline Finance Limited


Explanation:
India Infoline Finance Limited – 1%

9. Which of the following holds majority stake in CIBIL?


A. Bank of India
B. Bank of Baroda
C. United Bank of India
D. TransUnion International
E. ICICI Bank Ltd

Answer & Explanation

D. TransUnion International
Explanation:
TransUnion International Inc (66.1%)
ICICI Bank Ltd (6)%
Bank of Baroda (5)%
Bank of India (5)%
Union Bank of India (5)%

www.bankingpdf.com
Indian Overseas Bank (5)%
Aditya Birla Trustee Company Private Ltd. (4)%
India Alternatives Private Equity Fund (2.9%)
India Infoline Finance Limited (1%)

10. Which of the following is the slogan of CIBIL?


A. Empowering Banks
B. Empowering Women
C. Empowering India
D. Empowering you
E. None of the Above

Answer & Explanation

D. Empowering you
Explanation:
Slogan: Empowering you

300-310 Questions :

1. Which of the following is an apex financial institution for


housing?
A. AIFI
B. NHB
C. DFI
D. Both (A) and (B)
E. None of the Above

Answer & Explanation

B. NHB
Explanation:
NHB is an apex financial institution for housing. NHB has been

www.bankingpdf.com
established with an objective to operate as a principal agency to
promote housing finance institutions both at local and regional levels.

2. NHB is wholly owned by ______


A. GOI
B. RBI
C. SEBI
D. NABARD
E. None of the Above

Answer & Explanation

B. RBI
Explanation:
NHB is wholly owned by Reserve Bank of India.

3. The Headquarters of NHB located in ________


A. Chennai
B. Mumbai
C. New Delhi
D. Gurugram
E. None of the Above

Answer & Explanation

C. New Delhi
Explanation:
Headquarters of NHB located in New Delhi.

4. National Housing Bank(NHB) was set up under ___________


A. National Housing Bank Act, 1987
B. National Housing Bank Act, 1977
C. National Housing Bank Act, 1967
D. National Housing Bank Act, 1957
E. None of the Above

www.bankingpdf.com
Answer & Explanation

A. National Housing Bank Act, 1987


Explanation:
NHB was set up on 9 July 1988 under the National Housing Bank Act,
1987.

5. Which of the following contributed the entire paid-up capital in


National Housing Bank(NHB)?
A. GOI
B. RBI
C. SEBI
D. NABARD
E. None of the Above

Answer & Explanation

B. RBI
Explanation:
RBI contributed the entire paid-up capital in National Housing
Bank(NHB).

6. An HFC is required to have Minimum net owned fund of ______


A. Rs. 100 lakhs
B. Rs. 200 lakhs
C. Rs. 500 lakhs
D. Rs. 1000 lakhs
E. None of the Above

Answer & Explanation

D. Rs. 1000 lakhs


Explanation:
For commencing the housing finance business, an HFC is required to
have the following in addition to the requirements under the

www.bankingpdf.com
Companies Act, 1956:
♦ Certificate of registration from NHB
♦ Minimum net owned fund of Rs. 1000 lakhs

7. HFCs can accept public deposits for a period of __________


A. 2 to 5 years
B. 1 to 5 years
C. 2 to 7 years
D. 1 to 7 years
E. All of the Above

Answer & Explanation

D. 1 to 7 years
Explanation:
In terms of the Housing Finance Companies (NHB) Directions, 2001,
HFCs can accept public deposits for periods of one year and above and
upto seven years only.

8. What is/are the credit rating agencies approved for the


acceptance of public deposits by an HFC?
A. The Credit Rating Information Services of India Ltd. (CRISIL)
B. ICRA Ltd.
C. Credit Analysis and Research Limited (CARE)
D. FITCH Ratings India Pvt. Ltd.
E. All of the Above

Answer & Explanation

E. All of the Above


Explanation:
The HFC having credit rating can accept more deposits as compared to
an HFC without such rating.
The following credit rating agencies have been approved for the above
purpose

www.bankingpdf.com
♦ The Credit Rating Information Services of India Ltd. (CRISIL)
♦ ICRA Ltd.
♦ Credit Analysis and Research Limited (CARE)
♦ FITCH Ratings India Pvt. Ltd.

9. What is difference between banks & HFCs?


A. HFCs cannot accept demand deposits
B. It cannot issue cheques drawn on itself
C. deposit insurance facility of Deposit Insurance and Credit Guarantee
Corporation is not available to depositors of HFCs
D. All of the Above
E. None of the Above

Answer & Explanation

D. All of the Above


Explanation:
HFCs are doing functions similar to banks as banks also provides
housing loans.
However, there are a few differences as given below:
♦ HFCs cannot accept demand deposits
♦ HFCs do not form part of the payment and settlement system and
cannot issue cheques drawn on itself;
♦ Deposit insurance facility of Deposit Insurance and Credit Guarantee
Corporation(DICGC) is not available to depositors of HFCs, unlike in
case of banks.

10. In terms of _________, HFCs cannot conduct business of


housing finance without obtaining a Certificate of Registration
(CoR) from NHB
A. Section 29A of National Housing Bank 1987
B. Section 28A of National Housing Bank 1987
C. Section 27A of National Housing Bank 1987

www.bankingpdf.com
D. Section 26A of National Housing Bank 1987
E. None of the Above

Answer & Explanation

A. Section 29A of National Housing Bank 1987


Explanation:
In terms of section 29A of the National Housing Bank, 1987, HFCs
cannot conduct business of housing finance without obtaining a
Certificate of Registration (CoR) from NHB. Conduct of business
without obtaining certificate of registration is an offence punishable
under the provisions of the National Housing Bank Act, 1987. NHB can
also file application for winding up of such HFCs, under section 33B of
the said Act.

310-320 Questions :

1. Which of the following is an independent banking industry


watchdog that protects consumers of banking services in the
country?
A. BBB
B. IBA
C. BCSBI
D. IBRD
E. None of the Above

Answer & Explanation

C. BCSBI
Explanation:
The Banking Codes and Standards Board of India (BCSBI) is an
independent banking industry watchdog that protects consumers of
banking services in India.

www.bankingpdf.com
2. Which of the following is the headquarters of BCSBI?
A. Chennai
B. New Delhi
C. Mumbai
D. Hyderabad
E. None of the Above

Answer & Explanation

C. Mumbai
Explanation:
Headquarters of BCSBI is located in Mumbai.

3. BCSBI is governed by a _______ governing Council including


one Chairman.
A. six-member
B. Four-member
C. Five-member
D. Three-member
E. None of the Above

Answer & Explanation

A. six-member
Explanation:
The board is governed by a six-member governing Council including
one Chairman.

4. BCSBI registered as an independent and autonomous body


under ________
A. RBI Act 1934
B. Societies Registration Act, 1860
C. Banking Regulation Act, 1949
D. The Companies Act, 1956
E. Both (B) and (C)

www.bankingpdf.com
Answer & Explanation

B. Societies Registration Act, 1860


Explanation:
BCSBI is an independent and autonomous body, registered as a
separate society under the Societies Registration Act, 1860.

5. Which of the following is/are non-banking subsidiary of the


State Bank of India?
A. SBI Capital Markets Ltd.
B. SBI General Insurance Company Limited.
C. SBI Funds Management Pvt Ltd.
D. All of the Above
E. None of the Above

Answer & Explanation

D. All of the Above


Explanation:
SBI has the following Non-Banking Subsidiaries in India:
SBI Capital Markets Ltd.
SBI Funds Management Pvt Ltd.
SBI Global Factors Ltd.
SBI Cards & Payments Services Pvt. Ltd. (SBICPSL)
SBI DFHI Ltd.
SBI General Insurance Company Limited.
SBI Pension Funds Pvt Ltd (SBIPFPL)

6. The International Bank for Reconstruction and


Development(IBRD) is an international financial institutions
that offer loans to _____
A. Multinational Banks
B. Multinational Financial Institutions
C. Middle Income Developing Countries

www.bankingpdf.com
D. Non Government Organizations
E. None of the Above

Answer & Explanation

C. Middle Income Developing Countries


Explanation:
The International Bank for Reconstruction and Development (IBRD) is
an international financial institution that offers loans to middle-income
developing countries.

7. The “in-principle” approval for setting up ‘small finance banks’


granted by RBI will be valid for ______
A. 12 Months
B. 24 Months
C. 10 Months
D. 18 Months
E. None of the Above.

Answer & Explanation

D. 18 Months
Explanation:
The “in-principle” approval granted will be valid for 18 months to
enable the applicants to comply with the requirements under the
Guidelines and fulfil other conditions as may be stipulated by the RBI.

8. Which of the following is an independent commodity exchange


based in Mumbai?
A. MGEX
B. MCE
C. MDEX
D. MCX
E. MEX

www.bankingpdf.com
Answer & Explanation

D. MCX
Explanation:
Multi Commodity Exchange of India Ltd (MCX) (BSE: 534091) is an
independent commodity exchange based in India. It was established in
2003 and is based in Mumbai.

9. Which of the following state is known as “Cradle of Indian


Banking”?
A. Kerala
B. Maharashtra
C. Tamil Nadu
D. Karnataka
E. Uttar Pradesh

Answer & Explanation

D. Karnataka
Explanation:
The state of Karnataka, particularly the region comprising the coastal
districts of Dakshina Kannada and Udupi is called as the cradle of
banking in India. This is because seven of the country’s leading banks,
Canara Bank, Syndicate Bank, Corporation Bank, Vijaya Bank,
Karnataka Bank, Vysya Bank and the State Bank of Mysore originated
from this state.

10. Which of the following is a pension scheme focused on the


workers in the unorganiaed sector?
A. SSY
B. AMRUT
C. APY
D. PMAY
E. None of the Above

www.bankingpdf.com
Answer & Explanation

C. APY
Explanation:
Atal Pension Yojna (APY) a guaranteed pension scheme administered
by PFRDA and established by Government of India. This scheme mainly
for workers in unorganised sector.

320-330 Questions :

1. The safest form of Crossed Cheque is _____


A. Double Crossing
B. General Crossing
C. Special Crossing
D. Account payee crossing
E. None of the Above

Answer & Explanation

D. Account payee crossing


Explanation:
In Account payee crossing the amount will not be paid to anybody over
the counter. It will be credited to the account of the payee only. Thus
account payee crossing ensures safe transfer of funds.

2. _______ is a cheque that is not crossed on the left corner and


payable at the counter of the drawee bank on presentation of
the cheque.
A. Crossed Cheque
B. Open Cheque
C. General Crossing
D. Restrictive Crossing
E. None of the Above

www.bankingpdf.com
Answer & Explanation

B. Open Cheque
Explanation:
An open cheque is a cheque that is not crossed on the left corner and
payable at the counter of the drawee bank on presentation of the
cheque.

3. The words ‘not negotiable’ can be added to _______


A. Double Crossing
B. General Crossing
C. Special Crossing
D. Both (B) and (C)
E. Both (A) and (B)

Answer & Explanation

D. Both (B) and (C)


Explanation:
The words ‘not negotiable’ can be added to general-crossing as well as
special-crossing and a crossing with these words is known as not
negotiable crossing. The effect of such a crossing is that it removes the
most important characteristic of a negotiable instrument.

4. Which of the following endorsements is not valid?


A. Restrictive Endorsement
B. Partial Endorsement
C. Conditional Endorsement
D. Facultative Endorsement
E. Both (A) and (B)

Answer & Explanation

B. Partial Endorsement
Explanation:

www.bankingpdf.com
If the bill purposes to be endorsed for a part of the amount payable,
the endorsement is called partial. It is legally ineffective.

5. If the endorsement makes the payment of a bill subject to the


fulfilment of a condition the endorsement is called _______
A. Restrictive Endorsement
B. Partial Endorsement
C. Conditional Endorsement
D. Facultative Endorsement
E. Both (A) and (B)

Answer & Explanation

C. Conditional Endorsement
Explanation:
If the endorsement makes the payment of a bill subject to the
fulfilment of a condition the endorsement is called Conditional
Endorsement.

6. Which of the following endorsement specifies the name of the


transferee for the payment of the bill?
A. Restrictive Endorsement
B. Partial Endorsement
C. Conditional Endorsement
D. Facultative Endorsement
E. Special Endorsement

Answer & Explanation

E. Special Endorsement
Explanation:
An endorsement which specifies the name of the transferee for the
payment of the bill is called special endorsement.

www.bankingpdf.com
7. If the endorser signs, but does not give his/her name to whom
he/she wishes to transfer the cheque is called _________
A. Restrictive Endorsement
B. Partial Endorsement
C. Conditional Endorsement
D. Facultative Endorsement
E. Blank Endorsement

Answer & Explanation

E. Blank Endorsement
Explanation:
If the endorser signs, but does not give his name to whom he wishes
to transfer the cheque is called Blank Endorsement.

8. When an endorsee give up some of his/her rights then it is


called __________
A. Restrictive Endorsement
B. Partial Endorsement
C. Conditional Endorsement
D. Facultative Endorsement
E. Blank Endorsement

Answer & Explanation

D. Facultative Endorsement
Explanation:
When notice of dishonour is received by the endorsee instead of
endorser it is known as facultative endorsement.

9. The endorser expresses that he/she would not be liable for any
expenses incurred by endorsee or any holder, in case of
dishonour of the instrument the endorsement is called
__________
A. Restrictive Endorsement

www.bankingpdf.com
B. Partial Endorsement
C. Conditional Endorsement
D. Facultative Endorsement
E. Sans recourse Endorsement

Answer & Explanation

E. Sans recourse Endorsement


Explanation:
The endorser expresses that he would not be liable for any expenses
incurred by endorsee or any holder, in case of dishonour of the
instrument the endorsement is called Sans recourse Endorsement.

10. Which of the following is an endorsement that restrict the


further negotiation of the bill?
A. Restrictive Endorsement
B. Partial Endorsement
C. Conditional Endorsement
D. Facultative Endorsement
E. Sans recourse Endorsement

Answer & Explanation

A. Restrictive Endorsement
Explanation:
An endorsement which restrict the further negotiation of the bill.

330-340 Questions :

1. The seed capital of Bharatiya Mahila Bank is ________


A. Rs.5000 crore
B. Rs.1000 crore
C. Rs.500 crore

www.bankingpdf.com
D. Rs.100 crore
E. None of the above

Answer & Explanation

B. Rs.1000 crore
Explanation:
Bharatiya Mahila Bank (BMB) is an Indian financial services banking
company based in New Delhi, India. India is the third country in the
world to have a bank especially for women, after Pakistan and
Tanzania.The Bank’s initial capital consists of Rs 1,000 crores.

2. Which of the following is an investment advisory discipline?


A. Corporate Industrial Finance
B. Offshare Banking
C. Wholesale Banking
D. Wealth Management
E. Trade Finance

Answer & Explanation

D. Wealth Management
Explanation:
Wealth management as an investment-advisory discipline incorporates
financial planning, investment portfolio management and a number of
aggregated financial services. High-net-worth individuals (HNWIs),
small-business owners and families who desire the assistance of a
credentialed financial advisory specialist call upon wealth managers to
coordinate retail banking, estate planning, legal resources, tax
professionals and investment management.

3. Which of the following economic concepts is categorised on the


basis of Current Account or Capital Account or both?
A. Balance of Payments
B. Value of the food grain stock of a country

www.bankingpdf.com
C. Gross National Product
D. Gross National Income(GNI)
E. Total collection of Direct Taxes in a year

Answer & Explanation

A. Balance of Payments
Explanation:
A Balance of payments statements is a summary of a nation’s total
economic transactions undertaken on international account. It is
usually composed of two sections :-
1. Current Account
2. Capital Account

4. When there is a difference between all receipts and expenditure


of the Government of India both capital and revenue it is called
__________
A. Revenue Deficit
B. Budgetary Deficit
C. Zero Budgeting
D. Trade Gap
E. Balance of Payment Problem

Answer & Explanation

B. Budgetary Deficit
Explanation:
Budgetary deficit is the difference between all receipts and expenses in
both revenue and capital account of the government.If revenue
expenses of the government exceed revenue receipts, it results in
revenue account deficit. Similarly, if the capital disbursements of the
government exceed capital receipts, it leads to capital account deficit.
Budgetary deficit is usually expressed as a percentage of GDP.

www.bankingpdf.com
5. Lack of access to financial services is technically known as
______
A. Financial Instability
B. Financial Stability
C. Financial Inclusion
D. Financial Exclusion
E. Poverty

Answer & Explanation

D. Financial Exclusion
Explanation:
Financial exclusion can be defined as the unavailability of banking
services to people with low or non income. It is believed to be one
factor preventing poor people leave out poverty.

6. Sukanya Samriddhi Account can be opened up to age of


________ years only from the date of birth
A. Five years
B. Four years
C. Six years
D. Eight years
E. Ten years

Answer & Explanation

E. Ten years
Explanation:
Sukanya Samriddhi Account can be opened up to age of 10 years only
from the date of birth.

7. If minimum Rs.1000 is not deposited in Sukanya Samriddhi


Account in a financial year, account will become discontinued
and can be revived with a penalty of _______ per year.
A. Rs.50

www.bankingpdf.com
B. Rs.100
C. Rs.25
D. Rs.75
E. Rs.125

Answer & Explanation

A. Rs.50
Explanation:
In Sukanya Samriddhi Account, if minimum Rs 1000/- is not deposited
in a financial year, account will become discontinued and can be
revived with a penalty of Rs 50/- per year with minimum amount
required for deposit for that year.

8. In Sukanya Samriddhi Account, What is the maximum limit on


number of deposits either in a month or in a Financial year?
A. 5
B. 4
C. 6
D. 8
E. No limit

Answer & Explanation

E. No limit
Explanation:
No limit on number of deposits either in a month or in a Financial year.

9. Investment under senior citizen savings scheme qualifies for


the benefit of __________ of the Income Tax Act, 1961.
A. Section 60C
B. Section 70C
C. Section 80C
D. Section 90C
E. None of the Above

www.bankingpdf.com
Answer & Explanation

C. Section 80C
Explanation:
Investment under this scheme qualifies for the benefit of Section 80C
of the Income Tax Act, 1961.

10. What is the maturity period for senior citizen savings


scheme?
A. 3 years
B. 5 years
C. 2 years
D. 5 years
E. 4 years

Answer & Explanation

B. 5 years
Explanation:
There shall be only one deposit in the account in multiple of
INR.1000/- maximum not exceeding INR 15 lakh in senior citizen
savings scheme. Maturity period is 5 years.

340-350 Questions :

1. Axis Bank has tied up with which payments company to operate


its Unified Payments Interface (UPI) enabled app?
A) PayU
B) Paytm
C) Freecharge
D) Mobikwik
E) Oxigen

Answer & Explanation

www.bankingpdf.com
C) Freecharge
Explanation:
Country’s third largest private sector lender Axis Bank said it is ready
with a Unified Payments Interface (UPI) app, while it also announced a
tie-up with digital payments company Freecharge for it.

2. Yes Bank in August rolled out which facility under its ‘Yes We
Care’ initiative?
A) 2 months leave for new father
B) 6 months leave for pregnant women
C) work for home
D) adoption leave
E) None of these

Answer & Explanation

D) adoption leave
Explanation:
The new policy is called “Bringing A Beautiful Yes”.
Private sector lender Yes Bank has rolled out adoption benefits for its
workforce, in an effort to align organisational objectives with the needs
of employees.
Realising that employees need to spend time building an emotional
connect with new additions to the family and recognising that many
are choosing to adopt children, the organisation has introduced the
benefits, which are applicable 1st August onwards.

3. Kotak Mahindra Bank has enabled which of the following banks


online shopping portal on its mobile banking app to provide
seamless shopping experience for its customers?
A) Flipkart
B) Snapdeal
C) Amazon India

www.bankingpdf.com
D) Ebay India
E) Mytra

Answer & Explanation

A) Flipkart
Explanation:
Private sector lender Kotak Mahindra Bank has enabled online shopping
portal Flipkart in its mobile banking application in order to ensure a
seamless shopping experience for its customers.
“The biggest problem faced by consumers is the transaction failure
during the payment mode. Through this integration the bank will try to
solve that problem for our customers.

4. The Reserve Bank of India will transfer how much surplus to


the government for FY’16 as dividend?
A) Rs 68876 crore
B) Rs 65885 crore
C) Rs 75876 crore
D) Rs 65876 crore
E) Rs 65776 crore

Answer & Explanation

D) Rs 65876 crore
Explanation:
The Reserve Bank of India will Rs 65876 crore as surplus transfer to
the government for FY’16, Rs 20 crore less that previous year’s Rs
65896 crore, a central bank release said.
The surplus approved by the Reserve Bank of India board could be
used to fund the centre’s fiscal deficit .

5. Recently RBI has made necessary to have certification for bank


employees involved in specialised jobs. The deadline to get this
certification is

www.bankingpdf.com
A) April 1, 2017
B) April 1, 2018
C) January 1, 2018
D) January 1, 2017
E) July 1, 2017

Answer & Explanation

B) April 1, 2018
Explanation:
Bank employees working in specialised areas such as treasury, risk
management and accounting will need to be specially trained and go
through certification programmes, Reserve Bank of India directed
banks.
The central bank said that bank staff would have obtain the requisite
certification before April 1, 2018 for getting posted in these functional
areas.

6. Which bank has launched USSD based mobile app for basic
banking needs with NPCI
A) United Bank of India
B) Union Bank of India
C) UCO Bank
D) Indian Bank
E) Indian Overseas Bank

Answer & Explanation

B) Union Bank of India


Explanation:
National Payments Corporation of India (NPCI), the umbrella
organisation for all retail payments system in the country has
partnered with Union Bank of India to launch an USSD based *99#
mobile application for their customers.
This application can be downloaded in an android smartphone from the

www.bankingpdf.com
Google Play Store by the bank customers and can be used even
without any internet connection as it does not require any data.

7. SBI card is going to launch a co-branded credit cards for the


customers of private sector bank
A) Karnataka Bank
B) Federal Bank
C) City Union
D) South Indian Bank
E) RBL Bank

Answer & Explanation

D) South Indian Bank


Explanation:
South Indian Bank, leading private sector bank, and SBI Card, one of
India’s leading Credit Card issuers, announced the launch of co-
branded credit cards for the customers of South Indian Bank.

8. Times Internet has partnered with which Bank to launch co-


branded Debit Card?
A) Punjab National Bank
B) HDFC Bank
C) ICICI Bank
D) SBI
E) None of these

Answer & Explanation

B) HDFC Bank
Explanation:
Times Internet (part of Bennett, Coleman & Co that owns The
Economic Times), in partnership with HDFC Bank, announced the
launch of a co-branded card on Thursday called the Times Points Debit
Card.

www.bankingpdf.com
9. Which Financial institutions has announced a comprehensive
Housing finance scheme called prapti with an attractive 4%
interest rate?
A) Tata Capita Housing
B) LIC Housing
C) Reliance Home
D) HDFC Ergo
E) Housing and Urban Development Corporation Ltd.

Answer & Explanation

A) Tata Capita Housing


Explanation:
Tata Capita Housing Limited announced a Comprehensive Housing
finance scheme called Prapti with an attractive 4% interest rate for
people earning less than Rs 6 lakh a year.

10. Which bank partnered with a fintech start-up Fisdom for


personal wealth management services?
A) Bank of India
B) Allahabad Bank
C) Union Bank of India
D) Bank of Baroda
E) None of these

Answer & Explanation

D) Bank of Baroda
Explanation:
Vadodara headquartered Public Sector lender Bank of Baroda has
entered into partnership with a fintech start-up Fisdom for personal
wealth management services.

www.bankingpdf.com
350-360 Questions :

1. What does amortization mean?


A. Spreading payments over multiple periods
B. Buying a financial instrument in order to sell the same instrument at
a higher price in another market
C. Issuance approval,by a credit card issuer
D. Buying the underlying securities at a specified excercise price
E. None of the Above

Answer & Explanation

A. Spreading payments over multiple periods


Explanation:
Amortization is an accounting term that refers to the process of
allocating the cost of an intangible asset over a period of time. It also
refers to the repayment of loan principal over time.

2. What is the duration of Public Provident Fund (PPF) account?


A. 3 years
B. 5 years
C. 9 years
D. 15 years
E. 18 years

Answer & Explanation

D. 15 years
Explanation:
A PPF account matures in 15 years, but you can extend the tenure in
blocks of five years after maturity. The balance continues to earn
interest at the normal rate. The minimum investment of Rs 500 has to
be maintained even for accounts extended beyond 15 years.

www.bankingpdf.com
3. Under which Act are the KYC norms implemented?
A. SEBI Act 1992
B. Foreign Contribution and Regulation Act, 1976
C. Prevention of Money laundering Act 2002
D. Banking Regulation Act, 1949
E. Both (C) and (D)

Answer & Explanation

C. Prevention of Money laundering Act 2002


Explanation:
KYC guidelines/instructions are issued under under Prevention of
Money laundering Act PMLA, 2002.

4. When a customer deposits cheque to a bank, the bank is a(n)


A. Drawer
B. Agent
C. Principal
D. Drawee
E. Bailee

Answer & Explanation

B. Agent
Explanation:
When a customer deposits cheque to a bank then that particular bank
acts as an agent for the customer. Collection occurs when the drawee
bank (the bank ordered by the cheques to make payment) takes funds
from the account of the drawer (its customer who has written the
cheque) and presents it to the collecting bank.

5. Which of the following is a receipt, declaring ownership of


shares of a foreign company. which can be listed in India and
traded in rupees?
A. ADR

www.bankingpdf.com
B. GDR
C. IDR
D. EDR
E. None of the Above

Answer & Explanation

C. IDR
Explanation:
An IDR(Indian Depository Receipts)is a receipt, declaring ownership of
shares of a foreign company. These receipts can be listed in India and
traded in rupees.

6. What is the term for a bank without any branch network that
offers its services remotely?
A. Internet only Banks
B. Direct Bank
C. Lending Institution
D. Indirect Bank
E. Online Bank

Answer & Explanation

B. Direct Bank
Explanation:
A direct bank is a bank without any branch network that offers its
services remotely via online banking and telephone banking and may
also provide access via ATMs (often through interbank network
alliances), mail and mobile. By eliminating the costs associated with
bank branches, direct banks can make significant savings which they
may pass on to clients via higher interest rates or lower service
charges.

7. Derivatives contract which gives the buyer/holder of the


contract the right(but not the obligation) to buy/sell the

www.bankingpdf.com
underlying asset at a predetermined price within or at end of a
specified period is known as
A. Futures contract
B. Option Contract
C. Index Futures contract
D. Mini Derivative contract
E. None of the Above

Answer & Explanation

B. Option Contract
Explanation:
Options Contract is a type of Derivatives Contract which gives the
buyer/holder of the contract the right (but not the obligation) to
buy/sell the underlying asset at a predetermined price within or at end
of a specified period.

8. For expanding access to banking services, the RBI has advised


banks to open branches with minimum infrastructure support of
8 to 10BC units at a reasonable 3-4km. Such branches are
known as _______
A. White Label ATMs
B. Ultra Small Branches
C. Banking Kiosks
D. CBS Terminals
E. ICT Hubs

Answer & Explanation

B. Ultra Small Branches


Explanation:
Ultra Small Branches may be set up between the base branch and BC
locations so as to provide support to about 8-10 BC Units at a
reasonable distance of 3-4 kilometres. These could be either newly set
up or by conversion of the BC outlets. Such Ultra Small Branches

www.bankingpdf.com
should have minimum infrastructure such as a Core Banking Solution
(CBS) terminal linked to a pass book printer and a safe for cash
retention for operating large customer transaction and would have to
be managed full time by bank officers/ employees.

9. According to the Income Tax act of 1961, the age of Super


senior Citizens should be _______
A. 60 years
B. 70 years
C. 80 years
D. 75 years
E. 65 years

Answer & Explanation

C. 80 years
Explanation:
According to the Income Tax act of 1961, the age of Super senior
Citizens is 80 years.

10. Part of company’s earning or profit which is paid out to


share holders is known as _______
A. Premium
B. Dividend
C. Bonus
D. Sum Assured
E. Return

Answer & Explanation

B. Dividend
Explanation:
A dividend is a payment made by a corporation to its shareholders,
usually as a distribution of profits. When a corporation earns a profit or

www.bankingpdf.com
surplus, it can re-invest it in the business (called retained earnings),
and pay a fraction of the profit as a dividend to shareholders.

360-370 Questions :

1. Which of the following is the most liquid money?


A. M1
B. M2
C. Securities
D. M3
E. Call Money

Answer & Explanation

A. M1
Explanation:
The most liquid portions of the money supply are measured by M1
because it contains currency and assets that can be converted to cash
quickly. “Near money” and “near, near money,” which fall under M2
and M3, cannot be converted to currency as quickly.

2. Which of the following can issue commercial paper for raising


short term funds?
A. Corporates
B. Primary Dealers
C. All-India Financial Institutions
D. Both (B) and (C)
E. All of the Above

Answer & Explanation

E. All of the Above


Explanation:

www.bankingpdf.com
A Commercial Paper (CP) is an unsecured money market instrument
issued as a promissory note. It is a short-term funding tool that highly
rated companies, primary dealers (PDs), and all-India financial
institutions (AIFIs) can use, typically to meet short-term funding and
working capital requirements.

3. Alterations, if any, on cheques are required to be authenticated


by the signature of _______
A. Payee
B. Depositor
C. Drawer
D. Endorse
E. All of the above

Answer & Explanation

C. Drawer
Explanation:
Alterations, if any, on cheques are required to be authenticated by the
drawer’s signature against each such alteration.

4. What is the purpose of KYC in banking?


A. It is used for customer identification
B. It is used for increasing the CRR of banks
C. It is used against money laundering
D. It is used by the central bank to control liquidity
E. Both (A) and (C)

Answer & Explanation

E. Both (A) and (C)


Explanation:
Know your customer (KYC) is the process of a business verifying the
identity of its clients. Know your customer policies are becoming much

www.bankingpdf.com
more important globally to prevent identity theft, financial fraud,
money laundering and terrorist financing.

5. Which of the following types of cards is free from credit risk?


A. Credit card
B. Debit Card
C. Prepaid Cards
D. Charge Cards
E. Both (C) and (D)

Answer & Explanation

B. Debit Card
Explanation:
A debit card a safe, convenient, risk-free alternative to cash and has
no credit risks attached.

6. Which of the following types of economy is an example of


sustainable economy?
A. Capitalist Economy
B. Mixed Economy
C. Socialist Economy
D. Traditional Economy
E. None of the Above

Answer & Explanation

D. Traditional Economy
Explanation:
Traditional economies are considered sustainable because they are less
harmful to the environment.

7. In the field of banking, what does ADF stand for?


A. Additional Dearness Allowance
B. Automated Data Flow

www.bankingpdf.com
C. Additional Deposit Allowance
D. Automated Deposit Allowance
E. None of the Above

Answer & Explanation

B. Automated Data Flow


Explanation:
The Automated Data Flow (ADF) is a unique initiation by RBI aimed at
the purpose of meeting the needs for accurate and consistent flow of
data. Automated Data Flow ensures that the data submitted from the
banks to Reserve Bank of India is correct and consistent without any
kind of manual intervention.

8. Section 80E of the Income Tax Act provides for deduction of


interest paid on Education or study loan taken for higher
education. Deduction under section 80E is available for _____
A. 2 years
B. 4 years
C. 8 years
E. 10 years

Answer & Explanation

C. 8 years
Explanation:
An education loan can not only fund your higher studies but also help
save tax. The interest paid on the education loan can be claimed as
deduction, as per Section 80E of the Income Tax Act, 1961.The
deduction on education can be claimed only when you start the
repayment and is available up to eight years, or until the payment of
interest in full, whichever is earlier.

9. A financial contract that derives its value from another asset or


an index of asset values is known as __________

www.bankingpdf.com
A. Insurance Policy
B. Security
C. Derivative
D. Share
E. None of the above

Answer & Explanation

C. Derivative
Explanation:
In finance, a derivative is a contract that derives its value from the
performance of an underlying entity.

10. Inflation refers to a fall in the value of money. Which of the


following is the key measure of inflation in India?
A. Consumer Price Index
B. Wholesale Price Index
C. Personal Consumption Expenditures Price Index
D. GDP Deflator
E. All of the above

Answer & Explanation

B. Wholesale Price Index


Explanation:
Inflation rates in India are usually quoted as changes in the Wholesale
Price Index, for all commodities. Many developing countries use
changes in the Consumer Price Index (CPI) as their central measure of
inflation

www.bankingpdf.com
370-380 Questions :

1. Poverty line is expressed in terms of an overall _______


A. Per capita Income
B. Per Capita consumption expenditure
C. Per Capita entertainment expenditure
D. Per Capita development expenditure
E. Per Capita GDP

Answer & Explanation

B. Per Capita consumption expenditure


Explanation:
In India consumption-expenditure has been made the basis for the
measurement of the minimum standard. The usual method is to fix a
poverty level. This level is expressed in terms of an overall per capita
consumption-expenditure.

2. The process by which the central bank of a country controls the


supply of money in the economy by exercising its control over
interest rates in order to maintain price stability and achieve
high economic growth is known as:
A. Economic Policy
B. Monetary Policy
C. Fiscal Policy
D. Credit Policy
E. Budgetary Policy

Answer & Explanation

B. Monetary Policy
Explanation:
Monetary Policy is the process by which monetary authority of a
country, generally a central bank controls the supply of money in the
economy by exercising its control over interest rates in order to

www.bankingpdf.com
maintain price stability and achieve high economic growth. In India,
the central monetary authority is the Reserve Bank of India (RBI). is so
designed as to maintain the price stability in the economy.

3. Loans granted by a bank to an exporter popularly known as


‘Export credit’ is guaranteed, in case of default,by which of the
following
A. EXIM Bank
B. Ministry of International Trade, GOA
C. ECGC
D. DICGC
E. Other than those given as options

Answer & Explanation

C. ECGC
Explanation:
ECGC Limited (Formerly Export Credit Guarantee Corporation of India
Ltd) is a company wholly owned by the Government of India based in
Mumbai, Maharashtra. It provides export credit insurance support to
Indian exporters and is controlled by the Ministry of Commerce.

4. The ratio of deposits to loans of a bank is known as _______


A. NPA coverage Ratio
B. Return on asset ratio
C. Asset Coverage ratio
D. CD Ratio
E. Other than those given as options

Answer & Explanation

E. Other than those given as options


Explanation:
The loan to deposit ratio is used to calculate a lending institution’s
ability to cover withdrawals made by its customers.

www.bankingpdf.com
5. In Bank’s balance sheet, which of the following is an asset?
A. Its paid up capital
B. Its saved deposits
C. Its instrument in government securities
D. Its accumulated Reserve Funds
E. Its Current Deposits

Answer & Explanation

C. Its instrument in government securities


Explanation:
Bank assets comprises cash, money at short notice, bills and securities
discounted, bank’s investments in government securities, loans
sanctioned by the bank.

6. Who amongst the following is the regulator in the financial


system of the country?
A. Other than those given as options
B. SEBI
C. CRISIL
D. TRAN
E. CERC

Answer & Explanation

B. SEBI
Explanation:
The financial system in India is regulated by independent regulators in
the field of banking, insurance, capital market, commodities market,
and pension funds. Example of Financial Regulators: RBI, IRDAI, SEBI,
PFRDA.

7. Which of the following is the negotiable instrument?


A. Fixed Deposit of a Bank
B. Share certificate issued by a PSU

www.bankingpdf.com
C. Demand Draft issued by a bank
D. Debenture of a company
E. Airway Receipt

Answer & Explanation

C. Demand Draft issued by a bank


Explanation:
A demand draft is a negotiable instrument similar to a bill of exchange.
A bank issues a demand draft to a client (drawer), directing another
bank (drawee) or one of its own branches to pay a certain sum to the
specified party (payee).

8. In International Banking terms,”Appreciation” of Rupee means


________
A. Excess of exports over imports
B. Purchasing power of rupee has come down
C. Availability of less foreign currency vis-a-vis rupee
D. Excess of imports over exports
E. Availability of more foreign currency vis-a-vis rupee

Answer & Explanation

C. Availability of less foreign currency vis-a-vis rupee


Explanation:
Currency Appreciation refers to increase in the value of domestic
currency in terms of foreign currency. The domestic currency becomes
more valuable and less of it is required to buy the foreign currency.

9. Bank Holidays are covered by which of the following?


A. As per the order of the GOI
B. As per the order of the IBA
C. Negotiable Instruments Act
D. RBI Act
E. Other than those given as options

www.bankingpdf.com
Answer & Explanation

C. Negotiable Instruments Act


Explanation:
Bank Holidays are declared by Central/State Governments/ Union
Territory under the Negotiable Instruments (NI) Act, 1881.

10. A non performing Asset in Banking Business means


________
A. A fixed asset of Bank is not been utilized
B. A portion of deposits not been utilized
C. A loan asset on which interest and/or instalments remain
unchanged.
D. All of the Above
E. None of the Above

Answer & Explanation

C. A loan asset on which interest and/or instalments remain


unchanged.
Explanation:
A nonperforming asset (NPA) refers to a classification for loans on the
books of financial institutions that are in default or are in arrears on
scheduled payments of principal or interest. In most cases, debt is
classified as nonperforming when loan payments have not been made
for a period of 90 days.

380-390 Questions :

1. Earlier in July, which of the following companies was upgraded


to a high safety grade by Crisil?
A) Manappuram Finance Limited
B) Mahindra Finance

www.bankingpdf.com
C) Muthoot Finance
D) Kosamattam
E) None of these

Answer & Explanation

C) Muthoot Finance
Explanation:
Ratings firm Crisil has upgraded Muthoot Finance’s long-term debt
rating to a high safety grade of AA/stable from AA-/stable earlier.
Under Crisil scale, AA rating will fall under high safety category and
carries very low credit risk.

2. Which of the following US-based life and health reinsurer has


received the approval regulator IRDAI to set up a branch in the
country?
A) Swiss Reinsurance America Corporation
B) Everest Reinsurance Company
C) Reinsurance Group of America
D) National Indemnity Company
E) None of these

Answer & Explanation

C) Reinsurance Group of America


Explanation:
US-based life and health reinsurer Reinsurance Group of America
(RGA) has received the first level approval regulator Irdai to set up a
branch in the country, a company official said today.
The Insurance Regulatory and Development Authority of India (Irdai)
has approved R-1 (requisition for registration) application of the
company

3. Which of the following insurance companies has launched a


series of Youtube videos for insurance reminders to its

www.bankingpdf.com
customers?
A) Coverfox Insurance Broking Pvt. Ltd.
B) Max Life Insurance Co. Ltd.
C) Shriram Life Insurance Co. Ltd.
D) IDBI Federal Life Insurance Co. Ltd.
E) None of these

Answer & Explanation

A) Coverfox Insurance Broking Pvt. Ltd.


Explanation:
Online insurance portal Coverfox.com on Tuesday launched a series of
small films on youtube to engage people and alert them to renew their
insurance.
The campaign was created in collaboration with BBH India a digital
innovation on Youtube.

4. Which of the following has/have received ‘certificate of


registration’ from insurance regulator IRDAI to commence
insurance business in the country?
A) Sahayog Microfinance Ltd
B) Aditya Birla group
C) Edelweiss Financial Services
D) Both A and B
E) Both B and C

Answer & Explanation

E) Both B and C
Explanation:
Aditya Birla Health Insurance Co today said it has received ‘certificate
of registration’ from regulator Irda, moving a step closer to
commencing business.
The company is a step down subsidiary of Aditya Birla Nuvo
Edelweiss Financial Services has also informed BSE that Irda has

www.bankingpdf.com
accepted the registration application form IRDA /R1, for setting up a
General Insurance Company in India, filed by Edelweiss General
Insurance Company, a wholly owned subsidiary.

5. Which of the following has approved $500 million loan for


bridge over Ganga River in Bihar?
A) World Bank
B) Asian Development Bank
C) International Monetary Fund
D) European Development Bank
E) None of these

Answer & Explanation

B) Asian Development Bank


Explanation:
Asian Development Bank accorded $500 million to build bridge over
India’s longest river bridge which is in Bihar having distance of 9.8 km
road.

6. In July, the UK government’s development finance unit, CDC


announced to infuse Rs 1,000 crore into its Finance unit in India
to boost people’s access to affordable housing. This finance unit
is
A) SKS Microfinance Ltd
B) India Infoline Finance
C) IDF Financial Services Pvt Ltd
D) YVU Financial Services Pvt Ltd
E) None of these

Answer & Explanation

B) India Infoline Finance


Explanation:
Diversified financial services company IIFL Holdings announced that UK

www.bankingpdf.com
government-owned CDC Group will invest about Rs 1,000 crore in its
unit India Infoline (IIFL) Finance.
CDC’s investment will help the group in expanding its financing
business and address the capital needs of under-served segments
through diversified offerings, IIFL Holdings said in a release.

7. Which of the following companies has set up two wholly owned


subsidiaries in July 2016, one for its broker business and
another for insurance business?
A) EXIM Bank
B) Religare Enterprises
C) Muthoot Finance
D) Sahara Asset Management Company
E) None of these

Answer & Explanation

B) Religare Enterprises
Explanation:
Religare Enterprises has set up two wholly owned subsidiaries –
Religare Broking Limited and Religare Insurance Ltd – as part of its
restructuring plan.

8. The merger process of State bank subsidiaries with State Bank


of India will be completed by
A) December 2017
B) March 2017
C) April 2017
D) July 2017
E) None of these

Answer & Explanation

C) April 2017
Explanation:

www.bankingpdf.com
Ihe country’s largest lender State Bank of India, which started the
process of merging all its subsidiaries with itself about three months
back, would become a single entity by April next, indicating the speed
at which the administration is moving to revitalise the banking system.

9. RBI has eased norms in cheque dishonour cases of Rs ______


and above.
A) 50 lakh
B) 1 crore
C) 1 lakh
D) 10 crore
E) 5 crore

Answer & Explanation

B) 1 crore
Explanation:
Relaxing chequebook norms, the Reserve Bank today left it to the
lender’s discretion on whether to issue fresh chequebooks or not in
cases of dishonour of Rs 1 crore and above.
As per the existing directive, banks are not allowed to issue fresh
chequebooks in the event of cheque dishonour valuing Rs 1 crore and
above on four occasions during a financial year for want of sufficient
funds.

10. As per RBI said in its statement, banks have to seek whose
permission before selling stake in stressed companies?
A) GOI
B) CVC
C) RBI
D) CBI
E) Both B and D

Answer & Explanation

www.bankingpdf.com
E) Both B and D
Explanation:
Following the central bank’s directive to banks to ensure that stake
sales in stressed companies under the strategic debt restructuring
(SDR) does not get to fronts of the original promoters, lenders are
initiating permissions from the Central Bureau of Investigation (CBI)
and the Central Vigilance Commission (CVC) before closing such deals,
people familiar with the development said.

390-400 Questions :

1. The structure of the registry which will enable quick sharing of


information about unstructured borrowers and help banks fight
bad loans. The registry is called _______
A. CIBIL
B. Fraudelent Borrowers List
C. RBI wilful Defaulter List
D. Central Fraud Registry
E. None of the Above

Answer & Explanation

D. Central Fraud Registry


Explanation:
The Reserve Bank of India has operationalised a Central Fraud Registry
(CFR). The registry will make available more information to banks at
the time of starting a banking relationship, extending credit facilities or
at any time during the operation of an account. The CFR is a
centralised searchable database which can be accessed by banks.

2. The record of all economic transactions between the residents


of a country and the rest of the world in a particular period is
known as _______

www.bankingpdf.com
A. Balance in Current Account
B. Balance in Capital Account
C. Trade Settlements
D. Balance of Trade
E. Balance of Payments

Answer & Explanation

E. Balance of Payments
Explanation:
The balance of payments (BOP) is the method countries use to monitor
all international monetary transactions at a specific period of time.A
record of all transactions made between one particular country and all
other countries during a specific time period.

3. Public Debt Management Agency (PDMA) is being to set up to


prevent leakages and development of the ___________
A. Treasury Market
B. Bond Market
C. Money Market
D. Capital Market
E. Call Money Market

Answer & Explanation

B. Bond Market
Explanation:
The government’s intention to set up a Public Debt Management
Agency (PDMA) is not only required for the development of the bond
market in the country, but it would also prevent leakages of public
funds.

4. The agency which is tasked with collecting analysing and


disseminating information related to dubious financial
transactions is ________

www.bankingpdf.com
A. Financial Intelligence Unit(FIU)
B. Financial Action Task Force(FATF)
C. Central Economic Intelligence Bureau(CEIB)
D. Central Intelligence Bureau(CIB)
E. Financial Stability Board(FSB)

Answer & Explanation

A. Financial Intelligence Unit(FIU)


Explanation:
The Financial Intelligence Unit (FIU), an agency tasked with collecting,
analysing and disseminating information related to dubious financial
transactions.The main function of FIU-IND is to receive cash/suspicious
transaction reports, analyse them and, as appropriate, disseminate
valuable financial information to intelligence/enforcement agencies and
regulatory authorities.

5. Who finalises the value and volume of the notes to be printed


every year?
A. Independently by Ministry of Finance
B. Independently by Ministry of Government of India
C. Committee of Bank Heads
D. With Mutual Consultation of the GOI and Bank Heads
E. With Mutual Consultation of the RBI and GOI

Answer & Explanation

E. With Mutual Consultation of the RBI and GOI


Explanation:
The Reserve Bank based on the demand requirement indicates the
volume and value of banknotes to be printed each year to the
Government of India which get finalized after mutual consultation. The
quantum of banknotes to be printed, broadly depends on the
requirement for meeting the demand for banknotes, GDP growth,
replacement of soiled banknotes, reserve stock requirements, etc.

www.bankingpdf.com
6. e-KYC is possible only for those who have _______
A. PAN Card
B. Driving License
C. Ration Card
D. Aadhaar Number
E. None of the Above

Answer & Explanation

D. Aadhaar Number
Explanation:
e-KYC is possible only for those who have Aadhaar numbers. While
using e-KYC service, you have to authorise the Unique Identification
Authority of India (UIDAI), by explicit consent, to release your
identity/address through biometric authentication to the bank
branches/business correspondent (BC). The UIDAI then transfers your
data comprising name, age, gender, and photograph of the individual,
electronically to the bank/BC.

7. An open source platform for mobile-to-mobile fund transfers


without account details launched by NPCI was ________
A. Unified Payment Interface
B. Union of Payment Interface
C. Payment Interface
D. Unique Payment Interface
E. People’s Payment Interface

Answer & Explanation

A. Unified Payment Interface


Explanation:
UPI allows you to pay directly from your bank account to different
merchants without the hassle of typing your card details, or
netbanking/wallet password.

www.bankingpdf.com
8. The practice of lending money to unrelated individuals, or
“peers”, without going through a traditional financial
intermediary such as a bank or other traditional financial
institution is called__________
A. B2B transactions
B. C2C transactions
C. P2P lending
D. PPP Model
E. P2P Investing

Answer & Explanation

C. P2P lending
Explanation:
P2P lending, is the practice of lending money to individuals or
businesses through online services that match lenders directly with
borrowers.

9. Banks can not generally accept interest-free deposits. which of


the following is an exception to this rule?
A. Current Account
B. Savings Account of Minors
C. Deposit from Minority groups
D. BSBDA
E. None of the Above

Answer & Explanation

A. Current Account
Explanation:
Banks cannot accept interest free deposits other than in current
account.

10. Deposits under Foreign Currency Non-Resident(FCNR)


scheme can be accepted for a minimum of________

www.bankingpdf.com
A. Six months
B. Three months
C. One year
D. 15 days
E. None of the Above

Answer & Explanation

C. One year
Explanation:
The deposits should be accepted under the Foreign Currency Non-
Resident(FCNR) Scheme for the following maturity periods:
One year and above but less than two years
Two years and above but less than three years
Three years and above but less than four years
Four years and above but less than five years
Five years only

400-410 Questions :

1. Balance in a current account is classified as ________


A. Hybrid Deposit
B. Term Deposit
C. Demand Deposit
D. Flexi Deposit
E. other than those given as options

Answer & Explanation

C. Demand Deposit
Explanation:
Current accounts form a large portion of demand deposits of a bank. It
can be opened by Individuals, Business entities (firms, company),

www.bankingpdf.com
Institutions, Government bodies / departments, Societies, Liquidators,
Receivers, and Trusts.

2. The Foreign exchange of India is kept with _________


A. SBI
B. ECGC
C. RBI
D. NABARD
E. other than those given as options

Answer & Explanation

C. RBI
Explanation:
Reserve Bank of India accumulates foreign currency reserves by
purchasing from authorized dealers in open market operations. Foreign
exchange reserves of India act as a cushion against rupee volatility
once global interest rates starts rising. The Foreign exchange reserves
of India consists of below four categories. (a) Foreign Currency Assets
(b) Gold (c) SDRs (d) Reserve Tranche Position in the IMF.

3. Fiscal policy is concerned with which of the following?


A. Public revenue and Expenditure
B. Issue of Currency
C. Export Import
D. Population Control
E. Education for all

Answer & Explanation

A. Public revenue and Expenditure


Explanation:
Fiscal policy is the policy relating to government revenues from taxes
and expenditure on various projects. Monetary Policy, on the other
hand, is mainly concerned with the flow of money in the economy.

www.bankingpdf.com
4. In which of the following types of banking, there is a direct
execution of transaction between a bank and its consumers?
A. Retail Banking
B. Universal Banking
C. Virtual Banking
D. Unit Banking
E. other than those given as options

Answer & Explanation

A. Retail Banking
Explanation:
Retail banking also known as Consumer Banking is the provision of
services by a bank to individual consumers, rather than to companies,
corporations or other banks. Services offered include savings and
transactional accounts, mortgages, personal loans, debit cards, and
credit cards.

5. Maximum loan amount to individuals against security shares


can be _____
A. 20 lakh
B. 15 lakh
C. 25 lakh
D. 50 lakh
E. other than those given as options

Answer & Explanation

A. 20 lakh
Explanation:
Loans/ advances granted to individuals against the security of shares,
debentures and PSU bonds should not exceed Rs.10 lakh and Rs.20
lakh, if the securities are held in physical form and dematerialized form
respectively.

www.bankingpdf.com
6. Deposits under Foreign Currency Non-Resident(FCNR) scheme
can be accepted for a minimum of _______
A. 15 days
B. 3 months
C. 6 months
D. 1 year
E. 7 days

Answer & Explanation

D. 1 year
Explanation:
Term Deposit with maturity of minimum 1 year & maximum 5 years
can be opened under Foreign Currency Non-Resident(FCNR) scheme.

7. In case of failed ATM transactions, if the amount is not credited


to customer’s account within 7 working days from the date of
receipt of the complaint. Banks have to pay compensation at
the rate of Rs.________ per day.
A. 150
B. 200
C. 100
D. 10
E. 50

Answer & Explanation

C. 100
Explanation:
Reconciliation of failed transactions at ATMs – For any failure to re-
credit the customer’s account within 7 working days from the date of
receipt of the complaint, the bank will pay compensation of Rs.100/-,
per day, to the aggrieved customer.

www.bankingpdf.com
8. Which of the following forms of securing a loan is the safest
one?
A. Lien
B. Assignment
C. Pledge
D. Hypothecation
E. Mortgage

Answer & Explanation

C. Pledge
Explanation:
A pledged asset is collateral pledged by a borrower to a lender (usually
in return for a loan). The lender has the right to seize the collateral if
the borrower defaults on the obligation.

9. Which of the following is known as “Demat” account?


A. Account in which shares are held in electronic form
B. other than those given as options
C. Account allowed to be operated by guardian of minor
D. Account operated by business correspondents in rural centres.
E. Account opened with zero balance

Answer & Explanation

A. Account in which shares are held in electronic form


Explanation:
A Dematerialized account is opened by the investor while registering
with an investment broker (or sub-broker).

10. The Aadhaar-Enabled Payment Systems(AEPS) is a bank led


model that facilitates banking facilities by allowing transactions
at Point of Sale through the Business Correspondent (BC) using
the Aadhaar authentication number. Aadhaar enabled basic
types of banking do not include

www.bankingpdf.com
A. Balance Enquiry
B. Cash Withdrawal
C. Online Payment
D. Cash Deposit
E. Aadhaar to Aadhaar funds transfer

Answer & Explanation

C. Online Payment
Explanation:
Services Offered by AEPS:
Balance Enquiry
Cash Withdrawal
Cash Deposit
Aadhaar to Aadhaar Fund Transfer
Gateway Authentication Services

410-420 Questions :

1. Which of the following private banks has signed an agreement


with Reliance Jio Money for One-Click payment service ?
A) Laxmi Vilas Bank
B) Federal Bank
C) South Indian Bank
D) Karnataka Bank
E) Karur Vysya Bank

Answer & Explanation

B) Federal bank
Explanation:
Federal bank has signed an agreement with Reliance Jio Money for
One-Click payment service. Reliance Jio money is a wallet application

www.bankingpdf.com
that will be launched soon for mobile devices where Federal bank will
enable a direct payment option for its customers.

2. Which of the following General Insurance Company has entered


into agreement with Shamrao Vithal Co-operative (SVC) Bank
for bancassurance business?
A) Reliance General Insurance Co. Ltd.
B) L&T General Insurance Co. Ltd.
C) HDFC ERGO General Insurance Co. Ltd.
D) Future Generali India Insurance Co. Ltd.
E) Tata AIG General Insurance Co. Ltd.

Answer & Explanation

C) HDFC ERGO General Insurance Co. Ltd.


Explanation:
HDFC ERGO General Insurance Company and Shamrao Vithal Co-
operative (SVC) Bank announced their bancassurance agency
partnership to provide range of non-life insurance offerings to the
customers of the bank.
The partnership combines HDFC ERGO’s experience in introducing
innovative non-life insurance products with SVC Bank’s customer base
across various states.

3. The Reserve Bank of India imposed how much penalty on Bank


of Baroda in the alleged Rs 6000 crore forex related
irregularities that came to light in October last year?
A) Rs 5 crore
B) Rs 7 crore
C) Rs 5.5 crore
D) Rs 6.2 crore
E) Rs 6 crore

Answer & Explanation

www.bankingpdf.com
A) Rs 5 crore
Explanation:
The RBI carried out an inspection after Rs 6,100 crore import
remittances were effected by Bank of Baroda’s Ashok Vihar branch in
New Delhi.

4. Which of the following banks have become the first banks to


join SWIFT’s global payments initiative?
A) ICICI and HDFC Bank
B) IndusInd Bank and SBI
C) PNB and Federal Bank
D) Axis Bank and ICICI
E) ICICI and SBI

Answer & Explanation

D) Axis Bank and ICICI


Explanation:
Global payments leader SWIFT said that ICICI Bank and Axis Bank
have become the first domestic lenders to sign up for its global
payments innovation initiative, which already has over 70 other leading
banks globally.
The initiative aims to enhance cross-border payments by leveraging
Swift’s messaging platform and global reach and in the first phase, the
initiative will focus on business-to-business payments.

5. RBI has imposed how much fine on 13 banks for violation of


FEMA and lapses in know your client (KYC) rules?
A) Rs 24 crore
B) Rs 25 crore
C) Rs 27 crore
D) Rs 28 crore
E) Rs 29 crore

Answer & Explanation

www.bankingpdf.com
C) Rs 27 crore
Explanation:
The Reserve Bank of India has come down hard on 13 banks and fined
them for Rs 27 crore for violation of FEMA and lapses in know your
client (KYC) rules.

6. RBI has imposed a fine of Rs 1 crore on which of the following


banks for flouting bill discounting norms?
A) Syndicate Bank
B) United Bank of India
C) Indian Overseas Bank
D) UCO Bank
E) Vijaya Bank

Answer & Explanation

D) UCO Bank
Explanation:
Reserve Bank of India has told state-run Uco Bank to pay Rs 1 crore
penalty for flouting bill discounting norms which allowed current
account holders siphon out funds.

7. A payments company OKI installed 600 automatic teller


machines with cash recycling abilities for State Bank of India.
OKI has its headquarters in
A) England
B) Singapore
C) USA
D) Malaysia
E) Japan

Answer & Explanation

E) Japan
Explanation:

www.bankingpdf.com
The company which is headquartered in Japan and operates out of
Mumbai in India manufactures ATMs which work with recyclable cash
and has both cash acceptance mechanism along with cash dispensing
in their teller machines.
Indian banks which mostly deploy standalone ATMs have very recently
taken to the new technology of machines with both depositing as well
as dispensing abilities.

8. Recently, Yeldi Softcom received a licence for RBI to set up


which of the following?
A) open wallet
B) semi-closed wallet
C) closed wallet
D) semi-open wallet
E) None of these

Answer & Explanation

B) semi-closed wallet
Explanation:
Yeldi Softcom received its semi-closed wallet license on Wednesday
from the Reserve Bank of India. This opens up a new league of
possibilities in terms of the products that could be launched by Yeldi
Softcom to bring in further avenues to carry out cashless transactions.
Semi-closed wallet is a payment instrument that is accepted by a
predefined set of merchants that have been contracted specifically by
the issuer of the semi-closed wallet.

9. What is the initial minimum paid-up equity capital required for


new on-tap license to set up private bank as announced by
RBI?
A) Rs 5 crore
B) Rs 100 crore
C) Rs 500 crore

www.bankingpdf.com
D) Rs 50 crore
E) Rs 200 crore

Answer & Explanation

C) Rs 500 crore
Explanation:
The Reserve Bank of India released norms for on-tap permits for
private sector banks and said the validity of the in-principle approval
issued by the RBI will be 18 months from the date of granting in-
principle approval.
1. The initial minimum paid-up voting equity capital for a bank shall be
500 crore rupees. Thereafter, the bank shall have a minimum net
worth of 500 crore rupees at all times.
2. Resident individuals and professionals having 10 years of experience
in banking and finance at a senior level are also eligible to promote
universal banks.

10. Which of the following banks has teamed up with Apollo to


set up asset reconstruction company (ARC)?
A) SBI
B) Punjab National Bank
C) Axis Bank
D) ICICI Bank
E) Both A and D

Answer & Explanation

D) ICICI Bank
Explanation:
With the banking sector buffeted by bad loans, the country’s largest
private lender ICICI Bank teamed up with private equity major Apollo
Global Management to launch an asset reconstruction company.
ICICI and Apollo have signed a memorandum of understanding to set

www.bankingpdf.com
up the ARC, which will acquire debt from lenders and also take equity
stakes in borrowers, a bank statement read.

420-430 Questions :

1. The Headquarters of National Payments Corporation of


India(NPCI) is ________
A. New Delhi
B. Hyderabad
C. Mumbai
D. Chennai

Answer & Explanation

C. Mumbai
Explanation:
National Payments Corporation of India (NPCI), Mumbai is an umbrella
organization for all retail payments system in India. It was set up with
the guidance and support of the Reserve Bank of India (RBI) and
Indian Banks’ Association (IBA). The RBI, after setting up the Board for
Regulation and Supervision of Payment and Settlement Systems
(BPSS) in 2005, released a vision document incorporating a proposal to
set up an umbrella institution for all the RETAIL PAYMENT SYSTEMS in
the country. The core objective was to consolidate and integrate the
multiple systems with varying service levels into nation-wide uniform
and standard business process for all retail payment systems.

2. Micro Units Development Refinance Agency (MUDRA) has a


corpus of _______, and credit guarantee corpus of ________
A. Rs 25,000 crore and Rs.3,000 crore
B. Rs 20,000 crore and Rs.4,000 crore
C. Rs 15,000 crore and Rs.4,000 crore
D. Rs 20,000 crore and Rs.3,000 crore

www.bankingpdf.com
Answer & Explanation

D. Rs 20,000 crore and Rs.3,000 crore


Explanation:
The bank will have an initial corpus of Rs 20,000 crore and a credit
guarantee fund of and Rs.3,000 crore. The bank will initially function as
a non-banking financial company and a subsidiary of the Small
Industries Development Bank of India (SIDBI).

3. Authorised capital of National Payments Corporation of India


(NPCI) is _________ and paid up capital is _______
A. Rs 300 crore and Rs 100 crore
B. Rs 350 crore and Rs 50 crore
C. Rs 200 crore and Rs 100 crore
D. Rs 100 crore and Rs 50 crore

Answer & Explanation

A. Rs 300 crore and Rs 100 crore


Explanation:
National Payments Corporation of India (NPCI) was incorporated in
December 2008 and the Certificate of Commencement of Business was
issued in April 2009. It has been incorporated as a Section 8 company
under Companies Act, 2013 and is aimed to operate for the benefit of
all the member banks and their customers. The authorised capital has
been pegged at Rs 300 crore and paid up capital is Rs 100 crore so
that the company can create infrastructure of large dimension and
operate on high volume resulting payment services at fraction of the
present cost structure.

4. NABARD was set up with an initial capital of __________


A. 100 crore
B. 200 crore
C. 300 crore
D. 500 crore

www.bankingpdf.com
Answer & Explanation

A. 100 crore
Explanation:
NABARD was set up with an initial capital of 100 crore. Currently the
share capital between Government of India and RBI, the paid up
capital stood at 5000 crore with Government of India holding 4,980
crore (99.60%) and Reserve Bank of India 20.00 crore (0.40%).

5. Minimum amount that can be invested in Kisan Vikas


Patra(KVP) Certificate is _________
A. Rs.100
B. Rs.200
C. Rs.500
D. Rs.1000

Answer & Explanation

D. Rs.1000
Explanation:
KVP certificates are available in the denominations of Rs 1000, Rs
5000, Rs 10000 and Rs 50000. The minimum amount that can be
invested is Rs 1000. However, there is no upper limit on the purchase
of KVPs.

6. The amount invested in Kisan Vikas Patra(KVP) would get


doubled in ________ months.
A. 100
B. 105
C. 110
D. 115

Answer & Explanation

www.bankingpdf.com
C. 110
Explanation:
The amount invested in Kisan Vikas Patra would get doubled in 110
months or nine years and two months. The interest rate of KVP is 7.8
per cent annually.

7. The 5-year National saving certificate has the interest rate of


_____________
A. 7.1%
B. 7.2%
C. 8.1%
D. 8.2%

Answer & Explanation

C. 8.1%

Explanation:
National Savings Certificates, popularly known as NSC, is an Indian
Government Savings Bond, primarily used for small savings and
income tax saving investments in India. These can be purchased from
any Post Office in India by an adult (either in his/her own name or on
behalf of a minor), a minor, a trust, and two adults jointly. These are
issued for five and ten year maturity and can be pledged to banks as
collateral for availing loans. Currently the 5-year National saving
certificate has the interest rate of 8.1%.

8. In National Savings Certificates, investment up to ________


per annum qualifies for IT Rebate under section 80C of Income
Tax Act.
A. Rs.10 lakh
B. Rs.2 lakh
C. Rs.5 lakh
D. Rs.1 lakh

www.bankingpdf.com
Answer & Explanation

D. Rs.1 lakh
Explanation:
There is no maximum limit for investment. Certificates can be kept as
collateral security to get loan from banks. Investment up to INR
1,00,000/- per annum qualifies for IT Rebate under section 80C of
Income Tax Act. Trust and HUF cannot invest. Rate of interest effective
from 1st April 2016 is 8.10%

9. A minimum yearly deposit of ________ is required to open and


maintain a PPF account, and a maximum deposit of _____.
A. Rs. 500 and Rs 1.50 lakhs
B. Rs. 1000 and Rs 2.50 lakhs
C. Rs. 2000 and Rs 3.50 lakhs
D. Rs. 5000 and Rs 4.50 lakhs

Answer & Explanation

A. Rs. 500 and Rs 1.50 lakhs


Explanation:
A minimum yearly deposit of Rs. 500 is required to open and maintain
a PPF account, and a maximum deposit of Rs.1.5 lakhs (as on Financial
year 2015-2016) can be made in a PPF account in any given financial
year. The subscriber should not deposit more than Rs.1.50 lac per
annum as the excess amount will neither earn any interest nor will be
eligible for rebate under Income Tax Act. The amount can be deposited
in lump sum or in a maximum of 12 instalments per year.

10. The Public Provident Fund(PPF) has the interest rate of


_____________
A. 7.1%
B. 7.2%
C. 8.1%
D. 8.2%

www.bankingpdf.com
Answer & Explanation

C. 8.1%
Explanation:
The Public Provident Fund is a savings-cum-tax-saving instrument in
India, introduced by the National Savings Institute of the Ministry of
Finance in 1968.The aim of the scheme is to mobilize small savings by
offering an investment with reasonable returns combined with income
tax benefits. The current interest rate effective from 1 April 2016 is
8.1% Per Annum’ (compounded annually).

430-440 Questions :

1. The China-led Asian Infrastructure Investment Bank (AIIB)


approved its first set of loans amounting to $ 509 million to
four countries. Which of the following Is not among these 4
countries?
A) Bangladesh
B) Indonesia
C) Pakistan
D) Nepal
E) Tajikistan

Answer & Explanation

D) Nepal
Explanation:
AIIB headquartered in Beijing, China, has given loan to finance
projects in four countries viz. Bangladesh, Pakistan, Indonesia and
Tajikistan.

2. Which of the following has exited the shareholding of CIBIL?


A) ICICI and Bank of India

www.bankingpdf.com
B) United Bank of India and Aditya Birla Trustee Company Private Ltd.
C) United Bank of India and Bank of Baroda
D) ICICI and Bank of Baroda
E) Indian Overseas Bank and ICICI

Answer & Explanation

D) ICICI and Bank of Baroda


Explanation:
ICICI Bank has sold its whole six per cent stake in the company, while
Bank of Baroda has sold its whole five per cent holding.

3. With ICICI and BoB sold there shareholding in CIBIL, who has
bought their percentage of shares?
A) Aditya Birla Trustee Company Private Ltd.
B) Indian Overseas Bank
C) TransUnion International Inc
D) Union Bank of India
E) India Infoline Finance Limited

Answer & Explanation

C) TransUnion International Inc


Explanation:
Earlier the shareholding patterns of CIBIL were
TransUnion International Inc (66.1%)
ICICI Bank Ltd (6%)
Bank of Baroda (5%)
Bank of India (5%)
Union Bank of India (5%)
Indian Overseas Bank (5%)
Aditya Birla Trustee Company Private Ltd. (4%)
India Alternatives Private Equity Fund (2.9%)
India Infoline Finance Limited (1%)

www.bankingpdf.com
4. Which of the following bansk has launched ‘Elite’ card on the
Master Card World platform which is enabled with NFC (Near
Field Communication) technology?
A) ICICI
B) PNB
C) SBI
D) Axis Bank
E) Canara Bank

Answer & Explanation

C) SBI
Explanation:
This allows customers a contactless payment experience, adding more
speed and suave to their wallets.
The new card comes with a joining fee of Rs 4,999 and all customers
will receive a welcome gift worth Rs 5,000 in the form of vouchers from
popular lifestyle and travel brands.

5. Cabinet Committee on Economic Affairs (CCEA) cleared the


proposal of Axis Bank for increasing foreign shareholding limit
from existing 62 per cent to ____ per cent.
A) 70%
B) 84%
C) 74%
D) 65%
E) None of these

Answer & Explanation

C) 74%
Explanation:
Cabinet Committee on Economic Affairs (CCEA) headed by Prime
Minister Narendra Modi cleared the proposal of Axis Bank for increasing
foreign shareholding limit from existing 62 per cent to 74 per cent.

www.bankingpdf.com
6. According to rating agency Fitch, Indian banks will need
approximately how much capital by 2019 to meet new Basel III
capital norms?
A) $100 billion
B) $90 billion
C) $85 billion
D) $70 billion
E) $95 billion

Answer & Explanation

B) $90 billion
Explanation:
According to rating agency Fitch, Indian banks will need about $90
billion of capital to meet new Basel III capital norms to be implemented
by the financial year ending March 2019, more than 80% of which will
be needed by public sector lenders.

7. Who has been named as the best banking CEO in Asia?


A) Shikha Sharma
B) Uday Kotak
C) Chanda Kochar
D) Aditya Puri
E) Arundhati Bhattacharya

Answer & Explanation

D) Aditya Puri
Explanation:
Aditya Puri, the chief executive and managing director of the second
largest private sector lender HDFC Bank has been ranked as the best
banking CEO in Asia by the investor community.

8. Sudarshan Sen has been appointed as the Executive Director of


RBI. He replaced

www.bankingpdf.com
A) NS Vishwanathan
B) KR Subramaniam
C) Sarvesh Rana
D) NS Iyer
E) None of these

Answer & Explanation

A) NS Vishwanathan
Explanation:
The Union Government has appointed NS Vishwanathan as deputy
governor of the Reserve Bank of India (RBI). His appointment was
approved by the Appointments Committee of the Cabinet (ACC)
presided by Prime Minister Narendra Modi in New Delhi.
In his place, Sudarshan Sen has been appointed.

9. Who has been appointed as the head of Working Group on Fin


Tech and Digital Banking by RBI?
A) NS Vishwanathan
B) KR Subramaniam
C) Sarvesh Rana
D) NS Iyer
E) Sudarshan Sen

Answer & Explanation

E) Sudarshan Sen
Explanation:
The Reserve Bank of India (RBI) has set up an inter-regulatory
Working Group to study the regulatory issues relating to Financial
Technology (Fintech) and Digital Banking in India The inter-regulatory
Working Group will be headed by RBI’s Executive Director Sudarshan
Sen and consist of 12 members.

www.bankingpdf.com
10. Which of the following banks is ready to acquire
Microfinance Grama Vidiyal?
A) Bandhan Bank
B) Yes Bank
C) Axis Bank
D) Kotak Mahindra Bank
E) IDFC Bank

Answer & Explanation

E) IDFC Bank
Explanation:
Private sector lender IDFC Bank announced the acquisition of Tamil
Nadu-based Grama Vidiyal Microfinance that will help it widen the
reach.

440-450 Questions :

1. ECGC Limited (Export Credit Guarantee Corporation of India


Ltd) is a company wholly owned by?
A. RBI
B. Government Of India
C. IRDAI
D. NABARD

Answer & Explanation

B. Government Of India
Explanation:
ECGC Ltd. (Export Credit Guarantee Corporation of India Ltd.), wholly
owned by Government of India, was set up in 1957 with the objective
of promoting exports from the country by providing Credit Risk
Insurance and related services for exports. It functions under the

www.bankingpdf.com
administrative control of Ministry of Commerce & Industry, and is
managed by a Board of Directors comprising representatives of the
Government, Reserve Bank of India, banking, and insurance and
exporting community.

2. National Housing Bank (NHB), a wholly owned subsidiary of


_________
A. RBI
B. Government Of India
C. IRDAI
D. State Government

Answer & Explanation

A. RBI
Explanation:
National Housing Bank (NHB) is an apex level institution for housing.
NHB was set up on July 9, 1988 under the National Housing Bank Act,
1987. NHB is wholly owned by Reserve Bank of India, which
contributed the entire paid-up capital.

3. As per the Exim Bank Act, at a particular point in time, the Exim
Bank can have a maximum of _________ directors on its Board.
A. 16
B. 15
C. 14
D. 13

Answer & Explanation

A. 16
Explanation:
As per the Exim Bank Act, at a particular point in time, the Bank can
have a maximum of 16 directors on its Board.
Including Chairman and Managing Director, the Bank’s Board

www.bankingpdf.com
constitutes of 13 directors who are appointed by the GOI, they are five
top level Government of India functionaries, three directors from
scheduled commercial banks and four directors who are industry/trade
experts. Three other directors are nominated by the Reserve Bank of
India (RBI), Industrial Development Bank of India (IDBI) and ECGC Ltd
respectively.

4. Which of the following holds majority stake in NABARD?


A. RBI
B. Government Of India
C. IRDAI
D. None of the Above

Answer & Explanation

B. Government Of India
Explanation:
National Bank for Agriculture and Rural Development (NABARD) is an
apex development bank in India, having headquarters in Mumbai,
Maharashtra. RBI sold its stake in NABARD to the Government of India,
which now holds 99% stake.

5. Currently Small Industries Development Bank of India(SIDBI)


is a wholly owned subsidiary of _______
A. RBI
B. IDBI
C. 34 Government of India owned / controlled institutions
D. None of the Above

Answer & Explanation

C. 34 Government of India owned / controlled institutions


Explanation:
Small Industries Development Bank of India(SIDBI) was incorporated
initially as a wholly owned subsidiary of Industrial Development Bank

www.bankingpdf.com
of India. Currently the ownership is held by 34 Government of India
owned / controlled institutions.

6. Which of the following is/are an entity founded by SIDBI?


A. Credit Guarantee Fund Trust for Micro and Small Enterprises
B. SIDBI Venture Capital Ltd
C. ISARC
D. All of the Above

Answer & Explanation

D. All of the Above


Explanation:
Credit Guarantee Fund Trust for Micro and Small Enterprises – provides
collateral-free loans extended to SME.
SIDBI Venture Capital Ltd – a venture capital company focussed at
SME.
SME Rating Agency of India Ltd – provides composite ratings to SME.
ISARC – India SME Asset Reconstruction Company.

7. Micro Units Development and Refinance Agency(MUDRA) Bank


has been headquartered in _______
A. Mumbai
B. Bangalore
C. New Delhi
D. Hyderabad

Answer & Explanation

C. New Delhi
Explanation:
Micro Units Development and Refinance Agency Bank (or MUDRA Bank)
is a public sector financial institution has been headquartered in New
Delhi. It provides loans at low rates to micro-finance institutions and

www.bankingpdf.com
non-banking financial institutions which then provide credit to MSMEs.
It was launched by Prime Minister Narendra Modi.

8. Deposit Insurance and Credit Guarantee Corporation ( DICGC)


is a subsidiary of ________
A. RBI
B. IDBI
C. 34 Government of India owned / controlled institutions
D. None of the Above

Answer & Explanation

A. RBI
Explanation:
Deposit Insurance and Credit Guarantee Corporation ( DICGC) is a
subsidiary of Reserve Bank of India.
It was established under Deposit Insurance and Credit Guarantee
Corporation Act, 1961 for the purpose of providing insurance of
deposits and guaranteeing of credit facilities. DICGC insures all bank
deposits, such as saving, fixed, current, recurring deposits for up to
the limit of Rs. 100,000 of each deposits in a bank.

9. The Regional Rural Banks were owned by _________


A. Central Government
B. State Government
C. Sponsor Banks
D. All of the Above

Answer & Explanation

D. All of the Above


Explanation:
The Regional Rural Banks were owned by the Central Government, the
State Government and the Sponsor Bank.
Central Government-50%

www.bankingpdf.com
State Government- 15%
Sponsor Banks- 35%.

10. Which of the following holds majority stake in RRBs?


A. Central Government
B. State Government
C. Sponsor Banks
D. All of the Above

Answer & Explanation

A. Central Government
Explanation:
The Regional Rural Banks were owned by the Central Government, the
State Government and the Sponsor Bank.
Central Government-50%
State Government- 15%
Sponsor Banks- 35%.

450-460 Questions :

1. The Headquarters of Capital Local Area Bank is situated in


_______
A. Amritstar, Punjab
B. Ludhiana, Punjab
C. Bathinda, Punjab
D. Jalandhar, Punjab

Answer & Explanation

D. Jalandhar, Punjab
Explanation:
Currently, Capital Area Local Bank is operating in Punjab’s five districts

www.bankingpdf.com
– Jalandhar, Kapurthala, Hoshairpur, Ludhiana and Amritsar.Capital
Local Area Bank has emerged as the first bank to have received licence
from the Reserve Bank of India to start small finance bank. The bank
plans to start operations as a small finance bank from the next
financial year as it eyes a total business size of Rs 6,700 crore with 110
branches by 2020.

2. Which of the following is the Headquarters of City Union Bank?


A. Kanchipuram
B. Cuddalore
C. Dindigul
D. Kumbakonam

Answer & Explanation

D. Kumbakonam
Explanation:
City Union Bank (CUB) India is one of the private banks in India. It was
previously known as Kumbakonam Bank Limited
CEO: Dr. N. Kamakodi
Founded: 1904

3. The Headquarters of Kotak Mahindra Bank is located in


_______
A. Mumbai
B. Hyderabad
C. Chennai
D. Bangalore

Answer & Explanation

A. Mumbai
Explanation:
Kotak Mahindra Bank is an Indian private sector banking
headquartered in Mumbai, Maharashtra. Kotak Mahindra Bank

www.bankingpdf.com
Launches International Banking Unit in GIFT City recently. Kotak
Mahindra Bank CEO Uday Kotak, who has a net worth of about $7.1
billion, is the sole Indian in Forbes’ list of 40 of the most powerful
people in the financial world who control the flow of trillions of dollars
around the global economy.

4. RBS Bank has been headquartered in __________


A. Glasgow, United Kingdom
B. Edinburgh, United Kingdom
C. Skye, United Kingdom
D. Aberdeen, United Kingdom

Answer & Explanation

B. Edinburgh, United Kingdom


Explanation:
The Royal Bank of Scotland Group plc is a British banking and
insurance holding company, based in Edinburgh, Scotland. Royal Bank
of Scotland will close down it’s retail operations in India which will
resulting in shutting down of 10 branches.

5. Which of the following is the Headquarters of Axis Bank?


A. Mumbai
B. Hyderabad
C. Chennai
D. Bangalore

Answer & Explanation

A. Mumbai
Explanation:
Axis Bank Limited is the third largest private sector bank in India, has
been headquartered in Mumbai, Maharashtra.
Recent Activities of Axis Bank:
India’s largest insurer Life Insurance Corporation of India

www.bankingpdf.com
(LIC) has signed an agreement with private sector lender Axis
Bank as its bancassurance partner wherein the bank will distribute
LIC’s products to its customers.
Global payments leader SWIFT announced ICICI Bank and Axis Bank
have become the first domestic lenders to sign up for its global
payments innovation initiative, which already has over 70 other leading
banks globally.
Axis Bank has signed an agreement with Thailand’s Kasikorn
Bank to enhance cooperation in trade and investment.

6. DBS Bank has been headquartered in __________


A. United Kingdom
B. Spain
C. Singapore
D. South Africa

Answer & Explanation

C. Singapore
Explanation:
DBS Bank Ltd is a Singaporean multinational banking and financial
services company. The company was known as The Development Bank
of Singapore Limited. DBS Bank plans to set up new technology hub in
Hyderabad.

7. Which of the following is the Headquarters of Yes Bank?


A. Noida
B. Hyderabad
C. Chennai
D. Mumbai

Answer & Explanation

D. Mumbai
Explanation:

www.bankingpdf.com
Yes Bank is India’s fifth largest private sector Bank, co-founded by
Rana Kapoor in 2004. It has been headquartered in Mumbai. Yes Bank
is the only Greenfield Bank licence awarded by the RBI in the last two
decades.
YES Bank gets SEBI nod to provide custodian services.
SIDBI entered into a partnership with private sector lender Yes Bank to
guarantee 75 per cent of loans of up to Rs 15 crore extended to energy
efficiency projects under a World Bank initiative.

8. The Headquarters of Islamic Development Bank is _____


A. Sohar, Oman
B. Kuala Lumpur, Malaysia
C. Jeddah, Saudi Arabia
D. Baghdad, Iraq

Answer & Explanation

C. Jeddah, Saudi Arabia


Explanation:
Motto- Together We Build A Better Future
Headquarters – Jeddah, Saudi Arabia
Membership – 56 countries
President – Ahmad Mohamed Ali Al-Madani
The Islamic Development Bank (IDB) will open its first branch in India
at Ahmedabad, Gujarat.

9. Which of the following is the Headquarters of IndusInd Bank?


A. Noida
B. Hyderabad
C. Chennai
D. Mumbai

Answer & Explanation

www.bankingpdf.com
D. Mumbai
Explanation:
IndusInd Bank Limited is a Mumbai based Indian new generation
bank,established in 1994. The bank offers commercial, transactional
and electronic banking products and services. IndusInd Bank was
inaugurated in April 1994 by then Union Finance Minister Manmohan
Singh. Indusind Bank is the first among the new-generation
private banks in India.
IndusInd Bank one of the Indian firms that grab top spot for
transparency in survey by Institutional Investor.
ndusInd Bank opened its IFSC Banking Unit (IBU) at the Gujarat
International Finance Tec-City (GIFT City) to meet the requirements of
offshore banking operations from India.

10. Caixa Bank has been headquartered in __________


A. United Kingdom
B. Spain
C. Singapore
D. South Africa

Answer & Explanation

B. Spain
Explanation:
Caixa Bank, formerly Criteria CaixaCorp, is a Spanish financial services
company headquartered in Barcelona.
SBI and Spain’s CaixaBank have inked pact to provide loans to Indo-
Spanish joint ventures and local enterprises recently

www.bankingpdf.com
460-470 Questions :

1. Which of the following company has changed its name to


Bharat Financial Inclusion Ltd.?
A) SKS Microfinance Ltd
B) Spandana Sphoorty Financial Ltd
C) Bhartiya Samruddhi Finance Limited
D) Cashpor Micro Credit
E) None of these

Answer & Explanation

A) SKS Microfinance
Explanation:
The company had said the decision to change the name was taken as
its core had undergone a transformation, equipping it to play a major
role in fulfilling the national priority of financial inclusion.
The company is among the largest microfinance companies in India. It
has presence across 18 states covering 1,00,000 villages, catering to
its 63.65 lakh women members.

2. How much shareholding is must for the promoters of insurance


companies after their listing?
A) 40%
B) 50%
C) 49%
D) 52%
E) None of these

Answer & Explanation

B) 50%
Explanation:
Promoters of insurance companies will be forced to maintain their
shareholding at 50 per cent even after listing under the guidelines

www.bankingpdf.com
proposed by the insurance regulator.
This is in sharp contrast to the banking sector where the regulator
insists that promoters of banks dilute their shareholding to 20 per cent
within a time frame.
The minimum shareholding requirements are part of the Insurance
Regulatory and Development Authority of India’s (IRDAI) draft
guidelines for listed companies.

3. British bank Barclays has launched a new financial technology


platform in which of the following places in India?
A) New Delhi
B) Ahmedabad
C) Mumbai
D) Jaipur
E) Hyderabad

Answer & Explanation

C) Mumbai
Explanation:
British bank Barclays has launched a new financial technology platform
in MUMBAI which will provide these start-ups with a physical site to
work and interact with stakeholders and their peers in other markets
for a fee. The lender hopes that providing such a platform will help it
find innovative financial technology quickly and at a cheaper cost.

4. Soon to be set up India Post Payments Bank will have how


many branches in the country?
A) 650
B) 550
C) 500
D) 750
E) 600

Answer & Explanation

www.bankingpdf.com
A) 650
Explanation:
The communications and IT minister Ravi Shankar Prasad has said that
all 650 branches of the payment bank set up by the department of
posts will be established by September 2017.

5. Who is the head of committee which has been made to select


the next RBI governor, as present RBI Governor Mr. Raghuram
Rajan’s term will end in closer to September 2016?
A) PK Mishra
B) Urjit Patel
C) R Gandhi
D) P K Sinha
E) None of these

Answer & Explanation

D) P K Sinha
Explanation:
The committee route would be in sync with PM Narendra Modi’s remark
that the appointment is an “administrative decision” that will be taken
closer to September when Rajan’s term ends.
PK Sinha is the Cabinet Sectretary.

6. RBI’s which deputy governor/s is going to get retired on July 7


this year?
A) Urjit Patel
B) SS Mundra
C) R Gandhi
D) HR Khan
E) All of the above

Answer & Explanation

www.bankingpdf.com
D) HR Khan
Explanation:
Khan is due to retire on July 7 as he turns 62.
for his replacement, interview is to be conducted by a panel headed by
Cabinet Secretary Pradeep Kumar Sinha.
Urjit Patel had got a 3 yr extension in Jan this year.

7. The State bank of India has entered into alliance with which of
the following companies to provide the acceptance of latter’s
card in India?
A) Discover
B) American Express
C) Capital One
D) MasterCard
E) Amex

Answer & Explanation

B) American Express
Explanation:
India’s largest lender State Bank of India (SBI) and American Express
have entered into alliance to expand AmEx card acceptance in India.

8. RBI has launched new restructuring tool ‘S4A’ to raise banks’


moral hazard risk. What is it?
A) Scheme for Systematic Structuring of Stressed Assets
B) Scheme for Sustainable Structuring of Stressed Assets
C) Scheme for Sustainable Structuring of Scholastic Assets
D) Scheme for Sustainable Strength of Stressed Assets
E) None of these

Answer & Explanation

B) Scheme for Sustainable Structuring of Stressed Assets


Explanation:

www.bankingpdf.com
Under a new ‘Scheme for Sustainable Structuring of Stressed Assets’
(S4A), RBI allowed banks to take equity in debt laden firms permitting
them to split total loans of struggling companies into sustainable and
unsustainable based on the cash flows of the projects.

9. Which bank has launched India’s first certified green bond at


London Stock Exchange (LSE)?
A) State Bank of India
B) Bank of Baroda
C) ICICI Bank
D) Axis Bank
E) None of these

Answer & Explanation

D) Axis Bank
Explanation:
Axis Bank raised $500 million at the London Stock Exchange after it
launched India’s first internationally-listed certified green bond to
finance climate change solutions around the world.

10. Recently, Dr. R Seetharaman has been honored with the


2016 Green Economy Visionary Award. He is the CEO of
A) South Indian Bank
B) Federal Bank
C) Doha Bank
D) Deutsche Bank
E) None of these

Answer & Explanation

C) Doha Bank
Explanation:
A Qatar-based Indian banker has been awarded for his contribution in
environment-friendly activities and for promoting green economy for

www.bankingpdf.com
nearly two decades.
R Seetharaman, Group CEO of Doha Bank, was honoured with the
‘Green Economy Visionary award’ at the 2016 Union of Arab Banks
International Banking Summit held in Rome.

470-480 Questions :

1. The Department of Posts launched a logo and tagline design


contest for its soon-to-be set up India Post Payments Bank on
which of the following website?
A) GovPost
B) IndianPost
C) MyGov
D) PayPost
E) None of these

Answer & Explanation

C) MyGov
Explanation:
The cabinet on June 1, 2016, gave its nod to setting up of the IPPB
under the Department of Posts.
The Department of Post wants to connect with and involve the people
of India in designing the DNA of the India Post Payments Bank. So it
launched this contest.
The contest is open to all Indian citizens, institutions, agencies and
entities for a period of one month, until July 9, 2016.

2. To provide loans to Indo-Spanish joint ventures and local


enterprises in India, State Bank of India (SBI) has signed a
memorandum of agreement with Spain’s which bank to
enhance business synergies?
A) Caja Madrid

www.bankingpdf.com
B) CaixaBank
C) Liberbank
D) Gallego bank
E) None of these

Answer & Explanation

B) CaixaBank
Explanation:
As per the agreement, they will expand banks’ guarantee transaction
businesses by jointly providing credit to Indian-Spanish joint ventures
and Indian local enterprises
The banks will collaborate in areas of mutual interest such as
syndicated loan business, guarantee transactions, trade finance and
export credit agency finance, infrastructure finance and networking
services, among others.

3. Recently an Indian bank launched next generation features on


its app ‘iMobile’ which will enable customers to instantly pay
taxes and book railway tickets within the app. Name the bank.
A) Federal Bank
B) Punjab National Bank
C) Kotak Mahindra Bank
D) ICICI Bank
E) Bandhan Bank

Answer & Explanation

D) ICICI Bank
Explanation:
New features will enable customers to instantly pay taxes and book
railway tickets within the app. iMobile users will also be able to
purchase travel & motor insurance as well as buy mutual funds from 19
asset management companies.

www.bankingpdf.com
4. According to HSBC, India is expected to contribute how much of
the world GDP by the middle of next decade?
A) 4.3%
B) 4.9%
C) 5.2%
D) 3.4%
E) None of these

Answer & Explanation

B) 4.9%
Explanation:
India is expected to equal China’s 4.9% share of world GDP in 2005 by
the middle of next decade and if the country grows faster, the date can
be advanced.
India’s economy grew a stronger-than-expected 7.9% in the March
quarter, making it the fastest growing major economy.
HSBC said that if India grows at 8% rate, it would achieve the 4.9%
share by 2027.

5. Which payment company launched a new POS terminal which


seeks to ease payments across millions of small merchant
outlets across India?
A) PayUmoney
B) Udio
C) Paytm
D) MoneyGram
E) None of these

Answer & Explanation

A) PayUmoney
Explanation:
It can allow even the smallest of merchants to start accepting card
based payments. It can accept all credit/debit cards.

www.bankingpdf.com
The POS terminal is easy to set up, can be operated using a pre- or
post-paid SIM, and does away with the use of paper receipts—making
it cost & battery effective, and, of course, very environment friendly.
With a simple and hassle-free documentation policy, a merchant can
start using a PayUmoney POS terminal within 48 hours of signing up
for the service.

6. Which of the following company launched “Receive and Win”


campaign for Ramzan?
A) Mobikwik
B) Freecharge
C) Paytm
D) MoneyGram
E) Udio

Answer & Explanation

D) MoneyGram
Explanation:
Global money transfer service provider, MoneyGram, has rolled out
Receive & Win, a month-long a promotional campaign set to run
throughout the Ramadan period, beginning 6 June 2016. The exercise
affords customers the opportunity to win an all-expense paid Umrah
trip for two.
The promotional push by MoneyGram is set to run in the Indian states
of Telangana, Bihar, Uttar Pradesh, Gujarat, and Kerala.

7. Name the newly appointed MD of State Bank of Mysore.


A) NK Chari
B) Romil Bhattacharya
C) Surya Kumar
D) CR Sasikumar
E) DP Sardana

Answer & Explanation

www.bankingpdf.com
A) NK Chari
Explanation:
N.K.Chari, the new Managing Director of State Bank of Mysore,
assumed office on May 30. Prior to his new position, he was the Deputy
Managing Director of State Bank of India.

8. Name the newly appointed MD of State Bank of Travancore.


A) PN Naidu
B) Naresh Sharma
C) NK Chari
D) CR Sasikumar
E) Ravi Verma

Answer & Explanation

D) CR Sasikumar
Explanation:
CR Sasikumar assumed the charge of Managing Director of State Bank
of Travancore (SBT), one of the associate banks of State Bank of India
(SBI), on June 1.

9. Which of the following international organization has lead the


race to buy stake in government controlled IDBI bank?
A) Asian Development Bank
B) Commonwealth Development Corporation
C) International Finance Corporation
D) GIC of Singapore
E) None of these

Answer & Explanation

C) International Finance Corporation


Explanation:
World Bank’s private sector lending arm International Finance
Corporation (IFC) has emerged as the top contender to buy around 15

www.bankingpdf.com
per cent in government controlled IDBI Bank, even as talks have been
held with at least three other players-GIC of Singapore, Asian
Development Bank and Commonwealth Development Corporation.
IFC is World Bank arm and is looking for a greater say in running IDBI.

10. Government has asked banks to increase the coverage under


which of the following schemes?
A) Atal Pension Yojana
B) Jan Dhan Yojana
C) Suraksha Bima Yojana
D) Both A and C
E) None of these

Answer & Explanation

A) Atal pension Yojana


Explanation:
Government has asked banks to take steps to increase coverage under
the Atal pension scheme as banks failed in meeting the sales target for
the social security scheme in the last financial year.
All the large PSBs, including State Bank of India, fell short of the target
by a wide margin.

480-490 Questions :

1. Which of the following High Courts has ruled that banks can
recover bad loans from person’s saving account if the
agreement allows?
A) Punjab and Haryana
B) Madras
C) Mumbai
D) Assam
E) Kerala

www.bankingpdf.com
Answer & Explanation

B) Madras
Explanation:
If a debtor failed to settle the loan dues despite reminders from the
bank, then the money could be recovered from his savings bank
account, ruled the Madras High Court bench

2. The newest private sector bank, Bandhan Bank has announced


that it targets 180 new branches by March 2017. Who is the MD
and CEO of Bandhan Bank?
A) Vikram Limaye
B) Rajiv B. Lall
C) Chandra Shekhar Ghosh
D) Narain Chand
E) None of these

Answer & Explanation

C) Chandra Shekhar Ghosh


Explanation:
Bandhan Bank plans to open 180 new branches by March next year
with an aim to build a pan India brand.
It is headquartered at Kolkata.

3. Which Indian bank has signed an agreement with Thailand’s


Kasikorn Bank to enhance cooperation in trade and
investment?
A) State Bank of India
B) Yes Bank
C) Canara Bank
D) Axis Bank
E) Union Bank of India

Answer & Explanation

www.bankingpdf.com
D) Axis Bank
Explanation:
The MoU will serve to strengthen existing ties between the two banks
and will help facilitate and enhance cooperation in the areas of trade,
investment and other businesses
The agreement between the two banks were signed as the Thailand
Prime Minister Prayut Chan-o-cha is on a three-day visit to India.

4. Reserve Bank of India has granted in principle approval to the


common service Centres to act as which operating unit to make
bill payments in rural?
A) Bharat Bill Payment
B) National Bill Payment
C) Non Banking Financial
D) Mobile Banking Payment
E) None of these

Answer & Explanation

A) Bharat Bill Payment


Explanation:
Reserve Bank of India has granted in-principle nod to the common
service centres RBI (CSCs) as Bharat Bill Payment Operating Unit to
make bill payments at centres in rural bands.

5. Name the bank which has been awarded with CFBP (Council for
Fair Business Practices) Jamnalal Bajaj Award for Fair Business
Practices?
A) Karnataka Bank
B) Federal Bank
C) RBS Bank
D) Laxmi Vilas Bank
E) None of these

Answer & Explanation

www.bankingpdf.com
A) Karnataka Bank
Explanation:
Mangaluru based Karnataka Bank has bagged CFBP Jamnalal Bajaj
Award for Fair Business Practices, a certificate of merit under the
category service enterprises – large.

6. Name the US based bank which has become the first bank to
issue the first ever credit card to be used in Cuba?
A) American Express
B) SunTrust Bank
C) Citigroup
D) Stonegate Bank
E) Wells Fargo

Answer & Explanation

D) Stonegate Bank
Explanation:
Pompano Beach-based Stonegate Bank issued the first of its
MasterCard credit cards for use in Cuba
The South Florida bank was the first – and so far the only bank to
launch a debit card for use by U.S. travelers in Cuba.

7. The State bank of India and America’s which financial company


have jointly launched a distribution alliance with travel prepaid
cards distributor Centrum?
A) Visa
B) American Express
C) Capital One
D) MasterCard
E) Amex

Answer & Explanation

www.bankingpdf.com
D) MasterCard
Explanation:
India’s largest bank, State Bank of India (SBI) and MasterCard
announced pilot launch of a distribution alliance with travel prepaid
cards distributor Centrum. This partnership will aid the distribution
strength of the Bank for it’s multicurrency prepaid card and will largely
target the corporate customers of the bank.

8. Jayant Sinha, minister of state for finance reported how many


willful defaulters owing Rs 66,190 crore to state-owned banks
as of December 2015?
A) 6719
B) 7128
C) 5275
D) 7686
E) None of these

Answer & Explanation

D) 7686
Explanation:
Sinha also said that the total outstanding amount in top 100 non-
performing accounts (NPAs) with public sector banks (PSBs) was Rs
1.73 lakh crore as of December 2015.
Willful defaulters of PSBs rose from 5,554 to 7,686 in three years to
December 2015 while the amount involved more than doubled to Rs
66,190 crore from Rs 27,749 crore.

9. The State Bank of India has created how much crores fund for
investing in IT or Fintech startups?
A) 200
B) 50
C) 100

www.bankingpdf.com
D) 150
E) 300

Answer & Explanation

A) 200
Explanation:
India’s largest public sector lender State Bank of India has created IT
innovations and startup fund and have allotted Rs 200 crore for
investment in promising startups.
ii. The fund would be covering upto Rs 3 crore for any India registered
entity for their business promotion in India using information
technology for banking.

10. Max Life and Max Financial Services have entered into
merger agreement with which of the following insurance
company?
A) Oriental Insurance Company
B) ICICI Prudential Life Insurance Co. Ltd
C) HDFC Standard Life
D) Tata AIG general Insurance
E) Birla Sun Life Insurance

Answer & Explanation

C) HDFC Standard Life


Explanation:
HDFC Standard Life and Max Life and Max Financial Services are
entered into merger agreement that will create the largest private
player in India’s insurance sector.
The proposed merger between the companies will create an entity with
assets worth over Rs 1 lakh crore.
A combination of the entities would be overtaking ICICI Prudential Life
Insurance Co. Ltd

www.bankingpdf.com
490-500 Questions :

1. According to a recent news, the State Bank of India was being


put in “Hall of Shame” list along with some other banks. The
reason for this was
A) for not recovering the NPAs in the last 6 months
B) for financing a US cluster bomb manufacturer
C) for supporting Vijay Malaya
D) it listed the worst NPAs ever the last quarter
E) None of these

Answer & Explanation

B) for financing a US cluster bomb manufacturer


Explanation:
The State Bank of India has defended financing a US cluster bomb
manufacturer, saying its investments were legal, after being put in a
“Hall of Shame” along with several other major banks by a Dutch
campaign group.
A report by PAX last week listed the government-owned bank as one of
158 lenders – including JP Morgan Chase, Barclays, Bank of America
and Credit Suisse – that violated an international ban for investing in
cluster bomb firms.

2. How much amount is going to be disbursed by banks under


Pradhan Mantri Mudra Yojana in FY’17?
A) Rs 2.80 lakh crore
B) Rs 1.50 lakh crore
C) Rs 1.80 lakh crore
D) Rs 1.30 lakh crore
E) Rs 1.20 lakh crore

Answer & Explanation

www.bankingpdf.com
C) Rs 1.80 lakh crore
Explanation:
Having met the target last year, banks are expected to disburse Rs
1.80 lakh crore loans under the Pradhan Mantri Mudra Yojana (PMMY)
in the current fiscal, Minister of State for Finance Jayant Sinha said
today.
Last year, 3.5 crore beneficiaries availed Rs 1.22 lakh crore loans
under PMMY.

3. Star Union Dai-Ichi Life insurance is a life insurance joint


venture between life insurer Dai-Ichi and which banks?
A) Indian Overseas Bank and United Bank of India
B) Bank of India and Union Bank
C) Canara Bank and Bank of India
D) Allahabad Bank and OBC Bank
E) Union Bank and Bank of Baroda

Answer & Explanation

B) Bank of India and Union Bank


Explanation:
Star Union Dai-Ichi Life insurance = Japan’s largest life insurer Dai-Ichi
+ Bank of India + Union Bank

4. Bank of India has sold how much percent of its stake in Star
Union Dai-Ichi Life insurance to Dai-Ichi?
A) 18%
B) 26%
C) 38%
D) 15%
E) 12%

Answer & Explanation

www.bankingpdf.com
A) 18%
Explanation:
The shareholding of Dai-Ichi in the Indian life insurance venture will
now increase from 26% to 44% while Bank of India’s shareholding will
decrease from 30% to 48%. Union Bank will continue to hold 26%.

5. Which of the following banks has planned to commence urban


microfinance operations in its 85 branches across the country?
A) Yes Bank
B) Kotak Mahindra Bank
C) ICICI Bank
D) Axis Bank
E) None of these

Answer & Explanation

D) Axis Bank
Explanation:
India’s third largest private sector lender Axis Bank has forayed into
Urban Microfinance segment and will now provide collateral free credit
facilities to low-income women groups. The urban micro lending
initiative ‘Axis Sahyog’ under the Bank’s flagship microfinance
programme will be delivered via its technology driven ‘TAB based
lending’ solution.

6. Aadhar Housing Finance Ltd is going to be merged with which


of the following?
A) DHFL Vysya Housing Finance Ltd
B) Cent Bank Home Finance Limited
C) Housing and Urban Development Corporation Ltd.
D) Manipal Housing Finance Syndicate Ltd.
E) None of these

Answer & Explanation

www.bankingpdf.com
A) DHFL Vysya Housing Finance Ltd
Explanation:
Dewan Housing group companies Aadhar Housing Finance Ltd and
DHFL Vysya Housing Finance Ltd will be merged into one to create a
pan-India entity with Rs 3,300 crore of loan assets for financing
affordable housing, a priority area for the Narendra Modi-led
government.
The merger plan received backing from International Finance Company
(IFC), which holds 20% in Aadhar Finance and provides financial
support to India’s scheme of promoting dwelling units for low income
families.

7. Which of the following international organization is going to


invest Rs 60 crore in Aadhar Housing Finance?
A) International Monetary Fund
B) International Finance Corporation
C) Asian Development Bank
D) World Bank
E) European Investment Bank

Answer & Explanation

B) International Finance Corporation


Explanation:
The World Bank’s private sector funding arm International Finance
Corporation (IFC) and Wadhawan Global Capital Pvt Ltd are going to
raise their investments in Aadhar Housing Finance by putting in Rs 60
crore between them to help the housing loan provider to lower income
families double its business.

8. According to a report by Reserve Bank of India, country’s gross


bad loans are expected to grow from 7.6% to ________ by
March 2017.
A) 9.2%

www.bankingpdf.com
B) 8.2%
C) 8.5%
D) 8.0%
E) None of these

Answer & Explanation

C) 8.5%
Explanation:
Gross bad loans at banks may rise to 8.5 percent of total assets by
March 2017 from 7.6 percent in March 2016 if the central bank orders
them to conduct a second round of asset quality reviews.
Meanwhile, under a “severe stress” situation, total bad loans could rise
to 9.3 percent in March 2017.

9. Which of the following has decided to block LPG subsidy and


PAN of the tax defaulters?
A) RBI
B) SBI
C) Income Tax Department
D) Govt. of India
E) Finance Ministry

Answer & Explanation

C) Income Tax Department


Explanation:
Income Tax Department is set to block Permanent Account Number
(PAN) of wilful defaulters and their LPG subsidy and take measures to
ensure that they are not sanctioned loans.

10. According to United Nations Conference on Trade and


Development (UNCTAD) report, what is the position of India in
Asia among the largest recipients of Foreign Direct Investment
(FDI)?

www.bankingpdf.com
A) 12th
B) 10th
C) 4th
D) 9th
E) 6th

Answer & Explanation

C) 4th
Explanation:
Switzerland based United Nations Conference on Trade and
Development (UNCTAD) released the World Investment Report stated
that India is the fourth largest recipient of Foreign Direct Investment in
Asia and ranked tenth Position in World.
UN Economist said that involvement of India in Foreign Direct
Investment is likely to reach USD 60 billion in 2016

500-510 Questions :

1. Reserve Bank announced the cap under Market Stabilisation


Scheme (MSS) which has been fixed at Rs 30,000 crore for the
current fiscal. MSS was launched in the year
A) 2001
B) 2006
C) 2005
D) 2004
E) None of these

Answer & Explanation

D) 2004
Explanation:
The scheme was launched in 2004.Under this scheme securities are
issued with the objective of providing the RBI with a stock of securities

www.bankingpdf.com
with which it can intervene in the market for managing liquidity.
These securities are issued not to meet the government’s expenditure.

2. Which of the following has approved $500 million loan for


bridge over Ganga River in Bihar?
A) World Bank
B) Asian Development Bank
C) International Monetary Fund
D) European Development Bank
E) None of these

Answer & Explanation

B) Asian Development Bank


Explanation:
Asian Development Bank accorded $500 million to build bridge over
India’s longest river bridge which is in Bihar having distance of 9.8 km
road.

3. First Annual General Meeting of Asian Infrastructure


Investment Bank held at
A) Dhaka
B) Jakarta
C) Shanghai
D) Beijing
E) None of these

Answer & Explanation

D) Beijing
Explanation:
Finance Minister Arun Jaitley Participated in the First Annual General
Meeting of Asian Infrastructure Investment Bank at Beijing to discuss
about the investment in Infrastructure and accorded the approval to
establish the Regional Office of AIIB in New Delhi.

www.bankingpdf.com
4. Which of the following approved $63 million for Tejaswini
Project in Jharkhand?
A) World Bank
B) Asian Development Bank
C) International Monetary Fund
D) European Development Bank
E) None of these

Answer & Explanation

A) World Bank
Explanation:
The World Bank Board has approved the 63 million US dollar for
Tejaswini project that paves the way for Socioeconomic Empowerment
of Adolescent Girls and Young Women in Jharkhand.

5. Which of the following approved $201.50 million for Technical


Education Quality Improvement Project in India?
A) World Bank
B) Asian Development Bank
C) International Monetary Fund
D) European Development Bank
E) None of these

Answer & Explanation

A) World Bank
Explanation:
United States based World Bank’s Board of Executive Directors has
approved $201.50 million for Technical Education Quality Improvement
Project III in India

6. India is to get $1 bn funding for solar projects from which of


the following?
A) World Bank

www.bankingpdf.com
B) Asian Development Bank
C) International Monetary Fund
D) European Development Bank
E) None of these

Answer & Explanation

A) World Bank
Explanation:
The World Bank today committed USD 1 billion (about Rs 6,750 crore)
to support solar energy programme in India, which is reducing
dependence on conventional energy sources to reduce greenhouse gas
emissions.

7. SBI is going to offer its banking services on Facebook, Twitter


through?
A) SBI Quick
B) SBI Online
C) SBI Sort
D) SBI Social
E) SBI Mingle

Answer & Explanation

E) SBI Mingle
Explanation:
Leveraging on the popularity of Facebook and Twitter, State Bank of
India (SBI) today launched ‘SBI Mingle’, allowing its customers access
various banking services via these social platforms. Using SBI Mingle,
the bank’s customers can do a host of banking services on their
Facebook or Twitter accounts at their own convenience.

8. Name the newly appointed deputy governor of RBI.


A) NS Vishwanathan
B) KR Subramaniam

www.bankingpdf.com
C) Sarvesh Rana
D) NS Iyer
E) None of these

Answer & Explanation

A) NS Vishwanathan
Explanation:
The Union Government has appointed NS Vishwanathan as deputy
governor of the Reserve Bank of India (RBI). His appointment was
approved by the Appointments Committee of the Cabinet (ACC)
presided by Prime Minister Narendra Modi in New Delhi.

9. Reliance Industries (RIL) has signed agreement with which of


the following to set up payments bank?
A) Canara Bank
B) State Bank of India
C) Yes Bank
D) Axis Bank
E) Indian Overseas Bank

Answer & Explanation

B) State Bank of India


Explanation:
Telecom-to-energy conglomerate Reliance Industries (RIL) and India’s
largest lender State Bank of India (SBI) have signed Subscription and
Shareholders’ Agreement to set up payments bank. As per the
agreement, RIL will be the promoter with a 70 per cent equity
contribution in the payments bank and SBI as joint venture will have
30 per cent equity contribution.

10. Which of the banks has signed agreements with the World
Bank for $625 million to support grid connected rooftop solar
programme in the Country?

www.bankingpdf.com
A) Canara Bank
B) State Bank of India
C) Bank of India
D) Bank of Maharashtra
E) Indian Overseas Bank

Answer & Explanation

B) State Bank of India


Explanation:
State Bank of India has signed agreements with the World Bank for
$625 million to support grid connected rooftop solar programme in the
Country
SBI Capital Markets was keen advisor for structuring and setting up the
facility

510-520 Questions :

1. _________ is an enterprise where investment in plant and


machinery does not exceed Rs. 25 lakh.
A. Micro enterprise
B. Medium enterprise
C. Small enterprise
D. None of the Above

Answer & Explanation

A. Micro enterprise
Explanation:
Enterprises engaged in the manufacture or production, processing or
preservation of goods in where the investment in plant and machinery
does not exceed Rs. 25 lakh is defined as Micro enterprise.

www.bankingpdf.com
2. What is the maximum limit of investment in plant and
machinery by a small enterprise as per the MSMED Act 2006?
A. 5 crore
B. 10 crore
C. 15 crore
D. 20 crore

Answer & Explanation

A. 5 crore
Explanation:
In small enterprises the investment in plant and machinery is more
than Rs. 25 lakh but does not exceed Rs. 5 crore as per the Micro
Small and Medium Enterprises Development (MSMED) Act, 2006.

3. An enterprise where the investment in plant and machinery is


more than Rs.5 crore but does not exceed Rs.10 crore is termed
as _________
A. Micro enterprise
B. Medium enterprise
C. Small enterprise
D. None of the Above

Answer & Explanation

B. Medium enterprise
Explanation:
In medium enterprises the minimum limit of investment in plant and
machinery is Rs.5 crore and the maximum limit is Rs.10 crore.

4. What is the maximum limit of investment in equipment by a


micro enterprise as per the MSMED Act 2006?
A. 5 lakh
B. 10 lakh

www.bankingpdf.com
C. 15 lakh
D. 20 lakh

Answer & Explanation

B. 10 lakh
Explanation:
In micro enterprises the investment in equipment does not exceed Rs.
10 lakh.

5. What is the range of investment in equipment by a small


enterprise as per the MSMED Act 2006?
A. 5 lakh – 1 crore
B. 10 lakh – 2 crore
C. 15 lakh – 3 crore
D. 20 lakh – 4 crore

Answer & Explanation

B. 10 lakh – 2 crore
Explanation:
In small enterprises, the investment in equipment lies between Rs.10
lakh and Rs.2 crore.

6. According to MSMED Act, 2006 an enterprise where the


investment in equipment lies between Rs.2 crore and Rs.5 crore
is termed as _________
A. Micro enterprise
B. Medium enterprise
C. Small enterprise
D. None of the Above

Answer & Explanation

B. Medium enterprise
Explanation:

www.bankingpdf.com
Medium enterprise – An enterprise where the investment in equipment
is more than Rs. 2 crore but does not exceed Rs. 5 crore.

7. MUDRA stands for __________


A. Micro Units Development and Refinance Agency Bank
B. Micro Units Development and Refinance Authority Bank
C. Medium Units Development and Refinance Agency Bank
D. Medium Units Development and Refinance Authority Bank

Answer & Explanation

A. Micro Units Development and Refinance Agency Bank


Explanation:
Micro Units Development and Refinance Agency Bank (or MUDRA Bank)
is a public sector financial institution in India.
It provides loans at low rates to micro-finance institutions and non-
banking financial institutions which then provide credit to MSMEs.
It was launched by Prime Minister Narendra Modi in New Delhi.

8. Who among the following is/are eligible to borrow from MUDRA


Bank?
A. Small manufacturing unit
B. Artisans
C. Fruit and vegetable vendors
D. All of the Above

Answer & Explanation

D. All of the Above


Explanation:
Eligible persons to borrow from MUDRA bank are as follows
♦ Small manufacturing unit
♦ Shopkeepers
♦ Fruit and vegetable vendors
♦ Artisans

www.bankingpdf.com
9. Government has decided to provide an additional fund of
________ which will be allocated to Shishu, Kishore and Tarun
schemes.
A. Rs 1 trillion
B. Rs 1 million
C. Rs 1 billion
D. Rs 10 billion

Answer & Explanation

A. Rs 1 trillion
Explanation:
Government has decided to provide an additional fund of Rs 1 trillion to
the market and will be allocated as
♦ 40% to Shishu.
♦ 35% to Kishore.
♦ 25% to Tarun.

10. MUDRA Bank will deliver loan through _________


A. NBFCs
B. MFIs
C. Rural Banks
D. All of the Above

Answer & Explanation

D. All of the Above


Explanation:
MUDRA will deliver the loan through NBFCs, MFIs, Rural Banks, District
Banks, Nationalize Banks, Private Banks, Primary Lending Institutions
and other intermediaries.

www.bankingpdf.com
520-530 Questions :

1. __________ are certificates issued by banks that have


overreached their priority sector lending targets.
A. Commercial Papers
B. Certificate of Deposits
C. Treasury Bills
D. Priority Sector Lending Certificates

Answer & Explanation

D. Priority Sector Lending Certificates


Explanation:
Domestic banks as well as foreign banks(with more than 20 branches)
have to lend 40% of their total loans to eight priority sectors which
consist of agriculture, micro credits, education, social infrastructure
and housing so that these unprivileged sections can get credit. To
encourage lending to the priority sector, the central bank has
introduced the concept of Priority Sector Lending Certificates (PSLC).

2. How many types of PSLCs can be issued by the Banks?


A. Four
B. Five
C. Six
D. Eight

Answer & Explanation

A. Four
Explanation:
Banks can issue four types of PSLCs including three subsector PSLCs-
agriculture, small and marginal farmers, micro enterprises and one
PSLC for general.

www.bankingpdf.com
3. What is the expiry date of Priority Sector Lending Certificates
(PSLC)?
A. April 1
B. March 1
C. June 1
D. January 1

Answer & Explanation

A. April 1
Explanation:
All Priority Sector Lending Certificates(PSLCs) will be valid till March
31st and will expire on April 1st.

4. The Priority Sector Lending Certificates(PSLCs) can be traded


using the RBI’s_______
A. e-Kuber platform.
B. NEFT
C. RTGS
D. None of the above

Answer & Explanation

A. e-Kuber platform.
Explanation:
RBI launched a platform to enable trading in the priority sector lending
certificates (PSLC) through its Core Banking Solution (CBS) portal (e-
Kuber). Kuber refers to Lord Kubera – the lord of wealth in Hindu
Mythology.

5. The price of PSLCs will be determined on the basis of_______


A. demand and supply
B. Performance
C. Both(A) and (B)
D. None of the above

www.bankingpdf.com
Answer & Explanation

A. demand and supply


Explanation:
The price of PSLCs will be determined on the basis of demand and
supply that will be reflected in the auction under the RBI’s e-Kuber
trading platform.

6. Which among the following is/are the different type(s) of


category under priority sector?
A. Agriculture
B. Micro, Small and Medium Enterprises
C. Export Credit
D. All of the Above

Answer & Explanation

D. All of the Above


Explanation:
Priority Sector includes 8 different categories.
♦ Agriculture
♦ Micro, Small and Medium Enterprises(MSME)
♦ Export Credit
♦ Education
♦ Housing
♦ Social Infrastructure
♦ Renewable Energy
♦ Others

7. What is the loan limit for education in the country under


priority sector?
A. 15 lakh
B. 10 lakh
C. 8 lakh
D. 20 lakh

www.bankingpdf.com
Answer & Explanation

B. 10 lakh
Explanation:
Loan limit to individuals for education including vocational courses
under priority sector is upto Rs.10 lakh.

8. Which among the following is/are seller(s) & buyer(s) of


PSLCs?
A. Scheduled Commercial Banks (SCBs)
B. Regional Rural Banks (RRBs)
C. Local Area Banks (LABs)
D. All of the Above

Answer & Explanation

9. How many sub-categories are classified under Agriculture


sector?
A. Four
B. Five
C. Six
D. Three

Answer & Explanation

D. Three
Explanation:
Agriculture sector has been classified under three sub-categories
namely Farm credit, Agriculture infrastructure and Ancillary activities.

10. What is the maximum limit for housing loans under priority
sector in metropolitan cities?

www.bankingpdf.com
A. 35 lakh
B. 40 lakh
C. 45 lakh
D. 55 lakh

Answer & Explanation

A. 35 lakh
Explanation:
Loans to individuals up to Rs.28 lakh in metropolitan centres and does
not exceed Rs.35 lakh

530-540 Questions :

1. When the words “or bearer” appearing on the face of the


cheque are not cancelled, then the cheque is called a
__________
A. Order Cheque
B. Crossed Cheque
C. Ante-Dated Cheque
D. Bearer Cheque

Answer & Explanation

D. Bearer Cheque
Explanation:
A bearer cheque can be issued to a third party in his name or in the
name of the firm, who can draw the amount across the counter of the
bank.

2. When the word “bearer” appearing on the face of a cheque is


cancelled and when in its place the word “or order” is written
on the face of cheque is called ____________

www.bankingpdf.com
A. Order Cheque
B. Crossed Cheque
C. Ante-Dated Cheque
D. Bearer Cheque

Answer & Explanation

A. Order Cheque
Explanation:
When the word “or bearer” printed on the cheque is cancelled and and
the word ‘order’ may be written on the cheque, the cheque is called an
order cheque. An order cheque is one which is payable to a particular
person. The payee can transfer an order cheque to someone else by
signing his or her name on the back of it.

3. Which means drawing two parallel lines on the face of cheque


with or without additional words like “& CO” or “account
payee” or “Not Negotiable”?
A. Order Cheque
B. Crossed Cheque
C. Ante-Dated Cheque
D. Post-Dated Cheque

Answer & Explanation

B. Crossed Cheque
Explanation:
Any cheque that is crossed with two parallel lines, either across the
whole cheque or through the top left hand corner of the cheque is
called crossed cheque. It has been marked to specify an instruction
about the way it is to be redeemed.

4. If any cheque bears a date earlier than the date on which is


presented to the bank is called ___________
A. Order Cheque

www.bankingpdf.com
B. Crossed Cheque
C. Ante-Dated Cheque
D. Post-Dated Cheque

Answer & Explanation

C. Ante-Dated Cheque
Explanation:
An ante-dated cheque is one which bears a date earlier than the date
of issue.

5. If any cheque bears a date which is yet to come then it is


known as___________
A. Order Cheque
B. Mutilated Cheque
C. Ante-Dated Cheque
D. Post-Dated Cheque

Answer & Explanation

D. Post-Dated Cheque
Explanation:
post-dated cheque is a cheque written by the drawer (payer) for a date
in the future.

6. A Cheque which is torn into two or more pieces and presented


for payment, such a cheque is called _________
A. Stale Cheque
B. Mutilated Cheque
C. Ante-Dated Cheque
D. Post-Dated Cheque

Answer & Explanation

B. Mutilated Cheque
Explanation:

www.bankingpdf.com
If a cheque is torn into two or more pieces such cheque is Mutilated
Cheque. If that cheque is presented for payment,then the bank will not
make payment against such a cheque without getting confirmation of
the drawer.But if a cheque is torn at the corners and no material fact is
erased or cancelled, the bank may make payment against such a
cheque.

7. If a cheque is presented for payment after three months from


the date of the cheque is called ______
A. Stale Cheque
B. Mutilated Cheque
C. Self Cheque
D. Truncated Cheque

Answer & Explanation

A. Stale Cheque
Explanation:
A stale check is not an invalid check, but it may be deemed an
‘irregular’ bill of exchange.

8. Which of the following cheque is written by the account holder


as pay self to receive the money in the physical form from the
branch where he holds his account?
A. Stale Cheque
B. Mutilated Cheque
C. Self Cheque
D. Truncated Cheque

Answer & Explanation

C. Self Cheque
Explanation:
A self cheque: A self cheque is written by the account holder as pay

www.bankingpdf.com
self to receive the money in the physical form from the branch where
he holds his account.

9. A cheque which is truncated during the course of clearing cycle


is called __________
A. Stale Cheque
B. Mutilated Cheque
C. Self Cheque
D. Truncated Cheque

Answer & Explanation

D. Truncated Cheque
Explanation:
A truncated cheque means a cheque which is truncated during the
course of clearing cycle either by the clearing house or by the bank.

10. Person named in the instrument to whom or to whose order


the money is to be paid is known as ______
A. Drawer
B. Drawee
C. Payer
D. Payee

Answer & Explanation

D. Payee
Explanation:
A person to whom money is paid or is to be paid, especially the person
to whom a cheque is made payable.

www.bankingpdf.com
540-550 Questions :

1. National Housing Bank (NHB), a wholly owned subsidiary of the


central bank, was established in 1988 under the _________
A.National Housing Bank Act, 1987
B.National Housing Bank Act, 1988
C.National Housing Bank Act, 1986
D.National Housing Bank Act, 1985

Answer & Explanation

A.National Housing Bank Act, 1987


Explanation:
National Housing Bank (NHB), a wholly owned subsidiary of the central
bank, was established in 1988 under the National Housing Bank Act,
1987.

2. National Housing Bank (NHB), is an apex financial institution


for housing, having headquarters in ______
A. Mumbai
B. New Delhi
C. Bengaluru
D. Hyderabad

Answer & Explanation

B. New Delhi
Explanation:
National Housing Bank (NHB), a wholly owned subsidiary of RBI, was
set up by an Act of Parliament in 1987 having headquarters in New
Delhi.

3. The Banking Ombudsman Scheme was introduced under


__________ of the Banking Regulation Act, 1949 by RBI with
effect from 1995.

www.bankingpdf.com
A. Section 35A
B. Section 25A
C. Section 45A
D. Section 15A

Answer & Explanation

A. Section 35A
Explanation:
The Banking Ombudsman Scheme was introduced under Section 35 A
of the Banking Regulation Act, 1949 by RBI with effect from 1995.

4. Which of the following is/are covered under the Banking


Ombudsman Scheme, 2006?
A. Scheduled Commercial Banks
B. Regional Rural Banks
C. Primary Co-operative Banks
D. All of the Above

Answer & Explanation

D. All of the Above


Explanation:
All Scheduled Commercial Banks, Regional Rural Banks and Scheduled
Primary Co-operative Banks are covered under the the Banking
Ombudsman Scheme, 2006.

5. The Banking Ombudsman is a senior official appointed by the


___________
A. SEBI
B. Reserve Bank of India
C. Indian Bank’s Association
D. NABARD

Answer & Explanation

www.bankingpdf.com
B. Reserve Bank of India
Explanation:
The Banking Ombudsman is a senior official appointed by the Reserve
Bank of India to redress customer complaints against deficiency in
certain banking services.

6. How many Banking Ombudsmen have been appointed?


A. 12
B. 11
C. 10
D. 15

Answer & Explanation

D. 15
Explanation:
Fifteen Banking Ombudsmen have been appointed with their offices
located mostly in state capitals.

7. What is the maximum amount of compensation under Banking


Ombudsman scheme 2006?
A. Rs.4 lakh
B. Rs.3 lakh
C. Rs.5 lakh
D. Rs.10 lakh

Answer & Explanation

D. Rs.10 lakh
Explanation:
The amount, if any, to be paid by the bank to the complainant by way
of compensation for any loss suffered by the complainant is limited to
the amount of Rs 10 lakh.

www.bankingpdf.com
8. Any person can file a complaint before the the Banking
Ombudsman, if the reply is not received from the bank within a
period of _________
A. 15 days
B. one month
C. two months
D. three months

Answer & Explanation

B. one month
Explanation:
One can file a complaint before the Banking Ombudsman if the reply is
not received from the bank within a period of one month.

9. How much amount to be charged by the Banking Ombudsman


for filing and resolving customers’ complaints?
A. Rs.15
B. Rs.50
C. Rs.100
D. No fee

Answer & Explanation

D. No fee
Explanation:
The Banking Ombudsman does not charge any fee for filing and
resolving customers’ complaints.

10. The Indian Banks’ Association (IBA) is an association of


Indian banks and financial institutions having headquarters in
________
A. Mumbai
B. New Delhi

www.bankingpdf.com
C. Bengaluru
D. Hyderabad

Answer & Explanation

A. Mumbai
Explanation:
Indian Banks’ Association (IBA), formed in 1946 as a representative
body of management of banking in India. It is head-quartered in
Mumbai. IBA is managed by a managing committee, and the current
managing committee consists of one chairman, 3 deputy chairman, 1
honorary secretary and 26 members

550-560 Questions :

1. Which of the following Act provides provisions for regulation of


credit information companies?
A. Companies Act, 1956
B. Credit Information (Companies Regulation Act, 2005)
C. Information Technology Act, 2000
D. Banking Regulation Act, 1949

Answer & Explanation

B. Credit Information (Companies Regulation Act, 2005)


Explanation:
Credit Information (Companies Regulation Act, 2005) provides for
regulation of credit information companies and to facilitate efficient
distribution of credit and for matters connected therewith or incidental
thereto.

2. Which of the following Act refers to the acquisition and transfer


of the undertakings of certain banking companies?

www.bankingpdf.com
A. Companies Act, 1956
B. Credit Information (Companies Regulation Act, 2005)
C. Bankings Companies (acquisition and transfer of undertakings)
D. Banking Regulation Act, 1949

Answer & Explanation

C. Bankings Companies (acquisition and transfer of


undertakings)
Explanation:
Bankings Companies (acquisition and transfer of undertakings) provide
provisions for the acquisition and transfer of the undertakings of
certain banking companies having regard to their size, resources,
coverage and organization in order to control the heights of the
economy and to meet progressively.

3. Which act provide legal recognition for transactions carried out


by means of electronic communication, commonly referred to as
e-commerce?
A. Companies Act, 1956
B. Credit Information (Companies Regulation Act, 2005)
C. Information Technology Act, 2000
D. Banking Regulation Act, 1949

Answer & Explanation

C. Information Technology Act, 2000


Explanation:
Information Technology Act, 2000 provide legal recognition for
transactions carried out by means of electronic communication,
commonly referred to as e-commerce which involve the use of
alternative to paper based methods of communication.

4. As per the provisions of ________ act all financial institutions


have to maintain a record for all form of transactions?

www.bankingpdf.com
A. Companies Act, 1956
B. Credit Information (Companies Regulation Act, 2005)
C. Information Technology Act, 2000
D. PMLA Act, 2002

Answer & Explanation

D. PMLA Act, 2002


Explanation:
As per the provisions of Prevention of Money laundering Act(PMLA Act,
2002) all financial institutions (Ex:- NBFC, Co-operative Bank etc.,)
and intermediary which includes a stock broker, sub broker etc., have
to maintain a record for all form of transactions.

5. The income of a Bank is chargeable to income tax under


________ of Income Tax Act, 1961
A. Section 28
B. Section 29
C. Section 31
D. Section 32

Answer & Explanation

A. Section 28
Explanation:
The income of a Bank is chargeable to income tax under section
28[Profits and Gains of Business Profession]of Income Tax Act, 1961.

6. According to ___________ of the Negotiable Instruments Act


1881, a negotiable instrument means a promissory note, bill of
exchange or cheque payable either to order or to bearer.
A. Section 20
B. Section 19
C. Section 11
D. Section 13

www.bankingpdf.com
Answer & Explanation

D. Section 13
Explanation:
According to Section 13 of the Negotiable Instruments Act 1881, a
negotiable instrument means a promissory note, bill of exchange or
cheque payable either to order or to bearer.

7. Export–Import Bank of India(EXIM Bank) established in


_______ under the Export-Import Bank of India Act 1981.
A. 1983
B. 1982
C. 1984
D. 1985

Answer & Explanation

B. 1982
Explanation:
Export-Import Bank of India for
providing financial assistance to exporters and importers, and for
functioning as the
principal financial institution for co-ordinating the working of
institutions engaged in
financing export and import of goods and services was established in
1982 under the Export-Import Bank of India Act 1981.

8. Small Industries Development Bank of India (SIDBI), set up


under which Act?
A. SIDBI Act–1989
B. SIDBI Act–1987
C. SIDBI Act–1986
D. SIDBI Act–1985

Answer & Explanation

www.bankingpdf.com
A. SIDBI Act–1989
Explanation:
Small Industries Development Bank of India (SIDBI), set up on April 2,
1990 under SIDBI Act–1989, is the Principal Financial Institution for
the Promotion, Financing and Development of the Micro, Small and
Medium Enterprise (MSME) sector.

9. National Bank for Agriculture and Rural Development (NABARD)


is an apex development bank in India, having headquarters in
______
A. Mumbai
B. New Delhi
C. Bengaluru
D. Hyderabad

Answer & Explanation

A. Mumbai
Explanation:
National Bank for Agriculture and Rural Development (NABARD) is an
apex development bank in India, having headquarters in Mumbai.It
was established on 12 July 1982 by NABARD Act 1981.

10. The Committee to review arrangements for Institutional


Credit for Agriculture and Rural Development (CRAFICARD), set
up by the RBI under the chairmanship of _________
A. Shri B. Sivaraman
B. Shri D. Raman
C. Shri A. Ram Kumar
D. Shri Ajay Shankar

Answer & Explanation

A. Shri B. Sivaraman
Explanation:

www.bankingpdf.com
The Committee to review arrangements for Institutional Credit for
Agriculture and Rural Development (CRAFICARD), set up by the RBI
under the chairmanship of B. Sivaraman. As per the recommendations
of the committee NABARD was established.

560-570 Questions :

1. When was the currency system in India converted into decimal


system?
A. April 1 1959
B. April 1 1957
C. April 1 1955
D. April 1 1953

Answer & Explanation

B. April 1 1957
Explanation:
The Indian currency system was converted into decimal system by
Indian Coinage (Amendment) Act 1955 which brought into force from
April 1, 1957.

2. Dalal Street in Mumbai is famous for which of the following?


A. NABARD
B. Stock Exchange
C. RBI
D. SEBI

Answer & Explanation

B. Stock Exchange
Explanation:
The Bombay Stock Exchange (BSE) is located at Dalal Street.

www.bankingpdf.com
3. Deposits which can be withdrawn by a customer without notice
is called ________
A. Time Deposits
B. Demand Deposits
C. Variable Deposits
D. Low cost Deposits

Answer & Explanation

B. Demand Deposits
Explanation:
A deposit of money that can be withdrawn without prior notice is
known as Demand Deposits, Ex:- in a current account.

4. Which of the following is not e-banking software?


A. ECS
B. RTGS
C. PIPS
D. IBR

Answer & Explanation

C. PIPS
Explanation:
ECS – Electronic Clearing Service
RTGS – Real Time Gross Settlement
PIPS is not an e-banking software.

5. ICRA, CRISIL, and Standard and Poor’s (S&P) are ______


A. Financial Institutions
B. NGOs
C. NBFCs
D. Credit Rating Agencies

Answer & Explanation

www.bankingpdf.com
D. Credit Rating Agencies
Explanation:
A credit rating agency (CRA, also called a ratings service) is a company
that assigns credit ratings, which rate a debtor’s ability to pay back
debt by making timely interest payments and the likelihood of default.
Example:- ICRA, CRISIL, and Standard and Poor’s (S&P).

6. Which of the following is the feature of IBS?


A. Multi Currency
B. Multi Entity
C. Multi Branch
D. All of the above

Answer & Explanation

D. All of the above


Explanation:
Features of IBS:
Multi-bank, multi-branch, multi-currency, Highly parametric design,
Real time posting and End of Day Processing, Highly efficient Auditing
and Control.

7. The largest number of commercial bank’s offices are located in


______
A. Kerala
B. Karnataka
C. Uttar Pradesh
D. Tamil Nadu

Answer & Explanation

C. Uttar Pradesh
Explanation:
Uttar Pradesh has the largest number of commercial bank’s offices.

www.bankingpdf.com
8. Which of the following is not a member of the World Bank
Group?
A. International Development Association(IDA)
B. International Bank for Reconstruction and Development(IBRD)
C. International Finance Corporation(IFC)
D. Bank of International Settlement(BIS)

Answer & Explanation

D. Bank of International Settlement(BIS)


Explanation:
The World Bank Group is made of the following five organizations:
International Bank for Reconstruction and Development (IBRD),
established in 1945,
International Finance Corporation (IFC), established in 1956,
International Development Association (IDA), established in 1960,
Multilateral Investment Guarantee Agency (MIGA), established in 1988
and
International Centre for Settlement of Investment Disputes (ICSID),
established in 1966.
Where as Bank of International Settlement(BIS)is the Bank for
International Settlements is an international financial institution owned
by central banks which “fosters international monetary and financial
cooperation and serves as a bank for central banks”.

9. Indian Banking Regulation Act was passed in the year


___________
A. 1989
B. 1968
C. 1949
D. 1948

Answer & Explanation

www.bankingpdf.com
C. 1949
Explanation:
The Banking Regulation Act, 1949 is a legislation in India that
regulates all banking firms in India. Initially, the law was applicable
only to banking companies. But, 1965 it was amended to make it
applicable to cooperative banks.

10. Which of the following is an apex institution in the field of


Industrial Finance?
A. Industrial Reconstruction Bank of India
B. Industrial Finance Corporation of India
C. Industrial Development Bank of India
D. Industrial Credit & Investment Corporation of India

Answer & Explanation

C. Industrial Development Bank of India


Explanation:
IDBI is the apex financial institution and besides providing financial
assistance on consortium basis, the major function of coordination
between the various institutions is looked after by the bank. It also
provides refinance facility to the eligible financial institutions including
term loans. The bank sanctions the financial assistance to the industrial
concerns engaged in the manufacture or processing of goods, mining,
transport generation and distribution of power etc. both in private and
public sectors

570-580 Questions :

1. NEFT is an electronic fund transfer system that operates on a


_________ basis which settles transactions in batches.
A. Domain Net Settlement (DNS)
B. Defined Net Settlement (DNS)

www.bankingpdf.com
C. Declared Net Settlement (DNS)
D. Deferred Net Settlement (DNS)

Answer & Explanation

D. Deferred Net Settlement (DNS)


Explanation:
NEFT is an electronic fund transfer system that operates on a Deferred
Net Settlement (DNS) basis which settles transactions in batches. In
DNS, the settlement takes place with all transactions received till the
particular cut-off time. These transactions are netted (payable and
receivables) in NEFT.

2. In National Electronic Funds Transfer (NEFT) maximum amount


per transaction is limited to ________
A. Rs.75000
B. Rs.50000
C. Rs.80000
D. Rs.90000

Answer & Explanation

B. Rs.50000
Explanation:
No. There is no limit – either minimum or maximum – on the amount
of funds that could be transferred using NEFT. However, maximum
amount per transaction is limited to Rs.50,000/- for cash-based
remittances within India and also for remittances to Nepal under the
Indo-Nepal Remittance Facility Scheme.

3. The NEFT system facilitates ____cross-border transfer of funds


from India to Nepal which is known as the Indo-Nepal
Remittance Facility Scheme.
A. One Way
B. Two Way

www.bankingpdf.com
C. Multi way
D. None of the Above

Answer & Explanation

A. One Way
Explanation:
The NEFT system also facilitates one-way cross-border transfer of
funds from India to Nepal. This is known as the Indo-Nepal Remittance
Facility Scheme. A remitter can transfer funds from any of the NEFT-
enabled branches in to Nepal, irrespective of whether the beneficiary in
Nepal maintains an account with a bank branch in Nepal or not. The
beneficiary would receive funds in Nepalese Rupees.

4. What is the charge of Inward transactions at destination bank


branches in NEFT?
A. Rs.1
B. Rs.1.50
C. Rs.2
D. No Charge

Answer & Explanation

D. No Charge
Explanation:
Inward transactions – Free, no charge to be levied.

5. The beneficiary can expect to get credit for the NEFT


transactions within ______business hours.
A. 2
B. 3
C. 1
D. None of the Above

Answer & Explanation

www.bankingpdf.com
A. 2
Explanation:
The beneficiary can expect to get credit for the NEFT transactions
within two business hours (currently NEFT business hours is from
morning 8 AM to evening 7 PM on all week days and from morning 8
AM to afternoon 1 PM on Saturdays) from the batch in which the
transaction was settled.

6. NEFT operates in hourly batches and there are _______


settlements from 8 am to 7 pm on week days.
A. 3
B. 6
C. 8
D. 12

Answer & Explanation

D. 12
Explanation:
NEFT operates in hourly batches – there are twelve settlements from 8
am to 7 pm on week days (Monday through Friday) and six settlements
from 8 am to 1 pm on Saturdays.

7. The first 4 alpha characters in the 11 digit IFSC code represents


________
A. Bank Name
B. Branch Name
C. Place Name
D. None of the Above

Answer & Explanation

A. Bank Name
Explanation:
The Indian Financial System Code (IFS Code or IFSC) is an 11-

www.bankingpdf.com
character code with the first four alphabetic characters representing
the bank name, and the last six characters(usually numeric, but can be
alphabetic) representing the branch. The fifth character is 0 (zero).

8. In NEFT What is the charge for transactions up to Rs 10,000?


A. not exceeding Rs 2.50 (+ Service Tax)
B. not exceeding Rs 5 (+ Service Tax)
C. not exceeding Rs 15 (+ Service Tax)
D. None of the Above

Answer & Explanation

A. not exceeding Rs 2.50 (+ Service Tax)


Explanation:
Outward transactions at originating bank branches – charges applicable
for the remitter
For transactions up to Rs 10,000 : not exceeding Rs 2.50 (+ Service
Tax)

9. In NEFT, What is the charge for transactions above Rs 10,000


upto Rs.1 lakh?
A. not exceeding Rs 2.50 (+ Service Tax)
B. not exceeding Rs 5 (+ Service Tax)
C. not exceeding Rs 15 (+ Service Tax)
D. None of the Above

Answer & Explanation

B. not exceeding Rs 5 (+ Service Tax)


Explanation:
Outward transactions at originating bank branches – charges applicable
for the remitter
For transactions above Rs 10,000 up to Rs 1 lakh: not exceeding Rs 5
(+ Service Tax)

www.bankingpdf.com
10. In NEFT What is the charge for transactions above Rs 1 lakh
and upto Rs.2 lakh?
A. not exceeding Rs 2.50 (+ Service Tax)
B. not exceeding Rs 5 (+ Service Tax)
C. not exceeding Rs 15 (+ Service Tax)
D. None of the Above

Answer & Explanation

C. not exceeding Rs 15 (+ Service Tax)


Explanation:
Outward transactions at originating bank branches – charges applicable
for the remitter
For transactions above Rs 1 lakh and up to Rs 2 lakhs: not exceeding
Rs 15 (+ Service Tax)

580-590 Questions :

1. Which of the following is not a measure of the Risk


Management in Banks?
A. CRR
B. RTGS
C. SLR
D. Deposit Insurance

Answer & Explanation

B. RTGS
Explanation:
Cash reserve Ratio (CRR) is the amount of funds that the banks have
to keep with the RBI. Statutory liquidity ratio (SLR) is the Indian
government term for reserve requirement that the commercial banks
in India require to maintain in the form of gold, government approved

www.bankingpdf.com
securities before providing credit to the customers. Deposit insurance
is mandatory, and pays claims from a pool of funds to which every
depository institution regularly contributes.

2. “The Doing Business Report” is prepared by which of the


following organizations every year?
A. Asian Development Bank(ADB)
B. World Bank(WB)
C. New Development Bank(NDB)
D. World Trade Organization(WTO)

Answer & Explanation

B. World Bank(WB)
Explanation:
The Doing Business Report (DB) is a study elaborated by the World
Bank Group since 2003 every year that is aimed to measure the costs
to firms of business regulations. The study has become one of the
flagship knowledge products of the World Bank Group in the field of
private sector development, and is claimed to have motivated the
design of several regulatory reforms in developing countries.

3. Which of the following cannot be called as a debt instrument as


referred in financial transactions?
A. Certificate of Deposits
B. Bonds
C. Stock
D. Commercial Paper

Answer & Explanation

C. Stock
Explanation:
Debt instruments are assets that require a fixed payment to the

www.bankingpdf.com
holder, usually with interest. Examples of debt instruments include
bonds (government or corporate) and mortgages.

4. Which of the following is not a type of cheque issued by an


individual?
A. Bearer Cheque
B. Crossed Cheque
C. Order Cheque
D. Savings Cheque

Answer & Explanation

D. Savings Cheque
Explanation:
An order cheque can be a bearer cheque if the words or bearer are not
cancelled out.A crossed cheque is a cheque that has been marked to
specify an instruction about the way it is to be redeemed.

5. “World Investment Report” is annually published by ______


A. IBRD
B. WTO
C. IMF
D. UNCTAD

Answer & Explanation

D. UNCTAD
Explanation:
The World Investment Report has been published annually since
1991.Each year´s Report covers the latest trends in foreign direct
investment around the World and analyses in depth one selected topic
related to foreign direct investment and development.

6. An account which remains in operative for an extended period


of time is referred as ________

www.bankingpdf.com
A. DEMAT Account
B. Dormant Account
C. Nominal Account
D. Deposit Account

Answer & Explanation

B. Dormant Account
Explanation:
When there has been no financial activity for a long period of time,
other than posting of interest, an account can be classified as dormant.

7. Scheduled banks are required to keep cash reserve with RBI,


interms of _______
A. Section 24 of the Banking Regulation Act, 1949
B. Section 20 of the Banking Regulation Act, 1949
C. Section 42(i) of the RBI Act, 1934
D. Section 42 of Negotiable Instruments Act

Answer & Explanation

C. Section 42(i) of the RBI Act, 1934


Explanation:
Every bank included in the Second Schedule shall maintain with the
Bank an average daily balance the amount.

8. Treasury bills are issued in India by ______


A. RBI
B. State Government
C. Government of India
D. SEBI

Answer & Explanation

C. Government of India
Explanation:

www.bankingpdf.com
Treasury bills (T-bills) offer short-term investment opportunities,
generally up to one year. They are thus useful in managing short-term
liquidity. At present, the Government of India issues three types of
treasury bills through auctions, namely, 91-day, 182-day and 364-day.
There are no treasury bills issued by State Governments.

9. Banking Code & Standards Board of India(BCSBI) set up by


________
A. SEBI
B. Ministry of Finance
C. RBI
D. GOI

Answer & Explanation

C. RBI
Explanation:
The Banking Codes and Standards Board of India (BCSBI) is an
independent banking industry watchdog that protects consumers of
banking services in India. The board oversee compliance with the
“Code of Bank’s Commitment to Customers”

10. The law regarding negotiable instruments is contained in


__________
A. The Bill of exchange Act 1881
B. The Banking Regulation Act 1949
C. Cheques Act, 1881
D. The Negotiable Instruments Act, 1881

Answer & Explanation

D. The Negotiable Instruments Act, 1881


Explanation:
The Negotiable Instruments Act, 1881 – An Act to define and Law

www.bankingpdf.com
relating to negotiable instruments which are Promissory Notes, Bills of
Exchange and cheques.

590-600 Questions :

1. Which of the following is a special type of credit card with


stored money value?
A. Preset Value Card
B. Stored Value Card
C. Both(A) & (B)
D. None of the Above

Answer & Explanation

B. Stored Value Card


Explanation:
A stored-value card is a payments card with a monetary value stored
on the card itself, not in an external account maintained by a financial
institution. Stored-value cards differ from debit cards, where money is
on deposit with the issuer, and credit cards which are subject to credit
limits set by the issuer.

2. A financial product which provides senior citizens with funds


against their home equity is referred as _______
A. Mortgage
B. Reverse Mortgage
C. Repossession
D. Refinance

Answer & Explanation

B. Reverse Mortgage
Explanation:

www.bankingpdf.com
A reverse mortgage or home equity conversion mortgage (HECM) is a
special type of home loan for older home owners (62 years or older)
that requires no monthly mortgage payments. Borrowers are still
responsible for property taxes and home owner’s insurance.

3. ________ is a cheque rather an amount of cheque, which is


above the balance available in the account of the payer.
A. Travellers cheque
B. Multicity Cheque
C. Mutilated Cheque
D. Over Draft

Answer & Explanation

D. Over Draft
Explanation:
An overdraft is an extension of credit from a lending institution when
an account reaches zero. An overdraft allows the individual to continue
withdrawing money even if the account has no funds in it.

4. Which of the following term refers the illegal escape from the
tax payment?
A. Tax Break
B. Tax Evasion
C. Tax Shifting
D. None of the Above

Answer & Explanation

B. Tax Evasion
Explanation:
Tax evasion is an illegal practice where a person, organization or
corporation intentionally avoids paying his/her/its true tax liability.
Those caught evading taxes are generally subject to criminal charges
and substantial penalties.

www.bankingpdf.com
5. A situation which is providing some relief from tax in whole or
part is referred as _________
A. Tax Break
B. Tax Evasion
C. Tax Shifting
D. None of the Above

Answer & Explanation

A. Tax Break
Explanation:
Tax break is a term referring to any item which avoids taxes, including
any tax exemption, tax deduction, or tax credit.

6. A type of market in which there is a relatively high degree of


concentration is referred as _____
A. Mortgage Refinance
B. Oligopoly
C. Near Money
D. Open Economy

Answer & Explanation

B. Oligopoly
Explanation:
Large number of potential buyers but only a few sellers is known as
Oligopoly.

7. Which of the following cheque issued by a customer under a


pre-approved arrangement with the bank?
A. Travellers cheque
B. Multi-city Cheque
C. Mutilated Cheque
D. Crossed Cheque

www.bankingpdf.com
Answer & Explanation

B. Multi-city Cheque
Explanation:
A Multi-City Cheque (MCC) is one that can be written by the customer
in favour of his client and is payable at par at all branches of the Bank.
These are issued as Order Cheque.

8. A cheque for a fixed amount that may be cashed or used in


payment abroad after endorsement by the holder’s signature is
known as ___________
A. Travellers cheque
B. Multicity Cheque
C. Mutilated Cheque
D. Crossed Cheque

Answer & Explanation

A. Travellers cheque
Explanation:
Traveller’s cheque is a medium of exchange that can be used in place
of hard currency. Traveller’s cheques are often used by individuals
travelling on vacation to foreign countries.

9. A commission or fee paid, when an agent or an individual sells


an investment such as mutual funds or annuity is known as
_________
A. Redemption fee
B. Service Charge
C. Regressive tax
D. Sales tax

Answer & Explanation

www.bankingpdf.com
A. Redemption fee
Explanation:
A mutual fund redemption fee, also referred to as a “market timing
fee”, or “short-term trading fee”, is a charge by a mutual fund
company to discourage investors from making a short-term “round
trip” (i.e. a purchase, typically a transfer, followed by a sale within a
short period of time).

10. A legal agreement between the lender and borrower where


real estate property is used as a collateral for the loan is
termed as ________
A. Mortgage
B. Reverse Mortgage
C. Repossession
D. Refinance

Answer & Explanation

A. Mortgage
Explanation:
A loan that is secured by property or real estate is called a mortgage.
In exchange for funds received by the homebuyer to buy property or a
home, a lender gets the promise of that buyer to pay back the funds
within a certain time frame for a certain cost.

600-610 Questions :

1. Time period that is considered from the inception of the credit,


investment or negotiable instrument and ends upon the
maturity or expiry of the instrument is referred as ________
A. Amortisation Period
B. Amortising Swap

www.bankingpdf.com
C. Asset Backed Security
D. Attrition Analysis

Answer & Explanation

A. Amortisation Period
Explanation:
The amortisation period on a mortgage is the total length of time it will
take you to pay off your mortgage.

2. A debt which is irrecoverable and is therefore written off as loss


in the accounts of an institution or bank is known as
__________
A. external debt
B. good debt
C. bad debt
D. internal debt

Answer & Explanation

C. bad debt
Explanation:
The term bad debts usually refers to accounts receivable (or trade
accounts receivable) that will not be collected.

3. ___________ is basically charged when a person uses a credit


card to obtain cash.
A. Early Repayment Charge
B. Redemption Fee
C. Transaction Fee
D. Cash Advance Fee

Answer & Explanation

D. Cash Advance Fee


Explanation:

www.bankingpdf.com
A cash advance fee is a charge that a credit card issuer charges a
customer for accessing the cash credit line on his or her account, either
through an ATM, convenience check or at a bank’s teller window.

4. Which is an interest-free period that is to be given by a creditor


to debtor?
A. Fixed Period
B. Preset Period
C. Grace Period
D. None of the Above

Answer & Explanation

C. Grace Period
Explanation:
The grace period is the provision in most loan and insurance contracts
which allows payment to be received for a certain period of time after
the actual due date. During this period no late fees will be charged,
and the late payment will not result in default or cancellation of the
loan.

5. An account which is maintained by depositing undistributed


parts of profit for future needs is known as ________
A. Savings Account
B. Current Account
C. Reserve Account
D. None of the Above

Answer & Explanation

C. Reserve Account
Explanation:
Funds taken out of earnings to provide for anticipated future payments
is called Reserve Account.

www.bankingpdf.com
6. The difference between the total cost of a project and the
sanctioned loan amount is termed as ___________
A. Near Money
B. Hot Money
C. Gross Income
D. Margin Amount

Answer & Explanation

D. Margin Amount
Explanation:
The Difference between the market value of a collateral and amount of
the loan advanced against it. Margin Amount is also known as haircut.

7. A loan which is secured by collateral and for which borrower is


not personally liable is called __________
A. Secured loan
B. Syndicated loan
C. Non-Recourse loan
D. No Documentation loan

Answer & Explanation

C. Non-Recourse loan
Explanation:
A non-recourse debt is a type of loan that is secured by collateral,
which is usually property. If the borrower defaults, the issuer can seize
the collateral, but cannot seek out the borrower for any further
compensation, even if the collateral does not cover the full value of the
defaulted amount.

8. Which term refers closure of the loan account prior to the


tenure fixed for the account?
A. Pre Approved
B. Pre Closure

www.bankingpdf.com
C. Personal Disposable
D. None of the Above

Answer & Explanation

B. Pre Closure
Explanation:
Prepayment or pre-closure of a loan implies to the early repayment of
a loan, in part or in full. Prepayment is done to reduce the tenure of
the loan as well as the interest.

9. Taking back of property by a seller or a lender from the buyer


or borrower due to default of payment is termed as ________
A. Reconveyance
B. Repossession
C. Recurring Billing
D. Revalidation

Answer & Explanation

B. Repossession
Explanation:
Repossession is a term used to denote a financial institution taking
back an object that was either used as collateral or rented or leased.

10. A very large loan extended by a group of small banks to a


single borrower is known as ______
A. Secured loan
B. Syndicated loan
C. Non-Recourse loan
D. No Documentation loan

Answer & Explanation

B. Syndicated loan
Explanation:

www.bankingpdf.com
A syndicated loan is a loan offered by a group of lenders (called a
syndicate) who work together to provide funds for a single borrower.
The borrower could be a corporation, a large project, or a sovereignty
(such as a government). The loan may involve fixed amounts, a credit
line, or a combination of the two.

610-620 Questions :

1. Cash kept in the currency chest is owned by?


A. Currency Chest branch bank
B. State Bank of India
C. Central Government
D. Reserve Bank of India

Answer & Explanation

D. Reserve Bank of India


Explanation:
The amount of cash kept in the currency chest belongs to the RBI and
is used for maintaining the Cash Reserve Ratio. To facilitate the
distribution of banknotes and rupee coins, the Reserve Bank has
authorised select branches of scheduled banks to establish currency
chests. These are actually storehouses where banknotes and rupee
coins are stocked on behalf of the Reserve Bank.

2. What is the minimum amount of deposit into/withdrawal from


currency chest?
A. Rs.1,00,000
B. Rs.1,50,000
C. Rs.2,00,000
D. Rs.3,00,000

Answer & Explanation

www.bankingpdf.com
A. Rs.1,00,000
Explanation:
The minimum amount of deposit into/withdrawal from currency chest
will be Rs.1,00,000/- and thereafter, in multiples of Rs.50,000.

3. When more than one bank is allowing credit facilities to one


party in coordination with each other under a formal
arrangement, the arrangement is generally known
as___________
A. Consortium
B. Syndication
C. Multiple Banking
D. Participation

Answer & Explanation

A. Consortium
Explanation:
A banking syndicate formed by multiple banks, often from different
countries, for the singular purpose of financing a specific project that is
too large for any individual bank to finance on its own. Under this
arrangement participating banks acquire a common interest and share
equally in the risk and the profits. Upon completion of the project the
consortium bank is disbanded.

4. What is the maximum period for which a term deposit can be


normally opened?
A. 8 years
B. 7 years
C. 9 years
D. 10 years

Answer & Explanation

www.bankingpdf.com
D. 10 years
Explanation:
The tenure of an Fixed Deposit or Term Deposit can vary from 7, 15 or
45 days to 1.5 years and can be as high as 10 years.

5. _________ are safer than Post Office Schemes as that are


covered by the Deposit Insurance and Credit Guarantee
Corporation(DICGC)
A. Fixed Deposit
B. Demand Deposit
C. Flexi-Fixed Deposit
D. All of the Above

Answer & Explanation

A. Fixed Deposit
Explanation:
Fixed Deposit investments are safer than Post Office Schemes as they
are covered by the Deposit Insurance and Credit Guarantee
Corporation (DICGC).

6. DICGC guarantees amount up to _____________ per depositor


per bank.
A. Rs.1,00,000
B. Rs.1,50,000
C. Rs.2,00,000
D. Rs.3,00,000

Answer & Explanation

A. Rs.1,00,000
Explanation:
DICGC guarantees amount up to Rs. 1,00,000 per depositor per bank.
They also offer income tax and wealth tax benefits.

www.bankingpdf.com
7. Which of the following institutions do not hold a banking
license?
A. Small Banks
B. Foreign Banks
C. Commercial Banks
D. NBFCs

Answer & Explanation

D. NBFCs
Explanation:
Non-banking financial companies, or NBFCs, are financial institutions
that provide banking services, but do not hold a banking license. These
institutions are not allowed to take deposits from the public.

8. What is the full form of FATCA?


A. Forex Account Tax Compliance Act
B. Foreign Account Tax Compliance Act
C. Forex Account Total Compliance Act
D. Foreign Account Total Compliance Act

Answer & Explanation

B. Foreign Account Tax Compliance Act


Explanation:
The Foreign Account Tax Compliance Act (FATCA) is a 2010 United
States federal law to enforce the requirement for United States persons
including those living outside the U.S. to file yearly reports on their
non-U.S. financial accounts to the Financial Crimes Enforcement
Network (FINCEN).

9. What is the minimum initial deposit required for opening a


Basic Savings Bank Deposit Account(BSBDA)?
A. 500
B. 1000

www.bankingpdf.com
C. 100
D. No initial deposit

Answer & Explanation

D. No initial deposit
Explanation:
There is no requirement for any initial deposit for opening a BSBDA.

10. Who among the following decides the volume and value of
bank notes to be printed in India?
A. RBI
B. SBI
C. Security Printing and Minting Corporation of India
D. All of the Above

Answer & Explanation

A. RBI
Explanation:
The Reserve Bank based on the demand requirement indicates the
volume and value of banknotes to be printed each year to the
Government of India which get finalised after mutual consultation. The
quantum of banknotes to be printed, broadly depends on the
requirement for meeting the demand for banknotes, GDP growth,
replacement of soiled banknotes, reserve stock requirements, etc.

620-630 Questions :

1. An ECS transaction gets bounced and you are unable to recover


your money from your customer. Under which Act criminal
action can be initiated?
A. Negotiable Instrument Act 1881

www.bankingpdf.com
B. Criminal Procedure Code
C. Payment and Settlement Systems Act 2007
D. Indian Companies Act 1956

Answer & Explanation

C. Payment and Settlement Systems Act 2007


Explanation:
Under the Section 25 of Payment and Settlement Systems (PSS) Act,
2007, dishonour of an electronic fund transfer instruction due to
insufficiency of funds in the account etc., is an offence punishable with
imprisonment or with fine or both, similar to the dishonour of a cheque
under the Negotiable Instruments Act 1881.

2. In which of the following fund transfer mechanisms, can funds


be moved from one bank to another and where the transaction
is settled instantly without being bunched with any other
transaction?
A. RTGS
B. NEFT
C. TT
D. EFT

Answer & Explanation

A. RTGS
Explanation:
In RTGS, the beneficiary branches are expected to receive the funds in
real time as soon as funds are transferred by the remitting bank. The
beneficiary bank has to credit the beneficiary’s account within 30
minutes of receiving the funds transfer message.

3. A worldwide financial messaging network which exchanges


messages between banks and financial institutions is known as
_________

www.bankingpdf.com
A. SWIFT
B. Basel
C. RTGS
D. NEFT

Answer & Explanation

A. SWIFT
Explanation:
Society for Worldwide Inter bank Financial Telecommunication code. An
internationally-recognized identification code for banks around the
world. SWIFT codes are most commonly used for international wire
transfers and are comprised of 8 or 11 alphanumeric characters.

4. Which of the following is not a ‘Money Market Instrument’?


A. Treasury Bills
B. Commercial Paper
C. Certificate of Deposit
D. Equity Shares

Answer & Explanation

D. Equity Shares
Explanation:
There are several money market instruments, including treasury bills,
commercial paper, bankers’ acceptances, deposits, certificates of
deposit, bills of exchange, repurchase agreements, federal funds, and
short-lived mortgage-, and asset-backed securities.

5. When there is a difference between all receipts and expenditure


of the Govt. of India, both capital and revenue it is called
_______
A. Income Deficit
B. Fiscal Deficit

www.bankingpdf.com
C. Budgetary Deficit
D. None of the Above

Answer & Explanation

C. Budgetary Deficit
Explanation:
A budget deficit occurs whenever a government spends more than it
makes, which is nearly every year.Budgetary deficit is the difference
between all receipts and expenses in both revenue and capital account
of the government.

6. With reference to a cheque which of the following is the


‘drawee bank’ ?
A. The bank that collects the cheque
B. The payee’s bank
C. The endorser’s bank
D. The bank upon which the cheque is drawn

Answer & Explanation

D. The bank upon which the cheque is drawn


Explanation:
Drawee is a legal and banking term used to describe the party that has
been directed by the depositor to pay a certain sum of money to the
person presenting the check or draft. A typical example is if you are
cashing a paycheck.

7. Banks issue a letter to beneficiary on behalf of its constituents


like guarantee for making payment on their behalf on fulfilment
of its terms and conditions. What is this arrangement known in
banking context?
A. Line of Credit
B. Loan to Client

www.bankingpdf.com
C. Loan on Credit
D. Letter of Credit

Answer & Explanation

D. Letter of Credit
Explanation:
A letter issued by a bank to another bank (especially one in a different
country) to serve as a guarantee for payments made to a specified
person under specified conditions.

8. When a Bank provides a loan for purchase of white goods, it is


categorised as _____________
A. Consumption loan
B. White Goods loan
C. Consumer Durable loan
D. Working Capital

Answer & Explanation

C. Consumer Durable loan


Explanation:
Consumer Durable loan is a finance option for purchase of (White
Goods) household items like Washing Machines, Refrigerators, AC,
LED, LCD, Microwaves etc.

9. Fixed Deposits and Recurring Deposits are _________


A. repayable after an agreed period.
B. repayable on demand.
C. not repayable.
D. repayable on demand or after an agreed period as per bank’s
choice.

Answer & Explanation

www.bankingpdf.com
D. repayable on demand or after an agreed period as per bank’s
choice.
Explanation:
A fixed deposit (FD) is a financial instrument provided by banks which
provides investors with a higher rate of interest than a regular savings
account, until the given maturity date.And Recurring Deposit is a
special kind of Term Deposit offered by banks in India which help
people with regular incomes to deposit a fixed amount every month
into their Recurring Deposit account and earn interest at the rate
applicable to Fixed Deposits.

10. What is the fullform of ‘FSDC’ which is used in financial


sectors?
A. Financial Security and Development Council
B. Financial Stability and Development Council
C. Fiscal Security and Development Council
D. Fiscal Stability and Development Council

Answer & Explanation

B. Financial Stability and Development Council


Explanation:
Financial Stability and Development Council is an apex-level body
constituted by the government of India.
Chairperson: The Union Finance Minister of India
Members: Governor Reserve Bank of India (RBl), Finance Secretary
and/ or Secretary, Department of Economic Affairs (DEA), Secretary,
Department of Financial Services (DFS), Chief Economic Advisor,
Ministry of Finance, Chairman, Securities and Exchange Board of India
(SEBI), Chairman, Insurance Regulatory and Development Authority
(IRDA), Chairman, Pension Fund Regulatory and Development
Authority (PFRDA), Chairman, Forward Markets Commission
(FMC), Additional Secretary, Ministry of Finance, DEA, will be the
Secretary of the Council.

www.bankingpdf.com
630-640 Questions :

1. A provision in some loans that allows the borrower to change


the interest rate from fixed to variable or Vice Versa is termed
as __________
A. Convertible Bond
B. Convertible Clause
C. Convertible Security
D. None of the Above

Answer & Explanation

B. Convertible Clause
Explanation:
A provision that can be found on some bonds allowing the bondholder
to exchange their debt into common stock.

2. CRAR stands for _________


A. Capital to Risk-Weighted Assets Ratio
B. Capital to Risk Assets Ratio
C. Credit Rating-Weighted Assets Ratio
D. Credit Rating Assets Ratio

Answer & Explanation

A. Capital to Risk-Weighted Assets Ratio


Explanation:
The Capital Adequacy Ratio (CAR) or Capital-to-Risk weighted Assets
Ratio (CRAR) is a measure of a bank’s capital. It is expressed as a
percentage of a bank’s risk weighted credit exposures. It is used to
protect depositors and promote the stability and efficiency of financial
systems around the world.

www.bankingpdf.com
3. An index that is used to determine interest rates and/or
changes of interest rates of certain types of loans is known as
_________
A. Covenant
B. CRAR
C. COFI
D. CRAs

Answer & Explanation

C. COFI
Explanation:
A cost of funds index or COFI is a regional average of interest expenses
incurred by financial institutions, which in turn is used as a base for
calculating variable rate loans.

4. _________ are long term corporate bonds that are unsecured


in nature.
A. DCF
B. Debentures
C. Covenant
D. CRAs

Answer & Explanation

B. Debentures
Explanation:
A long-term security yielding a fixed rate of interest, issued by a
company and secured against assets are known as debentures.

5. A type of loan, where the bank or the lending institution


provides the borrower with a loan that helps the borrower to
pay off his all previous debts is termed as ________
A. Debt Recovery
B. Debt Settlement

www.bankingpdf.com
C. Debt Management
D. Debt Consolidation

Answer & Explanation

D. Debt Consolidation
Explanation:
Debt consolidation is a form of debt refinancing that entails taking out
one loan to pay off many others.

6. An account for which a bank acts as an uninterested third party


is termed as _______
A. Savings Account
B. Current Account
C. Reserve Account
D. Escrow Account

Answer & Explanation

D. Escrow Account
Explanation:
An escrow account is a temporary pass through account held by a third
party during the process of a transaction between two parties.
Definition: An escrow account is a temporary pass through account
held by a third party during the process of a transaction between two
parties.

7. The unpaid principal balance of a loan on property divided by


the asset’s appraised value is termed as ______
A. Liquidity Adjustment Facility
B. Loss Given Default(LGD)
C. Loan to Value(LTV)
D. Long term liabilities

Answer & Explanation

www.bankingpdf.com
C. Loan to Value(LTV)
Explanation:
The loan-to-value (LTV) ratio is a financial term used by lenders to
express the ratio of a loan to the value of an asset purchased. The
term is commonly used by banks and building societies to represent
the ratio of the first mortgage lien as a percentage of the total
appraised value of real property.

8. A signed undertaking from one party containing a promise to


pay a stated sum to a specified person or a company is known
as ________
A. Power of Attorney
B. Promissory Note
C. Purchasing Power Parity
D. None of the Above

Answer & Explanation

B. Promissory Note
Explanation:
A promissory note is a legal instrument in which one party (the maker
or issuer) promises in writing to pay a determinate sum of money to
the other (the payee)

9. __________ is a transfer of property to its real owner, once the


loan or mortgage is paid off.
A. Reconveyance
B. Repossession
C. Recurring Billing
D. Revalidation

Answer & Explanation

A. Reconveyance
Explanation:

www.bankingpdf.com
The transfer of real property that takes place when a mortgage is fully
paid off and the land is returned to the owner free from the former
debt.

10. Which of the following is defined as the difference between


current assets and current liabilities?
A. Venture Capital
B. Working Capital
C. Equitable Mortgage
D. None of the Above

Answer & Explanation

B. Working Capital
Explanation:
The capital of a business which is used in its day-to-day trading
operations, calculated as the current assets minus the current
liabilities.

640-650 Questions :

1. Which of the following is a financial ratio which gives an idea or


a measure of a company’s ability to meet its financial losses.
A. Credit Ratio
B. Interest Ratio
C. Leverage Ratio
D. None of these

Answer & Explanation

C. Leverage Ratio
Explanation: The financial leverage ratio measure the overall debt load
of a company and compare it with the assets or equity. This shows how

www.bankingpdf.com
much of the company assets belong to the shareholders rather than
creditors.

2. Security which one offers for taking an advance or loan from


someone is known as _______.
A. Collateral
B. Mortgage
C. Pledge
D. None of these

Answer & Explanation

B. Mortgage
Explanation: It is a way for a smaller regional bank to lend mortgages
to its customers without having to worry about whether the customers
have the assets to cover the loan. The bank acts as a middleman
between the home buyer and the investment markets.

3. When financial institutions and banks undertake activities


related to banking like investment, issue of debit and credit card
etc. it is called as _________
A. Retail Banking
B. Offshore Banking
C. Virtual Banking
D. Universal Banking

Answer & Explanation

D. Universal Banking
Explanation: A universal bank is a bank which offers commercial bank
functions plus other functions such as Merchant Banking, Mutual Funds,
Factoring, Credit cards, Housing Finance, Auto loans, Retail loans,
Insurance, etc.

www.bankingpdf.com
4. A bond that is sold at good discount as it has no coupon is
A. Premium Bond
B. One-coupon Bond
C. Zero coupon Bond
D. None of these

Answer & Explanation

C. Zero coupon Bond


Explanation: A zero-coupon bond is a bond that makes no periodic
interest payments and is sold at a deep discount from face value. The
buyer of the bond receives a return by the gradual appreciation of the
security, which is redeemed at face value on a specified maturity date.

5. A right of the creditors to retain possession of all goods given in


security to him by the debtor for any outstanding debt is
________.
A. General Lien
B. Particular Lien
C. Specific Lien
D. None of these

Answer & Explanation

A. General Lien
Explanation: A General Lien is a right to retain the goods and securities
belonging to the borrower for all dues payable by him. The creditor has
no right to sell the goods even though he retains possession thereof. The
creditor may only retain the goods/property till the realization of the
debt.

6. Where the name of the endorsee or transferee appears on the


instrument while making endorsement.
A. Endorsement in Full
B. Blank Endorsement

www.bankingpdf.com
C. Conditional Endorsement
D. Partial Endorsement

Answer & Explanation

A. Endorsement in Full
Explanation: Special or full endorsement is that which contains not only
the name of the endorser but also the name of the endorse. The effect
of special endorsement is that the endorse must endorse it again if he
wants to transfer the property in the cheque to somebody else.

7. A type of banking in which various types of financial services like


accepting bills arising out of trade, arranging and providing
underwriting, providing advice, information or assistance on
starting new business, acquisitions, mergers etc. are provided is
called as
A. Offshore Banking
B. Investment Banking
C. Merchant Banking
D. Commercial Banking

Answer & Explanation

C. Merchant Banking
Explanation: Merchant Banking is a combination of Banking and
consultancy services. It provides consultancy to its clients for financial,
marketing, managerial and legal matters. Consultancy means to provide
advice, guidance and service for a fee.

8. Who imposes moratorium on operations of a bank if the affairs


of the bank are not conducted as per banking norms.
A. SEBI
B. RBI
C. IRDA
D. NABARD

www.bankingpdf.com
Answer & Explanation

B. RBI
Explanation:It is a temporary delay. For instance a delay in the
payment of debt, if too many people are unable to repay loans, the
government may declare that no one is legally obligated to make debt
service payments for a period of six months. Similarly, if a company is
having a difficult year, it may declare a moratorium on research and
development funding for two years in order to save money.

9. It focuses on providing for financial needs of industry and


institutional clients.
A. Retail Banking
B. Narrow Banking
C. Offshore Banking
D. Wholesale Banking

Answer & Explanation

D. Wholesale Banking
Explanation: Wholesale Banking is in which banking services are
offered only to government agencies, pension funds, other institutional
customers and to corporations with strong balance sheets and sound
income statements.

10. The dilution or selling of the government stake in public


sector undertakings is known as
A. Privatization
B. Investment
C. Disinvestment
D. None of these

Answer & Explanation

www.bankingpdf.com
C. Disinvestment
Explanation:Disinvestment means to sell off certain assets such as a
manufacturing plant, a division or subsidiary, or product line.

650-660 Questions :

1. A bond giving the investor the option to convert the bond into
equity at a fixed conversion price is ________.
A. Coupon Bonds
B. Zero coupon Bond
C. Convertible Bond
D. None of these

Answer & Explanation

C. Convertible Bond
Explanation:Convertible bonds are issued by corporations. Because
they can be converted into a predetermined number of stock shares at
certain points of the bond’s life. The investor typically gets a lower rate
of return but can trade the bond for stock rather than be repaid the
cost of the bond.

2. Which of the following appoints Banking ombudsman?


A. State Government
B. Union government
C. RBI
D. SBI

Answer & Explanation

C. RBI
Explanation:The Banking Ombudsman is appointed by the Reserve
Bank of India to redress customer complaints against deficiency in

www.bankingpdf.com
certain banking services.The Banking Ombudsman Scheme enables an
inexpensive forum to bank customers for resolution of complaints
relating to certain services rendered by banks.

3. Treasury bills are issued by _________.


A. Commercial Banks
B. State government
C. EXIM Bank
D. RBI

Answer & Explanation

D. RBI
Explanation:Treasury bills (T-bills) offer short-term investment
opportunities, generally up to one year. They are thus useful in
managing short-term liquidity. At present, the Government of India
issues three types of treasury bills through auctions, namely, 91-day,
182-day and 364-day. There are no treasury bills issued by State
Governments.

4. Which of the following does not include Priority Sector Lending?


A. Micro and Small Enterprises
B. Education
C. Housing
D. Insurance

Answer & Explanation

D. Insurance
Explanation:Priority sector refers to those sectors of the economy
which may not get timely and adequate credit in the absence of this
special dispensation. These are small value loans to farmers for
agriculture and allied activities, micro and small enterprises, poor
people for housing, students for education and other low income

www.bankingpdf.com
groups and weaker sections. Priority Sector includes Agriculture, Micro
and Small Enterprises, Education, Housing, Export Credit & Others.

5. CORE stand for _________.


A. Centre Online Real-time Exchange
B. Centralized Online Real-time Exchange
C. Computerised Online Real-time Exchange
D. Centralized Online Real-time Export

Answer & Explanation

B. Centralized Online Real-time Exchange


Explanation:Centralized Online Real-time Exchange means that all the
bank’s branches access applications from centralized data centers. The
deposits made are reflected immediately on the bank’s servers and the
customer can withdraw the deposited money from any of the bank’s
branches throughout the world. These applications now also have the
capability to address the needs of corporate customers, providing a
comprehensive banking solution.

6. What is Multi city cheque?


A. Multi cheques paid
B. payable in any bank
C. payable in any branch of a particular bank
D. None of these

Answer & Explanation

C. payable in any branch of a particular bank


Explanation:Multi City Cheque or MCC is a facility wherein the
customer can issue cheques drawn at the base branch and payable at
any branch at remote centre.

7. If you are taking a housing loan by depositing your house


documents by mortgaging your house it is known as

www.bankingpdf.com
________________.
A. Mortgage
B. Pledge
C. Collateral
D. None of these

Answer & Explanation

B. Pledge
Explanation:A pledge is when the property is offered as collateral or
security but that there isn’t an active lien on it. It is a right to reserve a
legal interest in something. e.g. many banks and credit unions have
what is called cross collateral So for instance, if you have a vehicle loan
with a bank and also have a checking account with the bank, there is
an excellent chance that you’ve signed a contract provision where
you’ve pledged whatever funds you may have in your checking account
from time to time as additional security on the loan.

8. Which of the following is not related to bill of exchange?


A. Endorser
B. Payee
C. Drawer
D. Drawee

Answer & Explanation

A. Endorser
Explanation:There are three entities that involved with a bill of
exchange transaction. They are: Drawee. This party pays the amount
stated on the bill of exchange to the payee. Drawer. This party requires
the drawee to pay a third party (or the drawer can be paid by the
drawee). Payee. This party is paid the amount specified on the bill of
exchange by the drawee.

www.bankingpdf.com
9. Coins in India are minted at
A. Noida
B. Kolkata
C. A & B
D. None of these

Answer & Explanation

C. A & B
Explanation:There are four mints in India that produce coins for use
of the public. These are: India Government Mint, Mumbai
(Maharashtra), India Government Mint, Kolkata (West Bengal), India
Government Mint, Hyderabad (Andhra Pradesh), India Government
Mint, NOIDA (Uttar Pradesh). Apart from minting coins, the mints at
Mumbai, Kolkata and Hyderabad also make coin blanks.

10. One percent of shareholding in NABARD is by


A. Union Government
B. RBI
C. NHB
D. None of these

Answer & Explanation

B. RBI
Explanation:NABARD was established on 12 July 1982 by a special
Act of parliament and its main focus was on upliftment of rural India by
increasing the credit flow for elevation of agriculture & rural non farm
sector.

660-670 Questions :

www.bankingpdf.com
670-680 Questions :

1. Scheduled banks are listed on which schedule of the Reserve


Bank of India Act 1934?
A. First
B. Second
C. Third
D. None of these

Answer & Explanation

B. Second
Explanation: A scheduled bank is a bank which is listed in the 2nd
Schedule of the Reserve Bank of India Act, 1934. Banks not under this
Schedule are called non-scheduled banks. Scheduled banks comprised
of private, foreign and nationalised banks operating in India. A
scheduled bank is eligible for loans from the Reserve Bank of India at
bank rate.

2. Maximum note printable by RBI are


A. Rs.5000
B. Rs.1000
C. Rs.10000
D. Rs.50000

Answer & Explanation

C. Rs.10000
Explanation: Currently notes of Rs 10,Rs 100,Rs 500 and Rs 1,000
are only printed. However, the RBI has the powers to print currency
notes of up to Rs 10,000 denomination. But, an amendment to the
Reserve Bank of India Act, 1934 will be needed if any note of higher
denomination has to be printed.

www.bankingpdf.com
3. CRR must maintain on daily basis at what percent of NDTL?
A. 10%
B. 5%
C. 8%
D. 4%

Answer & Explanation

D.4%
Explanation: Cash Reserve Ratio is the fraction of the total Net
Demand and Time Liabilities(NDTL) of a Scheduled Commercial Bank
held in India, that it has to maintain as cash deposit with RBI. The
requirement applies uniformly to all banks in the country irrespective
of an individual bank’s financial situation or size.

4. In respect of bills purchased or discounted if the bill remain


overdue for more than 90 days then it is called ___________
A. NPA
B. NPL
C. NPM
D. OPA

Answer & Explanation

A. NPA
Explanation: The assets of the banks that don’t bring any return are
called Non Performing Assets(NPA). Bank’s assets are the loans and
advances given to customers. If customers don’t pay either interest or
part of principal or both, the loan turns into bad loan.

5. Loans in which initially the rate of interest is low as years go on


they will increase the rate are known as ________
A. Appliance Loan
B. Corporate Loan

www.bankingpdf.com
C. Teaser Loan
D. None of these

Answer & Explanation

C. Teaser Loan
Explanation: A Teaser loan is a special loan that is offered for a fixed
duration and could then be withdrawn. In India the State Bank of India
had two Teaser home Loans including the SBI Easy Home Loan and SBI
Advantage Home Loan, but, they were later withdrawn. These loans
offered a lower interest rate for the first three years and then the
normal rate.

6. These are assets which cannot be seen, touch are measured


physical.
A. Intangible Assets
B. Fictitious Assets
C. Tangible Assets
D. Goodwill

Answer & Explanation

A. Intangible Assets
Explanation:An intangible asset is an asset that you cannot touch.
Examples of intangible assets include copyrights, patents, mailing
lists, trademarks, brand names, domain names, and so on.

7. What is the maximum and minimum limit of NEFT?


A. Rs.2 lakh
B. Rs.10000
C. Rs.2000
D. No limit

Answer & Explanation

www.bankingpdf.com
D. No limit
Explanation:National Electronic Funds Transfer (NEFT) NEFT is
electronic funds transfer system, which facilitates transfer of funds to
other bank accounts. Customers can remit any amount using NEFT
Customer intending to remit money through NEFT has to furnish the
following particulars:IFSC (Indian Financial System Code) of the
beneficiary Bank/Branch, Full account number of the beneficiary, Name
of the beneficiary. The facility is also available through online mode for
all internet banking and mobile banking customers.

8. If financial assets are more than 50% of its total assets then
that company is treated as _______
A. NBFI
B. NBFC
C. Payment Banks
D. Bank

Answer & Explanation

B. NBFC
Explanation:A Non-Banking Financial Company(NBFC) is a company
registered under the Companies Act, 1956 engaged in the business of
loans and advances, acquisition of shares/stocks issued by Government
or local authority of a like nature, leasing, hire-purchase, insurance
business, chit business but does not include any institution whose
principal business is that of agriculture activity, industrial activity,
purchase or sale of any goods or providing any services and
sale/purchase/construction of immovable property.

9. Which act applies to all loans up to Rs.1 lakh and above?


A. SARFAESI Act
B. BIFR
C. DRT
D. SICA

www.bankingpdf.com
Answer & Explanation

A. SARFAESI Act
Explanation:SARFAESI Act is Securitisation and Reconstruction of
Financial Assets and Enforcement of Security Interest Act, 2002. Banks
utilize this act as an effective tool for NPA recovery. It is possible
where non-performing assets are backed by securities charged to the
Bank by way of hypothecation or mortgage or assignment.

10. This money is borrowed or lent for period between 2 days


and 14 days.
A. None
B. Term Money
C. Call money
D. Notice Money

Answer & Explanation

D. Notice Money
Explanation:Call Money, Notice Money and Term Money refer to the
markets for very short term funds. Call Money for the borrowing or
lending of funds for 1 day. Notice Money for the borrowing and lending
of funds for 2-14 days. Term money for borrowing and lending of funds
for a period of more than 14 days. Notice Money is also known as
Short Notice Money.

680-690 Questions :

1. NEFT is used for Electronic Transfer of Funds between two bank


accounts. Which of the following is not mandatory for NEFT
transaction?
A. MICR Code
B. IFSC Code

www.bankingpdf.com
C. Recipient’s Bank Account Number
D. Recipient’s Bank Account Name

Answer & Explanation

A. MICR Code
Explanation:NEFT (National Electronic Funds Transfer) offered by the
RBI allow electronic transfer of funds from the remitter, who has an
account in one bank, to the beneficiary, who has an account in any
other bank/branch. The transfer can be carried out using the Internet
banking facility. NEFT is settled in batches at times defined by the RBI.

2. Which committee recommended the set up of Regional Rural


Banks?
A. Nayak Committee
B. A P Shah Committee
C. Narsimhan Committee
D. Rangarajan Committee

Answer & Explanation

C. Narsimhan Committee
Explanation:Regional Rural Banks are local level banking
organizations and are created with a view to serve primarily the rural
areas of India with basic banking and financial services.

3. Bank for International Settlements which provides the Banking


supervision accords is located in ________.
A. England
B. USA
C. France
D. Switzerland

Answer & Explanation

www.bankingpdf.com
D. Switzerland
Explanation:The mission of the BIS is to serve central banks in their
pursuit of monetary and financial stability, to foster international
cooperation in those areas and to act as a bank for central banks.

4. Banks borrow funds from which of the following to meet the


Cash Reserve Ratio (CRR) and Statutory Liquidity Ratio (SLR)
requirements as mandated by the Central bank?
A. Notice Money Market
B. Call Money Market
C. Bill Market
D. Money Market

Answer & Explanation

B. Call Money Market


Explanation:Call money market the most active and sensitive part of
the organized money market. It is centered mainly at Bombay.
Calcutta and Madras are the market at Bombay being the most
important. It deals in one-day loans (called call loans or call money)
which may or may not be renewed the next day. The participants are
mostly banks. Therefore, it is also called inter-bank call money market.

5. The apex Bank of the USA is:


A. Central National Bank of USA
B. Federal Reserve Bank
C. Bank of America
D. Reserve Bank of USA

Answer & Explanation

B. Federal Reserve Bank


Explanation:A Federal Reserve Bank is a regional bank of the Federal
Reserve System, the central banking system of the United States. The
Federal Reserve System provides the government with a ready source

www.bankingpdf.com
of loans and serves as the safe depository for federal money. The
Federal Reserve Banks were created as instrumentalities to carry out
the policies of the Federal Reserve System.

6. Currency notes deposited in the currency chest belongs to


____.
A. SEBI
B. RBI
C. SBI
D. Government of India

Answer & Explanation

B. RBI
Explanation:Distribution of notes throughout the country is done
through designated bank branches and is called chests. It is a
repository in a commercial bank to store notes and coins on behalf of
the Reserve Bank. Deposit into chest leads to credit of the commercial
bank’s account and withdrawal, debit.

7. If a company which is not NBFC wants to collect public deposits


it is governed by which Act:
A. RBI Act 1934
B. Banking companies Act
C. Central Government
D. Companies Act 1956

Answer & Explanation

D. Companies Act 1956


Explanation:The Companies Act 1956 was an Act of the Parliament of
India, enacted in 1956, which enabled companies to be formed by
registration, and set out the responsibilities of companies, their
directors and secretaries.

www.bankingpdf.com
8. _________ is not an electronic Banking delivery medium?
A. Mobile Vans
B. Mobile Phone Banking
C. Internet Banking
D. Tele Banking

Answer & Explanation

A. Mobile Vans
Explanation:Electronic banking, also known as electronic fund transfer
(EFT) uses computer and electronic technology in place of checks and
other paper transactions. EFTs are initiated through devices like cards
or codes that let you, or those you authorize, access your account.
Many financial institutions use ATM or debit cards and Personal
Identification Numbers (PINs) for this purpose.

9. Full form of STT is


A. Standard Transaction Tax
B. Standard Transfer tax
C. Securities Transfer tax
D. Securities Transaction tax

Answer & Explanation

D. Securities Transaction tax


Explanation:STT is a kind of turnover tax where the investor has to
pay a small tax on the total consideration paid or received in a share
transaction. The objective behind the levy is to mitigate tax evasion as
the same is taxed at source. Stocks, futures, option, mutual funds and
exchange traded funds come under the ambit of STT.

10. The Chit funds are regulated by ___ in India.


A. RBI
B. Government of India

www.bankingpdf.com
C. State Governments
D. Local Bodies

Answer & Explanation

C. State Governments
Explanation:Regulator of chit funds is the Registrar of Chits appointed
by respective state governments under Section 61 of Chit Funds Act.
Powers of adjudication vest in the Registrar and the state government
concerned is the Appellate authority.
A chit fund is a kind of savings scheme practiced in India. A chit fund
company is a company that manages, conducts, or supervises a chit
scheme.

690-700 Questions :

1. Reserve Bank of India uses __________ to control inflation in


our country.
A. Interest rate
B. Decrease the SLR
C. Increase in CRR
D. None of these

Answer & Explanation

A. Interest rate
Explanation:Repo rate influences most the given money supply in the
economy. It is the rate at which the banks borrow short-term funds
from the RBI. RBI raises repo rate to increase the overall cost of funds
in the banking system. If the central bank hikes its repo rate, it
becomes costly for banks to borrow money from RBI so they in turn
hike the loan interest rates at which customers borrow money from
them to compensate for the hike in repo rate.

www.bankingpdf.com
2. NIM stands for
A. Net Interest Margin
B. Net Involvement Margin
C. Net Involution Margin
D. Net Inflation Margin

Answer & Explanation

A. Net Interest Margin


Explanation:Net interest margin is a measure of an investing
strategy’s success, especially when investors are attempting to
arbitrage the market by borrowing at a rate that they believe is below
what their potential returns will be. Banks are keenly interested in their
net interest margins because they lend at one rate and pay depositors
at another.

3. It is a contract where both parties agree to buy and sell a


particular asset of specific quantity and at a predetermined
price.
A. Forward Contract
B. Swap
C. Futures Contract
D. Options Contract

Answer & Explanation

C. Futures Contract
Explanation:Futures contract is a contract between two parties where
both parties agree to buy and sell a particular asset of specific quantity
and at a predetermined price, at a specified date in future. The
payment and delivery of the asset is made on the future date termed
as delivery date. The buyer in the futures contract is known as to hold
a long position or simply long. The seller in the futures contracts is said
to be having short position or simply short.

www.bankingpdf.com
4. ___________ illustrates the financial position of a bank at a
given point of time.
A. Balance sheet
B. Cash flow statement
C. Income statement
D. Audit

Answer & Explanation

A. Balance sheet
Explanation:The balance sheet is among the main financial
statements used in financial accounting. Keep in mind these main
concepts: The balance sheet is like a snapshot of a company’s financial
position, The balance sheet is comprised of two main sections:- Asset
and Liability & Equity, The Balance sheet must always balance: the
Asset section must always equal the Liability + Equity section.

5. The largest bank in the world is


A. BNP Paribas
B. SBI
C. HSBC Holdings
D. Industrial & Commercial Bank of China

Answer & Explanation

D. Industrial & Commercial Bank of China


Explanation:Four of the five largest banks in the world are Chinese.
It’s a big change from the past few years when only two Chinese banks
made the top five. Top 5 banks are-1. Industrial & Commercial Bank of
China (China) 2. China Construction Bank (China) 3. Agricultural Bank
of China (China) 4. HSBC (U.K.) 5. Bank of China (China)
*Industrial & Commercial Bank of China-Founded: January 1, 1984,
Beijing, China
Headquarters: Xicheng District, Beijing, China

www.bankingpdf.com
6. ________ is used when describing the monthly charges on a
mortgage.
A. BITI
B. CPI
C. DPI
D. PITI

Answer & Explanation

D. PITI
Explanation:The acronym PITI stands for principal, interest, taxes,
and insurance. These are the four components that make up a monthly
mortgage payment. When determining your eligibility for a loan, your
mortgage lender will look at your debt-to-income ratios, comparing
your PITI to your monthly gross income. This helps them to determine
how much you can afford.

7. The first Indian bank to get ISO was ___________.


A. PNB
B. Canara Bank
C. SBI
D. Bank of Baroda

Answer & Explanation

B. Canara Bank
Explanation:Canara Bank(HQ Bangalore,Karnataka) was established
at Mangalore in 1906, making it one of the oldest banks in the country.
The government nationalized the bank in 1969. In 1996 Canara Bank
became the first Indian Bank to get ISO certification for “Total Branch
Banking” for its Seshadripuram branch in Bangalore. Canara Bank has
now stopped opting for ISO certification of branches.
*Rakesh Sharma (MD & CEO)

www.bankingpdf.com
8. Which bank introduced cheque system in India?
A. Presidency Bank
B. Imperial Bank of India
C. Bengal Bank
D. PNB

Answer & Explanation

C. Bengal Bank
Explanation:Cheque system was first introduced by Bengal Bank
which was established in 1784. In 1833, cheque system was
introduced by Bengal Bank.

9. It is a tax which is levied at the same rate at all income levels.


A. Progressive Tax
B. Regressive Tax
C. Proportional Tax
D. Sales Tax

Answer & Explanation

C. Proportional Tax
Explanation:Proportional tax system is one in which income tax is the
same percentage of income from every person no matter how much
income the person makes. A proportional tax system, often also
referred to as a flat tax, does not consist of income tax brackets.

10. A person who lends money against a pledged article which


he or she is free to sell if the loan is not repaid within interest
within a stated period is _________.
A. Dealer
B. Pawnbroker
C. Investor
D. None of these

www.bankingpdf.com
Answer & Explanation

B. Pawnbroker
Explanation:Pawnbroker is a small lender who lends money at a high
interest rate and holds some of the borrower’s personal goods as
collateral, to be sold to the public in the event of default.

700-710 Questions :

1. What is the meaning of Lock period in Term deposit?


A. Customer cannot open another account
B. Nominee name need not be written
C. Pre-maturity is not possible
D. Locker must be opened by the customer in the branch

Answer & Explanation

C. Pre-maturity is not possible


Explanation:Term deposits are a fixed term investment which means
that your money is ‘locked away’ until the end of the term when it
reaches maturity. Term deposits come with a fixed rate as opposed to
a variable interest rate, and are designed to be left in the account until
the end of the term, which means if you need to close the account
early or you want to access your money before the term ends, you will
be charged a fee.

2. It is the market which expects the prices of shares rise in the


market.
A. Fundamental market
B. Bull market
C. Bear Market
D. Depressed market

www.bankingpdf.com
Answer & Explanation

B. Bull market
Explanation:In a bull market, investors are positive. The economy
tends to be strong. Unemployment is low. Consumers are spending
money, which increases business profits.

3. Which of the following is false about Carbon Credit?


A. Tradable certificate
B. Right of an Industry to emit certain quantity of Carbon Dioxide
C. Carbon Credits mechanism was formalized in Kyoto Protocol
D. None

Answer & Explanation

D. None
Explanation:A carbon credit (often called a carbon offset) is a
financial instrument that represents a tonne of carbon dioxide or
carbon dioxide equivalent gases removed or reduced from the
atmosphere from an emission reduction project, which can be used, by
governments, industry or private individuals to offset damaging carbon
emissions that they are generating.

4. MTSS stands for


A. Money Transfer Scheme for Service
B. Medium Transfer Service Strategy
C. Minimum Transfer Service Scheme
D. Money Transfer Service Scheme

Answer & Explanation

D. Money Transfer Service Scheme


Explanation:Money Transfer Service Scheme (MTSS) is a quick and
easy way of transferring personal remittances from abroad to
beneficiaries in India. Only inward personal remittances into India such

www.bankingpdf.com
as remittances towards family maintenance and remittances favouring
foreign tourists visiting India are permissible.

5. The first RRB named Prathama Grameen Bank was started by


______.
A. Punjab National Bank
B. State Bank of India
C. Syndicate Bank
D. Canara Bank

Answer & Explanation

C. Syndicate Bank
Explanation:The development process of RRBs started on 2 October
1975 with the forming of the first RRB, the Prathama Bank. Also on 2
October 1976 five regional rural banks were set up with a total
authorised capital of Rs.100 crore which later augmented to 500 crore.
RRBs were owned by the Central Government, the State Government
and the Sponsor Bank(There were five commercial banks, Punjab
National Bank, State Bank of India, Syndicate Bank, United Bank of
India and United Commercial Bank, which sponsored the regional rural
banks).

6. SWIFT stands for


A. Secure Window for Interbank Financial Transactions
B. Society Window In case of Financial Transaction
C. Society for Worldwide Interbank Financial Telecommunication
D. Secure Window Institute For Transactions

Answer & Explanation

C. Society for Worldwide Interbank Financial


Telecommunication
Explanation:Society for Worldwide Interbank Financial
Telecommunication code. An internationally-recognized identification

www.bankingpdf.com
code for banks around the world. SWIFT codes are most commonly
used for international wire transfers and are comprised of 8 or 11
alphanumeric characters.

7. Fiat money is defined as


A. Currency supported by Government Guarantee
B. Currency supported by tangible assets
C. Currency supported by gold reserves
D. Currency supported by budgetary support

Answer & Explanation

A. Currency supported by Government Guarantee


Explanation:As valid currency solely by virtue of a government
declaration, fiat money is not backed by any commodity, such as gold,
but only by the faith of the bearer. In this respect, unlike currencies
backed by gold or silver, fiat money does not have any intrinsic value
(e.g., paper money and much coinage). The U.S. dollar is an example
of fiat money.

8. Which among the following countries commenced first plastic


notes?
A. China
B. America
C. Canada
D. Australia

Answer & Explanation

D. Australia
Explanation:Modern polymer banknotes were first developed by the
Reserve Bank of Australia (RBA), Commonwealth Scientific and
Industrial Research Organisation (CSIRO) and The University of
Melbourne, first issued as currency in Australia in 1988. In 1996
Australia switched completely to polymer banknotes.

www.bankingpdf.com
*Recent: Plastic banknotes that can survive a spin in the washing
machine are to be brought into circulation by the Bank of England in
2016.

9. The movement that encouraged the foundation of commercial


banks was ________.
A. Swadeshi movement
B. Quit India Movement
C. Non Cooperation Movement
D. Swadhar Movement

Answer & Explanation

A. Swadeshi movement
Explanation:Swadeshi movement, which began in 1906, encouraged
the formation of a number of commercial banks. Banking crisis during
1913-1917 and failure of 588 banks in various parts of the country
during the decade ended 1949 underlined the need for regulating and
controlling commercial banks. The Banking Companies Act was passed
in February 1949, which was subsequently amended to read as
Banking Regulation Act, 1949. This Act provided the legal framework
for regulation of the banking system in India.

10. The special drawing right granted by the International


Monetary Fund to its member countries is _______________.
A. Cold money
B. Hot money
C. Soft money
D. Paper gold

Answer & Explanation

D. Paper gold
Explanation:International financing instrument created in 1970 by the
International Monetary Fund (IMF) to coincide with the disfavor of the

www.bankingpdf.com
US dollar as the principal currency of the world trade. Also called paper
gold, an SDR is neither paper nor gold but an accounting entry. It is
not backed by any currency or precious metal, and is used only among
governments and IMF for balance Of payments settlements.

710-720 Questions :

1. Demat Accounts are


A. Accounts in which shares of various companies are traded in
electronic form
B. Accounts which are operated through internet banking facility
C. Accounts which are opened to facilitate repayment of a loan taken
from the bank.
D. Zero Balance Accounts

Answer & Explanation

A. Accounts in which shares of various companies are traded in


electronic form
Explanation:Shares and securities are held in dematerialised form in
demat account. It is mandatory to have a demat account to carry out a
transaction in stock exchange. Once an account is opened, you can buy
shares by visiting a broker personally or online. To open a demat
account, you need to get in touch with a registered depository
participant (DP). It is just like a bank or broker.

2. Cryptocurrency is a _______
A. Digital payment
B. Digital Medium of Exchange
C. Both B & D
D. Digital Currency

www.bankingpdf.com
Answer & Explanation

C. Both B & D
Explanation:A cryptocurrency is a medium of exchange like normal
currencies such as USD, but designed for the purpose of exchanging
digital information through a process made possible by certain
principles of cryptography.

3. Which one of the following is not true about Banking


regulations?
A. Credit Allocation
B. Prudential Objectives
C. Systematic Risk Reduction
D. All are true

Answer & Explanation

D. All are true


Explanation:The objectives of bank regulation, and the emphasis,
vary between jurisdictions. The most common objectives
are:1.Prudential: to reduce the level of risk bank creditors are exposed
to 2.Systemic risk reduction:to reduce the risk of disruption resulting
from adverse trading conditions for banks causing multiple or major
bank failures 3.Avoid misuse of banks:to reduce the risk of banks being
used for criminal purposes,e.g.laundering the proceeds of crime 4.To
protect banking confidentiality 5.Credit allocation:to direct credit to
favored sectors

4. It is the ratio of a bank’s capital in relation to its risk weighted


assets and current liabilities.
A. Solvency Ratio
B. Liquidity Ratio
C. Capital Adequacy Ratio
D. None of these

www.bankingpdf.com
Answer & Explanation

C. Capital Adequacy Ratio


Explanation:Capital Adequacy Ratio (CAR) is the ratio of a bank’s
capital in relation to its risk weighted assets and current liabilities. It is
decided by central banks and bank regulators to prevent commercial
banks from taking excess leverage and becoming insolvent in the
process.

5. In banking CDR stands for


A. Corporate Debt Restructuring
B. Corporate Debt Rollover
C. Corporate Deposit Restructuring
D. None of these

Answer & Explanation

A. Corporate Debt Restructuring


Explanation:Corporate Debt Restructuring mechanism is a voluntary
non statutory mechanism under which financial institutions and banks
come together to restructure the debt of companies facing financial
difficulties due to internal or external factors, in order to provide timely
support to such companies.

6. The maximum time period for which domestic term deposits are
generally accepted by banks in India is _________.
A. 5 years
B. 4 years
C. 10 years
D. 3 years

Answer & Explanation

C. 10 years
Explanation:A term deposit is a cash investment held at a financial

www.bankingpdf.com
institution – bank, building society or credit union – for an agreed rate
of interest over a fixed amount of time, known as a term.

7. Which one of the following is not a key function of a Bank?


A. Collecting Cheques/Drafts customers
B. Granting Loans
C. Easing import of goods
D. Issuing Bank Drafts

Answer & Explanation

C. Easing import of goods


Explanation:Primary functions:Collection of deposits, Making loans
and advances
Secondary functions:Agency services:General utility services

8. Which asset can be mortgaged?


A. Land and Building
B. Sweat equity
C. Illegally obtained funds
D. Undocumented funds

Answer & Explanation

A. Land and Building


Explanation:A mortgage is a way to use one’s real property, like land,
a house, or a building, as a guarantee for a loan to get money. Many
people do this to buy the home they use for mortgage: the loan
provides them the money to buy the house and the loan is guaranteed
by the house.

9. What does the lowering of Bank Rate by the Reserve Bank of


India leads to
A. More liquidity in the market
B. Less liquidity in the market

www.bankingpdf.com
C. Stability liquidity in the market
D. None of these

Answer & Explanation

A. More liquidity in the market


Explanation:Bank Rate is the rate at which central bank of the
country (in India it is RBI) allows finance to commercial banks. Bank
Rate is a tool, which central bank uses for short-term purposes.
*Current Bank Rate: 7.75%

10. Which is the loan of very small amounts given to low income
groups?
A. Macro Credit
B. Micro Credit
C. Micro Finance
D. None of these

Answer & Explanation

B. Micro Credit
Explanation:Microcredit is the extension of very small loans
(microloans) to impoverished borrowers who typically lack collateral,
steady employment and a verifiable credit history. It is designed not
only to support entrepreneurship and alleviate poverty, but also in
many cases to empower women and uplift entire communities by
extension

www.bankingpdf.com
730 – 740 Questions :

1. It gives the right to retain possession only of goods in respect


of which the dues have arisen and not for other dues.
A. General Lien
B. Particular Lien
C. Specific Lien
D. None of these

Answer & Explanation

B. Particular Lien
Explanation:A lien is a right of any one person to retain that, which is
in his possession, belonging to another, until certain demands of a
person in possession are satisfied.

2. ___________ can purchase immovable property in India.


A. NRI
B. PIO
C. Both A & B
D. None of these

Answer & Explanation

C. Both A & B
Explanation:Person resident outside India is categorized as Non-
Resident Indian (NRI) or a foreign national of Indian Origin (PIO) or a
foreign national of non-Indian origin.

3. NBFCs whose asset size is of __________ or more as per last


audited balance sheet are considered as systemically important
NBFCs.
A. Rs.500 Crore
B. Rs.100 Crore

www.bankingpdf.com
C. Rs.400 Crore
D. Rs.200 Crore

Answer & Explanation

A. Rs.500 Crore
Explanation:A non-banking institution which is a company and has
principal business of receiving deposits under any scheme or
arrangement in one lump sum or in installments by way of
contributions or in any other manner, is also a non-banking financial
company (Residuary non-banking company).

4. CSGL account stands for


A. Connected Subsidiary General Ledger account
B. Corelated Sub General Ledger account
C. Composite Subsidiary General Ledger account
D. Constituents’ Subsidiary General Ledger account

Answer & Explanation

D. Constituents’ Subsidiary General Ledger account


Explanation:CSGL, i.e. Constituents’ Subsidiary General Ledger
account, means an SGL account opened and maintained with RBI by an
agent on behalf of the constituents of such agent, i.e. a second SGL
account opened by an agent with the RBI to hold the securities on
behalf of their constituents. The constituents are known as the Gilt
Account Holders (GAHs).

5. Bond Ledger Account(BLA) means an account with RBI or an


agency bank in which the Government securities are held in a
__________ form to the credit of the holder.
A. materialzied
B. dematerialized
C. balanced
D. None of these

www.bankingpdf.com
Answer & Explanation

Explanation:The investor in this case receives a Certificate of Holding


or Certificate of Investment from RBI/Agency Banks.
Dematerialization in short called as ‘demat is the process by which an
investor can get physical certificates converted into electronic form
maintained in an account with the Depository Participant.

6. ATS stands for _________.


A. Applied Tracking System
B. Application Transferring System
C. Application Tracking System
D. Asynchronous Tracking System

Answer & Explanation

C. Application Tracking System


Explanation:It is hosted on the public website of the Reserve Bank of
India (RBI), which has been developed for members of the public to
submit any individual application to RBI and keep track of the status of
its disposal thereafter. An application can be any application,
addressed to any department of RBI, through which members of the
public can apply.

7. Who decided the limits on cash withdrawal at ATMs and for


purchase of goods and services?
A. Issuer Bank
B. RBI
C. Government
D. SEBI

Answer & Explanation

A. Issuer Bank
Explanation:In case of cash withdrawal at other bank’s ATM, there is

www.bankingpdf.com
a limit of Rs 10,000/- per transaction. Cash withdrawal at POS has also
been enabled by certain banks wherein, a maximum of Rs.1000/- can
be withdrawn daily by using debit cards.

8. What is the option for a card holder if his complaint is not


redressed by the issuer bank within a maximum period of thirty
days from the date of his lodging the complaint?
A. Approach RBI Governor
B. Approach Banking Ombudsman
C. Approach Branch Manager
D. Approach Executive Director

Answer & Explanation

B. Approach Banking Ombudsman


Explanation:The Banking Ombudsman is a senior official appointed by
the Reserve Bank of India to redress customer complaints against
deficiency in certain banking services.

9. The type of cards that can be used at an ATM/WLA


___________.
A. Credit Cards
B. Pre-paid Cards
C. Debit Cards
D. All of these

Answer & Explanation

D. All of these
Explanation:The ATM/ATM cum debit cards, credit cards and open
prepaid cards issued by banks can be used at ATMs/WLAs for various
transactions. ATMs set up, owned and operated by non-banks are
called White Label ATMs. Non-bank ATM operators are authorized
under Payment & Settlement Systems Act, 2007 by the Reserve Bank
of India.

www.bankingpdf.com
10. An Individual can have _____ number of ‘Basic Savings Bank
Deposit Account’ in one bank?
A. One
B. Two
C. Four
D. Any

Answer & Explanation

A. One
Explanation:Holders of ‘Basic Savings Bank Deposit Account’ will not
be eligible for opening any other savings bank account in that bank. If
a customer has any other existing savings bank account in that bank,
he / she will be required to close it within 30 days from the date of
opening a ‘Basic Savings Bank Deposit Account’.

11. Whose signature is found on a 100 rupee currency note in


India?
A. Deputy Governor of RBI
B. Finance Secretary
C. RBI Governor
D. None of these

Answer & Explanation

C. RBI Governor
Explanation:Rupee one notes bearing the signature of Secretary,
Ministry of Finance while other denominations bearing the signature of
the Governor of Reserve Bank of India are Bank Notes.In any INDIAN
note there are 15 Languages.

www.bankingpdf.com
740 – 750 Questions :

1. Who decides Bank rate?


A. Government
B. RBI
C. SEBI
D. SBI

Answer & Explanation

B. RBI
Explanation:Bank rate is the rate at which RBI lends money to
commercial banks for meeting shortfall for a long period without selling
or buying any security. Bank rate would be always higher than Repo
rate.

2. These are the reserves that are held by Commercial Banks over
and above the statutory minimum with the central bank.
A. Excess reserves
B. Cash reserves
C. Deposit reserves
D. Momentary reserves

Answer & Explanation

A. Excess reserves
Explanation:Banks are required to have at the end of each day
reserves equal at least to a certain percentage of their loans (possibly
adjusted for the risk of each of the loans). If the bank has taken in
more deposits or increased owner’s equity (with retained profits or by
selling more stock) or had loans paid off or borrowed money, it might
have more reserves than the minimum required. Those banks loan
money to the banks that are short on reserves and vice versa.

www.bankingpdf.com
3. Who sponsors Regional Rural Banks?
A. Government of India
B. Reserve Bank of India
C. Nationalised Commercial Bank
D. None of these

Answer & Explanation

C. Nationalised Commercial Bank


Explanation:RRBs like any other commercial banks (PSBs or PVT
sector banks) are governed by BRAct, RBI Act and other guidelines
issued by Ministry of Finance, Government from time to time. But each
RRB has a Board of Directors consisting of nine members headed by
the Chairman, who generally belongs to the Sponsor Bank. NABARD is
responsible for overall supervision and inspection of the RRBs.

4. India’s first bi-metallic coins issued in the year 2009 were


denominated in ____.
A. Rs. 1
B. Rs. 50
C. Rs. 5
D. Rs. 10

Answer & Explanation

D. Rs. 10
Explanation:Bi-metallic coins are coins consisting of more than one
metal or alloy, generally arranged with an outer ring around a
contrasting center. India’s much awaited first ever bimetallic 10 rupee
coin was released in 2005 under the theme “Unity in Diversity”. But,
due to its controversial design resembling a cross, it was criticised and
was not minted in large numbers. Another reason for its availability
being scarce is that it was minted only in one (Noida) of four mints in
India.

www.bankingpdf.com
5. Because of the existence of _________ India is called a mixed
economy.
A. Private Sector
B. Corporate Sector
C. Public Sector
D. Both A & C

Answer & Explanation

D. Both A & C
Explanation:A mixed economy consists of both private companies and
government/state-owned entities. Both have control of owning,
making, selling, and exchanging goods in the country. Where there is
both capitalism and socialism. Mixed economy is somehow referred as
the best economy. Mixed economy increases the competition between
socialism and capitalism and hence deliver the best quality.

6. RRBs are working in all states of the country except _______.


A. Sikkim
B. Goa
C. Andhra Pradesh
D. Both A & B

Answer & Explanation

D. Both A & B
Explanation:RRB’s were established by the Government of India in
the year 1975. Their establishment was followed by the 1976 Regional
Rural Banks Act. These have had the main agenda to develop the
economy of the rural areas, and to enlarge credit amount of these
areas as well as the agricultural sectors.

7. It is the amount outstanding in the borrowal account in books


of the bank other than the one which has been recorded and
not debited to the borrowal account.

www.bankingpdf.com
A. Net NPAs
B. Gross NPAs
C. Both A & B
D. None of these

Answer & Explanation

B. Gross NPAs
Explanation:The term NPA or Non-Performing Asset is usually used in
Indian banking to define a loan which is not getting repaid. Such loans
also have to be provided for, by reducing the profit by a certain
predetermined formula. So assuming that a bank gave a loan of
Rs.100/- of which maybe Rs.3/- was repaid, then Gross NPA is Rs.97/-.
Then the bank makes a provision of Rs.2/-, so the Net NPA becomes
Rs.95/-. In this highly simplified example, interest charged on the loan
has been subsumed into the original Rs.100/-

8. Non-Performing Asset is an advance where the a/c remains


overdue for a period of more than ______ in the case of respect
of an Overdraft/Cash Credit.
A. 180 Days
B. 90 Days
C. 60 Days
D. 6 Months

Answer & Explanation

B. 90 Days
Explanation:If there are no customer-induced credits in the account
to provide for the bank’s claims in terms of interest and other charges
for a period of at least 90 days.

9. It is a number which basically represents the value in rupees


per fund units as on a particular date of the assets of the fund
less liabilities and outstanding expanses.

www.bankingpdf.com
A. YTM
B. NAV
C. MFs
D. None of these

Answer & Explanation

B. NAV
Explanation:NAV is often associated with mutual funds, and helps an
investor determine if the fund is overvalued or undervalued. When we
talk of open-end funds, NAV is crucial. NAV gives the fund’s value that
an investor will be entitled to at the time of withdrawal of investment.
In case of a close-end fund, which is a mutual fund with fixed number
of units, price per unit is determined by market and is either below or
above the NAV.

10. The banks joins together to meet the working capital


requirements of the borrowing concerns and it is called as
A. Consortium finance
B. CMA
C. Credit Syndication
D. None of these

Answer & Explanation

A. Consortium finance
Explanation:Consortium financing occurs for transactions that might
not take place with a single lender. Several banks may agree to jointly
supervise a single borrower with a common appraisal, documentation
and follow-up. Consortiums are not built to handle international
transactions such as a syndication loan; instead, a consortium may
arise because the size of the project at hand is simply too large or too
risky for any single lender to assume.

www.bankingpdf.com
750 – 760 Questions :

1. Rate at which the domestic currency can be converted into


foreign currency and vice-versa is known as _______.
A. External Value
B. Exchange Rate
C. Spot Rate
D. Interest Rate

Answer & Explanation

B. Exchange Rate
Explanation:An exchange rate represents the relative price of two
currencies. For example, the dollar–euro exchange rate implies the
relative price of the euro in terms of dollars. If the dollar–euro
exchange rate is $0.95, it means that you need $0.95 to buy €1.
Therefore, the exchange rate states how many units of one currency
you need to buy one unit of another currency.

2. These are the reserves that can act as a liquidity buffer for
commercial banks during their crisis times.
A. SLR
B. Forex Reserves
C. DSLR
D. Semi SLR

Answer & Explanation

A. SLR
Explanation:Statutory Liquidity Ratio is amount that a commercial
bank should have before giving credits to its customers which should
be either in the form of gold, money or bonds.

3. At which rate is TDS deducted If PAN details are not provided?


A. 50%

www.bankingpdf.com
B. 30%
C. 10%
D. 20%

Answer & Explanation

D. 20%
Explanation:TDS (tax deducted at source) rate without dedcutee PAN
is the Rate provided in the act (normal TDS rate or 20 % whichever is
higher .This clause is applicable to all type of deductee .The Maximum
TDS rate is applicable in following case
1)If pan is not provided by the deductee
2)If pan provided by the the deductee is invalid

4. ALM in Banking/Finance sector stands for _________


A. Asynchronous Liquidity Management
B. Asset Liability Manufacturing
C. Asset Liability Management
D. Anti-Liability Management

Answer & Explanation

C. Asset Liability Management


Explanation:Asset Liability Management (ALM) can be defined as a
mechanism to address the risk faced by a bank due to a mismatch
between assets and liabilities either due to liquidity or changes in
interest rates.

5. Bank lends to Micro Finance Institutions and is considered as


finance to __________.
A. Priority Sector
B. Secondary Sector
C. Tertiary Sector
D. None of these

www.bankingpdf.com
Answer & Explanation

A. Priority Sector
Explanation:Categories under priority sector are Agriculture, Export
Credit, Education, Housing but recently,Reserve Bank of India (RBI)
has revamped priority sector lending (PSL) norms. Now, loans to
sectors such as social infrastructure, renewable energy and medium
enterprises will also be treated as PSL.

6. It is a mortgage loan with the interest rate on the note


periodically adjusted based on an index which reflects the cost
to the lender of borrowing on the credit markets.
A. Adjustable-Rate, Mortgages
B. Fixed-Rate, Mortgages
C. Variable-Rate, Mortgages
D. None of these

Answer & Explanation

A. Adjustable-Rate, Mortgages
Explanation:An adjustable-rate mortgage (ARM) is a loan with an
interest rate that changes. ARMs may start with lower monthly
payments than fixed-rate mortgages. Your monthly payments could
change. They could go up sometimes by a lot even if interest rates
don’t go up.

7. A negotiable money market instrument and issued in


dematerialised form or as a Usance Promissory Note against
funds deposited at a bank for a specified time period is known
as
A. Bill of Exchange
B. Line of Credit
C. Cheque
D. Certificate of Deposit

www.bankingpdf.com
Answer & Explanation

D. Certificate of Deposit
Explanation:A Certificate of Deposit in India can be issue by: All
scheduled commercial banks excluding Regional Rural Banks (RRBs)
and Local Area Banks (LABs) Select All India Financial Institutions
permitted by RBI. Minimum amount is Rs. 1 Lakh, to be accepted from
a single subscriber and Larger amounts have to be in the multiples of
Rs. 1 Lakh. Maturity period is between seven days to one year for
commercial banks. For Financial Institutions, the maturity is not less
than a year and not more than three years.

8. Which of the following is an account on which funds may not be


withdrawn until a lien is satisfied and a court order or other
legal process makes the account available for withdrawal?
A. Frozen Account
B. Fixed Account
C. Hot Account
D. None of these

Answer & Explanation

A. Frozen Account
Explanation:When your bank account is frozen, it’s because you owe
money to someone. Any creditor that has a judgment against you can
have your bank account frozen. The creditor can actually freeze your
account for up to twice the amount you owe.

9. It is a collection of evidence of dishonor, under which the


Notary presents bill again to the drawee and on dishonor, given
a noting on the bill, which makes mention of date and reasons
of dishonor.
A. Protesting
B. Noting

www.bankingpdf.com
C. Crossing
D. Acceptance

Answer & Explanation

B. Noting
Explanation:Noting is a minute or memorandum made by a Notary
Public on a bill of exchange which has been dishonoured. The Bills of
Exchange Act instructs that noting to be done within 24 hours of
dishonour. It consists of their initials, their charges and the date.

10. Remittance received from RBI/other chest should be


examined immediately not exceeding ____ from date of
receipts.
A. 20Days
B. 15Days
C. 10Days
D. None of these

Answer & Explanation

B. 15Days
Explanation:Currency chests are select branches of scheduled banks,
which are authorised by the RBI to facilitate distribution of notes and
coins. In these branches, notes and coins are stocked on behalf of the
RBI. These currency chests are expected to distribute notes and coins
to other bank branches in their area of operation. There are 4,211
currency chests and 3,990 small coin depots spread across the
country.

www.bankingpdf.com
760 – 770 Questions :

1. Which of the following is a financial institution that assists


wealthy individuals, corporations, and governments in raising
capital by underwriting and/or acting as the client’s agent in
the issuance of securities?
A. Narrorw Bank
B. Investment Bank
C. Merchant Bank
D. Virtual Bank

Answer & Explanation

B. Investment Bank
Explanation:An investment bank may also assist companies with
mergers and acquisitions and may provide support services in market
making and trading of various securities. The primary services include:
Corporate finance, M&A, equity research, sales & trading, and asset
management. They can earn profit by charging fees and commissions
for providing these services and other kinds of financial and business
advice.

2. It is not a separate legal entity of the parent company and


liability wise there is no limit to the parent company’s liability.
A. Subsidiary
B. Branch
C. Kiosk
D. None of these

Answer & Explanation

B. Branch
Explanation:*Recent: RBI has permitted to Foreign Banks to change
from branch mode to the wholly owned Subsidiaries.

www.bankingpdf.com
3. A bank which is not included in second schedule and is not
eligible for loan from RBI is called as _________.
A. Scheduled Bank
B. Non-Scheduled Bank
C. SBI
D. None of these

Answer & Explanation

B. Non-Scheduled Bank
Explanation:Non-schedule banks are those banks whose total paid up
capital is less than Rs 5 lakh and RBI has no specific control over these
banks. These banks are not included in the second schedule of RBI Act,
1934.

4. Which of the following is a ratio that shows the efficiency of a


company’s management by comparing operating expense to net
sales?
A. Operating Ratio
B. Capital Adequacy Ratio
C. Gearing Ratio
D. None of these

Answer & Explanation

A. Operating Ratio
Explanation:Operating ratios show the essential connections between
income statement and balance sheet items. Operating ratios include
operating cash flow to sales ratio, operating expenses to gross margin
ratio, operating expenses to sales ratio, operating performance ratio,
and hundreds such others.

5. Fixed Interest Government Securities traded on Stock


Exchanges do not include Treasury bills are known as
A. Collateral Security

www.bankingpdf.com
B. Equity Securities
C. Gilt Edged Securities
D. Foreign Securities

Answer & Explanation

C. Gilt Edged Securities


Explanation:Government securities, also called the gilt edged
securities or G-secs, are not only free from default risk but also provide
reasonable returns and, therefore, offer the most suitable investment
opportunity to provident funds.

6. ___________ is usually used to stabilize the value of a


currency.
A. Fixed Currency
B. Currency Peg
C. Currency War
D. None of these

Answer & Explanation

B. Currency Peg
Explanation:When a country has a pegged currency it typically means
that the country wants to maintain a steady exchange rate to help
build stable trade relations. If a country’s currency is constantly
fluctuating relative to major currencies, it discourages potential trade
partners who are worried about unfavorable changes in exchange
rates. For example, oil-rich countries like Saudi Arabia and Venezuela
have their currencies pegged to the US dollar as the US is a major oil
importer (Dollarization).

7. DIPP is the nodal agency in the field of foreign investments in


India. What is the full form of DIPP?
A. Department of Industry Policy and Promotion
B. Department of Innovation Policy and Promotion

www.bankingpdf.com
C. Department of Industrial Policy and Promotion
D. Department of Industrial Policy and Partnership

Answer & Explanation

C. Department of Industrial Policy and Promotion


Explanation:It is working under the Ministry of Commerce and
Industry, Government of India. This department is responsible for
formulation and implementation of promotional and developmental
measures for growth of the industrial sector, keeping in view the
national priorities and socio-economic objectives.

8. Types of loans granted by banks for purchase of white goods


are
A. Consumer Durable Loans
B. White Loans
C. Consortium loans
D. None of these

Answer & Explanation

A. Consumer Durable Loans


Explanation:Consumer Durable loan is a finance option for purchase
of household items like Washing Machines, Refrigerators etc.

9. The letter ‘R’ denotes which word in the term IFRS?


A. Restructuring
B. Reporting
C. Recognizing
D. Remote

Answer & Explanation

B. Reporting
Explanation:International Financial Reporting Standards (IFRS) are a
set of accounting standards developed by the International Accounting

www.bankingpdf.com
Standards Board (IASB) that is becoming the global standard for the
preparation of public company financial statements.

10. Monetary policy is administrated by which Bank?


A. RBI
B. SEBI
C. SBI
D. OBC

Answer & Explanation

A. RBI
Explanation:Monetary policy is the method by which monetary
authority of a country, normally a central bank(RBI) controls the
supply of money in the economy by its control over interest rates in
order to maintain price stability and achieve high economic growth.

770 – 780 Questions :

1. Which of the following is a type of short term loan provided to


companies to fulfill their working capital requirements?
A. Cash Credit
B. Overdraft
C. Collateral
D. None of these

Answer & Explanation

A. Cash Credit
Explanation:The cash credit account is functions like a current
account with cheque book facility. The facility is provided against
pledge or hypothecation of stock i.e. raw materials, work in progress,

www.bankingpdf.com
finished goods, etc. or on the guarantee of book debts (debtors) or
other collateral security as per banking company norms.

2. A card which is used to make payments by the card holder, but


needs to be paid in full, on the due date when the statement is
received is known as ______.
A. Credit Card
B. Charge Card
C. Post-paid Card
D. Pre-paid Card

Answer & Explanation

B. Charge Card
Explanation:The card will not allow you to further carry the balance to
the next month and so you can borrow money for a particular period
only. Non repayment of money within stipulated time may cause you a
penalty or it may also amount to suspension of your account. Card
issuer puts up a boundary for the spendings the card holder makes
with the card, i.e. When the card holder makes payment, each and
every purchase is approved or denied at the register.

3. Which of the following is a bank account opened by NRI to


deposit income mainly generated from an Indian source?
A. NRI
B. NRP
C. NRO
D. FCNR

Answer & Explanation

C. NRO
Explanation:Non-Resident Ordinary Account held to manage the
income earned in India like rent, pension, dividend, interest, etc. It is
opened either by the citizen of India, while temporarily shifting to

www.bankingpdf.com
abroad or by an NRI by transferring money from his home country or
from his other NRO account. A normal bank account can also be
redesignated to NRO account when the residential status of the
account holder is changed to NRI. The account is mainly used to park
money earned from Indian sources in India.

4. In it a worldwide recognized identification code used at the


time of international credit transfer between banks and also
when there is an exchange of messages between banks.
A. IFSC Code
B. SWIFT Code
C. NEFT Code
D. None of these

Answer & Explanation

B. SWIFT Code
Explanation:SWIFT, Society for Worldwide Interbank Financial
Telecommunication Code. The code is also used when there is an
interchange of messages between banks. The code is a combination of
8 or 11 alphanumeric characters. First 4 characters bank code, Next 2
characters country code, Next 2 characters location code, Last 3
characters are optional that represents branch code.

5. In which of the following a credit facility is provided by the


bank to an entity for a specific purpose, to be repayable after a
short duration?
A. Advances
B. Loans
C. Security
D. None of these

Answer & Explanation

www.bankingpdf.com
A. Advances
Explanation:Advances are the source of finance, which are provided
by the banks to the companies to meet short term financial
requirement. They are granted against securities which are as under:
•Primary Security: Hypothecation of Debtors, Stock Pro-notes etc.
•Collateral Security: Mortgage of land and buildings, machinery, etc.
•Guarantees: Guarantees given by partners, directors or promoters,
etc.

6. This card usually has Maestro or Cirrus or Plus logo.


A. Debit Card
B. Credit Card
C. ATM Card
D. Pre-paid Card

Answer & Explanation

C. ATM Card
Explanation:ATM card is a card made up of a plastic, issued by the
bank, contains the basic details about the bank account of the
customer. The bank allows its customer to withdraw money at any
time with the use of the card through an Automated Teller Machine.
For the convenience of the public the bank has made this facility
available to its users 24 hours a day.

7. All Scheduled Commercial Banks (SCBs) having their current


account and Subsidiary General Ledger (SGL) with an RBI are
eligible for this rate.
A. Bank Rate
B. Repo Rate
C. MSF
D. Reverse Repo Rate

Answer & Explanation

www.bankingpdf.com
C. MSF
Explanation:Marginal Standing Facility Rate is referred to as a facility,
in which the scheduled commercial banks can borrow funds from the
central bank overnight, against the government approved securities of
Statutory Liquidity Ratio (SLR) quota (which is in excess of the current
SLR) up to a certain percentage of their Net Demand and Time
Liabilities. It is at the discretion of RBI whether to grant the loan or
not.

8. ________ of Rs. 20000 or more should be issued with “A/c


payee” crossing.
A. Banker’s Cheque
B. Cheque
C. FD
D. Demand Draft

Answer & Explanation

D. Demand Draft
Explanation:Demand Draft is a negotiable instrument issued by the
bank on the behalf of a customer, containing an order to pay a certain
sum to the payee from one branch to another branch of the same
bank. The validity period of a demand draft is 3 months, but it can be
re-validated against an application. It can never be dishonored because
its payment is done in advance.

9. Which of the following are the funds in which investor’s type is


Pension fund, endowment fund, high net worth individuals?
A. Mutual Fund
B. Hedge Fund
C. Debt Fund
D. Hybrid Funds

Answer & Explanation

www.bankingpdf.com
B. Hedge Fund
Explanation:A hedge fund is an investment partnership, where only
few high net worth investors can make an investment in the fund. The
minimum amount of initial investment in the fund is relatively high.
The fund is set free from strict regulations. The risk factor is very high
in hedge funds.

10. _________ is a negotiable instrument issued by the


international depository bank, representing foreign company’s
stock trading globally.
A. ADR
B. GDR
C. ARM
D. None of these

Answer & Explanation

B. GDR
Explanation:Global Depository Receipt is a negotiable instrument
used to tap the financial markets of various countries with a single
instrument. The receipts are issued by the depository bank, in more
than one country representing a fixed number of shares in a foreign
company. The holders of GDR can convert them into shares by
surrendering the receipts to the bank.

780 - 790 Questions :

1. It is the ability of commercial banks to increase their deposits


by expanding their loans and advances.
A. Capital Expansion
B. Capital Creation
C. Credit Expansion
D. Credit Creation

Answer & Explanation

www.bankingpdf.com
D. Credit Creation
Explanation:The creation of credit or deposits is one of the most
important functions of commercial banks. Like other corporations,
banks aim at earning profits. For this purpose, they accept cash in
demand deposits and advance loans on credit to customers.

2. The first bank in India to obtain license for Internet Banking


from Reserve Bank of India was
A. Oriental Bank of Commerce
B. State Bank India
C. Punjab National Bank
D. Syndicate Bank

Answer & Explanation

C. Punjab National Bank


Explanation:The main difference between Mobile Banking and
Internet Banking is that
Mobile banking can be operated from any mobile, even the most basic
one (through SMS). In smartphones mobile banking can be operated
through gprs and/or sms. Fund transferred(NEFT) through mobile
banking takes less time(as you do not have to register the beneficiary).
Where as internet banking needs internet connection. Unlike mobile
banking it can be operated through computers also. To transfer funds
through net banking you have to register the beneficiary which takes
some time ranging 2 to 12 working hrs.

3. To fight with raising inflation RBI has to


A. Increase reserve ratios
B. Sell government securities
C. Increase bank rate
D. All of the above

Answer & Explanation

www.bankingpdf.com
D. All of the above
Explanation:Inflation is Sustained increase in level of prices over time
for goods and services (food, health care, cost of education etc.). Each
unit of currency buying comparatively fewer goods and services.
Decrease in supply for goods and services. Following could be the
driving force –
• Industrial disputes
• Shortage of factories and production
• Natural calamities

4. One among the following is NOT a subsidiary of RBI and that is


A. National Housing Bank
B. NABARD
C. Bharatiya Reserve Bank Note Mudran Private Limited(BRBNMPL)
D. None of these

Answer & Explanation

D. None of these
Explanation:Subsidiaries of RBI
National Housing Bank (NHB)
Bharatiya Reserve Bank Note Mudran Private Limited (BRBNMPL)
National Bank for Agriculture and Rural Development (NABARD)
Deposit Insurance and Credit Guarantee Corporation (DICGC)

5. FATCA stands for


A. Foreign Account Tax Compliance Act
B. Foreign Account Trade Company Act
C. Financial Account Tax Compliance Act
D. Fiscal Account Tax Compliance Act

Answer & Explanation

A. Foreign Account Tax Compliance Act


Explanation:Foreign Account Tax Compliance Act (FATCA) is intended

www.bankingpdf.com
to detect and deter the evasion of US tax by US persons who hide
money outside the US. FATCA creates greater transparency by
strengthening information reporting and compliance by providing rules
around the processes of documenting, reporting and withholding on a
payee.

6. The following index used as the key measure of inflation in


India
A. Interest rate that bank offers on the deposits
B. Wholesale Price Index(WPI)
C. Consumer Price Index(CPI)
D. Sensex and Nifty

Answer & Explanation

C. Consumer Price Index(CPI)


Explanation:Consumer price index (CPI) measures changes in the
price level of a market basket of consumer goods and services
purchased by households.

7. R stands for _______ in CDR.


A. Ratio
B. Restructuring
C. Reserve
D. Regulation

Answer & Explanation

B. Restructuring
Explanation:Corporate Debt Restructuring (CDR) mechanism is a
mechanism under which financial institutions and banks come together
to restructure the debt of companies facing financial difficulties due to
internal or external factors, in order to provide timely support to such
companies.

www.bankingpdf.com
8. SARFAESI Act 2002 is mainly related to _________
A. Regulation of foreign exchange
B. Fixation of interest rates
C. Recovery of bad loans
D. Acquisition of small banks

Answer & Explanation

C. Recovery of bad loans


Explanation:The full form of SARFAESI Act as we know is
Securitisation and Reconstruction of Financial Assets and Enforcement
of Security Interest Act, 2002. Banks utilize this act as an effective tool
for bad loans ( NPA) recovery. It is possible where non-performing
assets are backed by securities charged to the Bank by way of
hypothecation or mortgage or assignment. Upon loan default, banks
can seize the securities (except agricultural land) without intervention
of the court.

9. Decrease of the bank rate by RBI leads to ________.


A. Mobilization of more deposits by commercial banks
B. Less liquidity in the market
C. More liquidity in the market
D. No change in the liquidity in the market

Answer & Explanation

C. More liquidity in the market


Explanation:Bank Rate is a Penal rate and has been merged with the
Marginal Standing Facility rate(MSF), which is generally kept at 1%
more than the Repo Rate by the RBI(India). MSF is the amount of Cash
that a Scheduled bank can borrow from the RBI by pledging its
reserves even though it is below the SLR(Statutory Liquidity Ratio),but
at a penal rate.

www.bankingpdf.com
10. Which of the following is or are the right(s) of a customer
towards his banker?
A. To sue the banker for not maintaining the secrecy of his account
B. To receive a statement of his account from a banker
C. To sue the bank for any loss and damages
D. All mentioned above

Answer & Explanation

D. All mentioned above


Explanation:A banker has certain rights also. Following are the major
rights that a banker can exercise on his customer.
• Right of Lien
• Right of set-off
• Automatic right of set off
• Right of Appropriation
• Right to charge interest
• Right to charge service charges

790 - 800 Questions :

1. A Committee on Banking Supervision which provides a forum


for regular cooperation on banking supervisory matters and to
enhance understanding of key supervisory issues and improve
the quality of banking supervision worldwide.
A. BIS
B. BASEL
C. European Banking Supervisors
D. None of these

Answer & Explanation

www.bankingpdf.com
B. BASEL
Explanation:BASEL is a set of international banking regulations put
forth by the Basel Committee on Bank Supervision, which set out the
minimum capital requirements of financial institutions with the goal of
minimizing credit risk.

2. Your account is _____ if you have taken more account than was
actually available as cleared funds. This can happen if you have
arranged a direct debit without having enough money in your
account to cover the payment.
A. Overdrawn
B. Default
C. Solvent
D. None of these

Answer & Explanation

A. Overdrawn
Explanation:If you try to use your debit card when there is not
enough money in your account to cover the transaction and your
account does not allow overdrawing, the transaction will be declined.
No fee is charged. If your account allows overdrawing, you can be
charged a fee, like with a check.

3. Any form of credit where you don’t have to provide an asset as


security against the loan is known as ____________.
A. Secured Credit
B. Cash Credit
C. Unsecured credit
D. None of these

Answer & Explanation

C. Unsecured credit
Explanation:

www.bankingpdf.com
Credit cards, store cards and some personal loans and overdrafts are
usually unsecured forms of credit. Unsecured forms of credit usually
have higher interest rates than secured forms of credit.

4. BIPS stands for


A. Board Internet Payment System
B. Bank Instrument Payment System
C. Bank Internet Payment Setup
D. Bank Internet Payment System

Answer & Explanation

D. Bank Internet Payment System


Explanation:It is an electronic system for making payments by
moving money directly into a bank account over the Internet

5. CHIPS stands for


A. Central House Interbank Payment System
B. Clearing House Interbank Payment System
C. Clearing House International Payment System
D. Clearing House Interbank Priority System

Answer & Explanation

B. Clearing House Interbank Payment System


Explanation:It is an electronic system for making international
payments in dollars and for changing money from one currency to
another

6. Which among the following is a metal container in the wall of a


bank that you can put money into when the bank is closed?
A. Wallet
B. Fireproof Safe
C. Safety Deposit Box
D. None of these

www.bankingpdf.com
Answer & Explanation

C. Safety Deposit Box


Explanation:Safety deposit boxes are used to store valuable
possessions, such as gemstones, precious metals, currency,
marketable securities, important documents such as wills, property
deeds, and birth certificates etc. In the United States, renting out a
safe deposit box in a bank does not mean that the property is
automatically insured. An individual should still purchase insurance for
the safe deposit box in order to cover theft and natural disasters.

7. Which among the following are the bonds that give the issuer
the right to redeem the bonds before their stated maturity?
A. Straight Bonds
B. Convertible Bonds
C. Call Bonds
D. Callable Bonds

Answer & Explanation

D. Callable Bonds
Explanation:A callable bond is a bond with an embedded call option.
If the issuer agrees to pay more than the face value amount of the
bond when called, the excess of the payment over the face amount is
the “call premium”. In most cases, the call price is greater than the par
price.

8. A combination of two or more securities into a single


investment position for the purpose of reducing or eliminating
risk is known as _____.
A. Leverage
B. Hedge
C. Mutual Funds
D. Speculation

www.bankingpdf.com
Answer & Explanation

B. Hedge
Explanation:A risk management strategy used in limiting or offsetting
probability of loss from fluctuations in the prices of commodities,
currencies, or securities. In effect, hedging is a transfer of risk without
buying insurance policies. Hedging is used in protecting one’s capital
against effects of inflation through investing in high-yield financial
instruments (bonds, notes, shares), real estate, or precious metals.

9. A financial ratio that measure the amount of debt being used to


support operations and the ability of the firm to service its debt
is ________.
A. Leverage Ratio
B. Liquidity Ratio
C. Risk based Capital
D. Debt Ratio

Answer & Explanation

A. Leverage Ratio
Explanation:The financial leverage ratios measure the overall debt
load of a company and compare it with the assets or equity. This
shows how much of the company assets belong to the shareholders
rather than creditors. When shareholders own a majority of the assets,
the company is said to be less leveraged. When creditors own a
majority of the assets, the company is considered highly leveraged. All
of these measurements are important for investors to understand how
risky the capital structure of a company and if it is worth investing in.

10. RSI stands for


A. Relative Statutory Index
B. Remote Strength Index
C. Relational Strength Index
D. Relative Strength Index

www.bankingpdf.com
Answer & Explanation

D. Relative Strength Index


Explanation:It is a stock’s price that changes over a period of time
relative to that of a market index such as the Standard & Poor’s 500,
usually measured on a scale from 1 to 100, 1 being the worst and 100
being the best.

800 - 810 Questions :

1. Different ways you can operate your account are


A. Telephone or Mobile banking
B. ATM (Automated Teller Machine)
C. Branch or Over the counter service
D. All of the above

Answer & Explanation

D. All of the above


Explanation:You can operate your bank accounts in different ways
like a) Internet banking b) Telephone or Mobile banking c) Branch or
Over the counter service d) ATM ( Automated Teller Machine)

2. Different types of Loans offered by banks


A. Unsecured Personal Loan
B. Auto Loans
C. Mortgage Loans
D. All of these

Answer & Explanation

D. All of these
Explanation:Currently in India we have different types of loans
available ranging from personal loans to marriage loans.

www.bankingpdf.com
1) Home Loan 2) Personal Loan 3) Car Loan or Vehicle Loan 4)
Education Loan 5) Gold Loan 6) Loan against Insurance Policies 7)
Loan against Bank FDs 8) Loan from PPF or EPF 9) Loan against Shares
or Mutual Funds 10) Loan from unrecognized sector.

3. It is a particular system of banking, in which a bank places its


funds in risk free assets with maturity period matching its
liability’s maturity timing and maintains liquidity at the time of
demand payment and their funds have no chance of becoming
an NPA.
A. Universal Banking
B. Merchant Banking
C. Narrow Banking
D. Virtual Banking

Answer & Explanation

C. Narrow Banking
Explanation:Narrow banking is a proposed type of bank called a
narrow bank also called a safe bank. Narrow banking would restrict
banks to holding liquid and safe government bonds. Loans would be
made by other financial intermediaries.

4. Which among the following is a system of payment in which the


bank account holder can authorize the customer to withdraw
the due amount, directly from his bank account?
A. Direct Order
B. Standing Order
C. Indirect Order
D. None of these

Answer & Explanation

A. Direct Order
Explanation:In this system, the payee can withdraw any sum at any

www.bankingpdf.com
point of time. The payee is having ultimate control over the payments
to decide the amount and time for payment without taking further
authorization from the payer. The payer will not have any control over
the withdrawals, but he can cancel them any time and need not to
mention any reason for that. The Direct Debit System notifies the
payee for cancellations or failures in payment, if any. Example – Gas
bill, Electricity Bill, Mobile Bills etc.

5. CORE Banking Solution where online multiple delivery


channel/(s) like _____ under one roof.
A. ATM
B. ABB
C. Debit Card
D. All of these

Answer & Explanation

D. All of these
Explanation:Core Banking can be understood as centralization on
banking activities. There was a time banking activities such as
deposits, withdrawal, etc. were limited to base branch (the branch
where the account belongs) only. But now you can do all that from
anywhere you wish. Not only the same banks but different banks as
well.

6. DTC stands for


A. Due tax clip
B. Direct turn code
C. Direct tax code
D. Debit tax company

Answer & Explanation

C. Direct tax code


Explanation:The Direct Tax Code (or DTC) aimed to make the system

www.bankingpdf.com
more efficient and easy for tax payers, with simplified rules and
regulations. It is a step towards replacing the four decade old Income
Tax Act of India.

7. The buyer of goods requests his bank to give guarantee that the
payment for the goods will be paid to the seller. In such case
the bank issues _______.
A. Credit Card
B. Line of Credit
C. Bill of Exchange
D. Letter of Credit

Answer & Explanation

B. Line of Credit
Explanation:A line of credit is the most you can borrow under a
revolving credit arrangement with a credit card issuer, bank, or
mortgage lender. When you borrow against a line of credit, you pay
interest on the amount of money you actually borrow, not on the
available balance, or full amount you are able to borrow.

8. When the repayment of a loan starts after certain period of its


disbursement, that period is
called ________.
A. Esoteric
B. Foreclosure
C. Diffusion
D. Moratorium

Answer & Explanation

D. Moratorium
Explanation:It is a period of time in which there is a suspension of a
specific activity until future events warrant a removal of the suspension
or issues regarding the activity have been resolved. In bankruptcy law,

www.bankingpdf.com
a legally binding halt of the right to collect debt. For example, if a
company is going through rough times it might have a moratorium on
advertising spending. In other words, to cut costs, it won’t spend any
money on advertising.

9. Which type of crisis occurs when the banks which have


obligation to facilitate money flow may get struck their revenue
assets with insolvents and become toxic assets?
A. Sub-prime Crisis
B. Super prime Crisis
C. Basic Crisis
D. None of these

Answer & Explanation

A. Sub-prime Crisis
Explanation:The term “sub-prime” refers to the credit status of the
borrower (being less than ideal), not the interest rate on the loan itself.
“sub-prime” is any loan that does not meet “prime” guidelines.
Subprime lending, also called B-paper, near-prime, or second chance
lending, is the practice of making loans to borrowers who do not
qualify for the best market interest rates because of their deficient
credit history.

10. “Rupee” symbol designed by Uday Kumar and he belongs to


A. Tamil Nadu
B. Kerala
C. UP
D. Delhi

Answer & Explanation

A. Tamil Nadu
Explanation:The Indian rupee with a unique symbol – a blend of the
Devanagri ‘Ra’ and Roman ‘R’ — joining elite currencies like the US

www.bankingpdf.com
dollar, euro, British pound and Japanese yen in having a distinct
identity. The new symbol, designed by Bombay IIT post-graduate D
Udaya Kumar, was approved by the cabinet reflecting the fact that the
Indian currency, backed by an over-trillion dollar economy, was finally
making its presence felt on the international scene.

810 - 820 Questions :

1. When the bank has not enough funds in the relevant account or
the account holder requests that the cheque is ________ then
the bank will return the cheque to the account holder.
A. Bounced
B. Honoured
C. Restarted
D. None of these

Answer & Explanation

A. Bounced
Explanation:On the dishonour of a cheque, one can file a suit for
recovery of the cheque amount along with the cost & interest under
order XXXVII of Code of Civil Procedure 1908 (which is a summary
procedure and) can also file a Criminal Complaint u/s 138 of Negotiable
Instrument Act for punishment to the signatory of the cheque for
haring committed an offence. However, before filing the said complaint
a statutory notice is liable to be given to the other party.

2. CGT stands for


A. Capital gain tribe
B. Crucial gain tax
C. Capital gain tax
D. Capital gross tax

www.bankingpdf.com
Answer & Explanation

C. Capital gain tax


Explanation:It is a direct tax that will be levied on sales and
purchases of capital assets such as Shares, stakes, even costlier items
which won’t have depreciation such as monuments, paintings.

3. It is the process at which the government Undertake austerity


measures to put restrictions on its economy such as de
licensing of industries, disinvestment from stack etc.
A. Fiscal Consolidation
B. Fiscal Policy
C. Monetary Consolidation
D. None of these

Answer & Explanation

A. Fiscal Consolidation
Explanation:Fiscal Consolidation refers to the policies undertaken by
Governments (national and sub-national levels) to reduce their deficits
and accumulation of debt stock. Key deficits of government are the
revenue deficit and the fiscal deficit. Our India is following under RBI, a
perfect measure by FRBM act (fiscal responsibility and budgetary
management act) 2003.

4. ________ is not a pillar of Basel III.


A. Minimum capital standards
B. Supervisory review
C. Market discipline
D. Consolidation of assets

Answer & Explanation

D. Consolidation of assets
Explanation:Basel III framework is based on 3 pillars including

www.bankingpdf.com
• Pillar 1: Minimum capital standards,
• Pillar 2: Supervisory review
• Pillar 3: Market discipline.

5. The first credit card facility to be recognized worldwide was


_______.
A. RuPay card
B. Maestro card
C. MasterCard
D. Visa card

Answer & Explanation

D. Visa card
Explanation:Visa Card- A credit, debit or prepaid card branded by
Visa Inc., a major payments technology company. Visa cards are
available to individual and business consumers with excellent, good,
poor or no credit. Each card has different terms that reflect the
consumer’s credit worthiness.

6. A provision in most loan and insurance contracts which allows


payment to be received for a certain period of time after the
actual due date and is known as _______.
A. Term Period
B. Loan Period
C. Grace Period
D. None of these

Answer & Explanation

C. Grace Period
Explanation:A grace period is usually the only a feature of a loan on
which interest is calculated monthly, if it is calculated at all. Under
some loan contracts, payments outstanding during grace periods are
interest free, but the majority have interest compounding during the

www.bankingpdf.com
grace period. Be sure to check your loan contract for the specifics on
any grace periods.
Credit cards for example, on which interest is calculated daily, do not
have any grace periods.

7. As per Banking, a Certificate of Deposit is


A. Negotiable Instrument
B. Transferrable Instrument
C. Honored Instrument
D. Only A & B

Answer & Explanation

D. Only A & B
Explanation:A Certificate of Deposit in India can be issue by: All
scheduled commercial banks excluding Regional Rural Banks (RRBs)
and Local Area Banks (LABs) Select All India Financial Institutions
permitted by RBI

8. Which among the following is/are eligible to issue Commercial


papers in India?
A. Corporates
B. Primary dealers
C. Financial Institutions
D. All of the above

Answer & Explanation

D. All of the above


Explanation:Commercial paper is an unsecured, short-term loan
issued by a corporation, typically for financing accounts receivable and
inventories. CP can be issued for maturities between a minimum of 7
days and a maximum of up to one year from the date of issue.
However, the maturity date of the CP should not go beyond the date
up to which the credit rating of the issuer is valid.

www.bankingpdf.com
9. Which of the following is the regulator of the Infrastructure
Debt Fund set up as a trust?
A. RBI
B. SEBI
C. IRDA
D. PFRDA

Answer & Explanation

B. SEBI
Explanation:Securities Exchange Board of India (SEBI) was set up in
1988 to regulate the functions of securities market. SEBI promotes
orderly and healthy development in the stock market but initially SEBI
was not able to exercise complete control over the stock market
transactions.

10. It takes around _________ time depending on the drawee


centre and collection arrangements to get outstation cheques
realised on a Collection basis.
A. 3weeks
B. 5weeks
C. 1week
D. 2week

Answer & Explanation

A. 3weeks
Explanation:Speed Clearing refers to collection of outstation cheques
(a cheque drawn on non-local bank branch) through the local clearing.
It facilitates collection of cheques drawn on outstation core-banking-
enabled branches of banks, if they have a net-worked branch locally.

www.bankingpdf.com
820 - 830 Questions :

1. Original headquarters of RBI were located at ___?


a. Calcutta
b. Bombay
c. New Delhi
d. Madras

Answer

calcutta
* Original headquarters of RBI were in Kolkata, but in 1937, it
was shifted to Shahid Bhagat Singh Marg, Mumbai.

2. In which year, Indian Rupee was devalued for the first time?
a. 1965
b. 1968
c. 1966
d. 1971

Answer

c. 1966

3. What was the initial share capital of Reserve Bank of India?


a. 5 crore
b. 10 crore
c. 15 crore
d. 20 crore

Answer

a. 5 crore
* RBI did not start as a Government owned bank but as a
privately held bank without major government ownership. It
started with a Share Capital of Rs. 5 Crore, divided into shares

www.bankingpdf.com
of Rs. 100 each fully paid up. In the beginning, this entire
capital was owned by private shareholders. Out of this Rs. 5
Crore, the amount of Rs. 4,97,8000 was subscribed by the
private shareholders while Rs. 2,20,000 was subscribed by
central government.

4. What was the name of State Bank of India (SBI) before it was
created by SBI Act?

a. Imperial bank of India


b. Imperial bank of Bengal
c. Imperial bank of Madras
d. Imperial bank of Hindustan

Answer

d. Imperial Bank of India


* The Presidency banks viz. Bank of Bengal, Bank of Bombay
and Bank of Madras amalgamated on January 27, 1921 and the
Imperial Bank of India was formed under the Imperial Bank of
India Act, 1920. After Independence, the Imperial Bank of India
was nationalized under the State Bank of India Act, 1955 and
State Bank of India (SBI) was formed.

5. Nainital Bank is a subsidiary of which of the following banks?


a. Bank of Baroda
b. SBI
c. Syndicate Bank
d. RBI

Answer

a. Bank of Baroda

www.bankingpdf.com
6. What is the share of the Foreign Banks in the branch network in
India?
a. Less than 1%
b. Less than 13%
c. Less than 6%
d. Less than 10%

Answer

a. Less than 1%
* Foreign Banks account for less than 1% of the total branch
network in the country.

7. Which of the following bodies regulates the Regional Rural


Banks?
a. Union Government
b. SBI
c. RBI
d. NABARD

Answer

s. NABARD
* They are regulated by NABARD; which is a subsidiary of RBI.
Other banks are regulated by RBI directly.

8. The owners of the cooperative bank are also its ______


a. All of the below
b. Creditors
c. Debters
d. Customers

Answer

d. Customers
* A cooperative is jointly owned enterprise in which same

www.bankingpdf.com
people are its customers who are also its owners. Thus, basic
difference between scheduled commercial banks and scheduled
cooperative banks is in their holding pattern.

9. In which country, the Unit Banking System originated?


a. France
b. Spain
c. United Kingdom
d. United states

Answer

d. United States
* Unit banking is a system of banking which originated in US. It
is a limited way of banking where banks operate only from a
single branch (or a few branches in the same area) taking care
of local community.

10. Which country has largest shadow banking market in the


world?
a. China
b. UK
c. Netherlands
d. USA

Answer

d. USA
* Shadow banking refers to the activities of financial
institutions that operate outside government-regulated banking
systems. United States has the largest shadow banking market
followed UK and China.

www.bankingpdf.com
830 - 840 Questions :

1. if a bank ties up with a retail vendor and then both of them


sponsor a credit card, then such card would be known as_____
a. Retail Credit Card
b. Vendor card
c. Co – Branded Card
d. Cash Back Card

Answer

c. Co- Branded Card


* It is a special type of credit card which is sponsored by both
the credit card issuing company and the participating retail
company or vendor. Co-branded credit card carries special deals
and savings from the participating merchants.

2. In terms of Banking Terminology, Affinity Card refers to ______


a. The Credit Cards that are linked to special organizations
b. The Credit Cards with zero interest rate on repayments
c. The Credit Cards exclusively for the Bank’s employees
d. All above the above

Answer

a. The Credit Cards that are linked to special organizations


* Credit cards linked to special organizations like sports clubs,
exclusive clubs and charities. Affinity credit cards can also help
raise funds, when a part of income from every transaction goes
toward the benefit of relevant organization.

3. From which country, the concept of Credit Card originated?


a. UK
b. USA

www.bankingpdf.com
c. France
d. Australia

Answer

b. USA
* The use of Credit Card first started in 1920s in United States
of America for selling the fuel to the automobile owners. Later,
it reached the customers when Diners Club was launched in
early 1950s. In 1958, the Bank of America issued the
BankAmericard in the California state and this is known to be
the first successful modern credit card.

4. The business entity that is authorized to accept cards for the


payment of goods and services is called?
a. Issuer
b. Acquirer
c. Merchant
d. Authorised Issuer

Answer

c. Merchant
* Merchant or Point of sale is any business entity that is
authorized to accept cards for the payment of goods and
services; it can be a brick and mortar shop or a website.

5. The international standard which defines the shape and size of


the I-Cards is ______
a. ISO/IEC 5200
b. ISO/IEC 9910
c. ISO/IEC 14000
d. ISO/IEC 7810

Answer

www.bankingpdf.com
d. ISO/IEC 7810

* ISO/IEC 7810 is the international standard which defines the


shape and size of the I-Cards including debit / credit cards.

6. Which is the first credit card facility to be recognised


worldwide?
a. Visa Card
b. Maestro Card
c. Master Card
d. Diner Card

Answer

a. Visa Card
* In 1977, Visa was adopted internationally and became the
first credit card facility to be recognised worldwide. It is a
conditional authorization given by a competent authority of a
country for a person who is not a citizen of that country to
enter its territory and to remain there for limited duration.

7. With reference to the prepaid credit cards, which among the


following is / are correct statements?
1. credit facility is offered for the purchaser of the card
2. no interest charged from the purchaser of the card
3. no purchasing fee and monthly fees charged before an
arbitrary time period Select the correct option from the codes
given below:

a. Only 1 & 2
b. Only 2 & 3
c. Only 1 & 3
d. 1 & 2 & 3

Answer

www.bankingpdf.com
b. Only 2 & 3
* The Prepaid cards require the cardholder to load money onto
the card before the card can be used. Purchases are withdrawn
from the card’s balance. The spending limit does not renew
until more money is loaded onto the card. Some points in
context of prepaid cards.

8. What does the cirrus logo on ATM / debit cards signifies__?

a. have cash access facility anywhere in India only


b. have cash access facility outside the India only
c. have cash access facility in or outside the India
d. have cash access facility in and outside the India

Answer

d. have cash access facility in and outside India


* The Cirrus is one of the logos found on the ATM or debit card
and provides the card owner worldwide cash access facility .
The Cirrus ATM network gives immediate access to the card
owner’s funds available in the local currency when travelling
abroad. Cirrus logo provides cash access facility in and outside
the India .

9. The Secured credit cards are generally available to the person


with _____:

a. Strong credit history


b. Poor Credit History
c. High Net worth Income
d. Government Job

Answer

www.bankingpdf.com
b. Poor Credit History
* The Secured credit cards are an option to allow a person with
a poor credit history or no credit history to have a credit card
which might not otherwise be available. These cards require a
security deposit to be placed on the card. The credit limit on a
secured credit card is typically equal to the deposit made on the
card, but it could be more in some cases. These cards are
available with both Visa and MasterCard logos on them.

10. What is the time period during which no interest is charged


on a credit card?
a. Grace period
b. Term period
c.Loan period
d. Sanction Period

Answer

a. Grace period
* A credit card’s grace period is the time the customer has to
pay the balance before interest is charged to the balance. Grace
periods vary, but usually range from 20 to 30 days depending
on the type of credit card and the issuing bank. If a customer is
late paying the balance, finance charges will be calculated and
the grace period does not apply.

840 - 850 Questions :

1. ____ nominates Jin Liqun for President of AIIB


a. China
b. India

www.bankingpdf.com
c. Pakistan
d. Japan

Answer

a. China
* China has nominated its former finance Minister Jin Liqun as
the Presidential candidate for $100 billion Asian Infrastructure
Investment Bank (AIIB). Presently, Jin is Secretary General of
AIIB’s interim secretariat. Prior to this he was chairman of
China International Capital Corporation (CICC), one of the
mainland’s largest investment banks. He also had held various
positions at the World Bank. He was the first Chinese national
to serve as a Vice President of Asian Development Bank (ADB)
which is controlled by Japan. Jin headed programmes for South,
Central and West Asia and private sector operations during his
5-year-long tenure at ADB.

2. Union Government appoints Sriram Kalyanaraman as MD and


CEO of ____
a. NHB
b. SBI
c. PNB
d. NABARD

Answer

a. NHB
* Union government has appointed Sriram Kalyanaraman as the
Managing Director (MD) and Chief Executive Officer (CEO) of
National Housing Bank (NHB) for a period of five years. With
this he became the first person from private sector to be
appointed as head of a public sector financial institution.
Presently, Kalyanaraman is director-business development
Equifax Credit Information Services. Prior to joining Equifax, he

www.bankingpdf.com
was Director, Business Clients and Asset Products at Deutsche
Bank India.

3. _____ launches mobile wallet service ‘Speed Pay’


a. BSNL
b. NABARD
c. SBI
d. PNB

Answer

a. BSNL
* State run telecom giant Bharat Sanchar Nigam Limited (BSNL)
launched pre-paid card linked mobile wallet service ‘Speed
Pay’. It was launched by Union Telecom Minister Ravi Shankar
Prasad in New Delhi. This service was launched by BSNL has
launched the service in partnership with IT company Pyro.

4. ____ nations, including India sign 60-article agreement on


China-led AIIB

a. 50
b. 60
c. 80
d. 90

Answer

a. 50
* 50 founding nations including India have signed 60-article
agreement of China-led multilateral Asian Infrastructure
Investment Bank (AIIB). Australia was the first country to sign
the agreement and was followed by 49 other members. Indian
Ambassador Ashok Kantha signed the agreement on behalf of
country.

www.bankingpdf.com
5. ____ to join China led AIIB

a. UK
b. QATAR
c. Australia
d. UAE

Answer

c. Australia
* Australia has decided to join the China-led Asian
Infrastructure Investment Bank (AIIB) as a founding member.
In this regard, Australia will contribute paid-in capital of 718.5
million US dollars over next five years i.e. till 2020. With this
investment it will become sixth largest shareholder in AIIB.

6. RBI extends deadline for exchanging pre- ____ currency notes


till December 2015
a. 2005
b. 2006
c. 2003
d. 2004

Answer

a. 2005
* The Reserve Bank of India (RBI) has extended the date to
exchange pre-2005 bank notes till December 31, 2015 for third
time. Earlier in December 2014, RBI had set the last date for
public to exchange these notes as June 30, 2015. RBI has urged
people to deposit the old design notes in the bank accounts or
exchange them at a bank branch. The apex bank also has stated
that the notes can be exchanged for their full value and all such
notes continue to remain legal tender.

www.bankingpdf.com
7. ______ Bank launches 10-second paperless instant loan
Scheme
a. HDFC
b. SBI
c. PNB
d. IDBI

Answer

a. HDFC

* HDFC Bank, India’s second-largest private sector lender has


launched a 10-second paperless instant loan Scheme for its
existing customers. The 10-second loan Scheme was launched
as part of HDFC Bank’s digital banking platform GoDigital. With
this, HDFC bank became first banking institution to completely
automate the entire process of loan approval and disbursement.
Under this scheme, bank will now disburse personal loans to its
customers in just 10 seconds and a pre-approved loan amount
will be available to them 24×7. The entire process of availing
the loan under this scheme will be completely paperless. In this
case, its customers by using net-banking or mobile banking
avail of this loan. Earlier, the bank had launched PayZapp
application to cater to e-commerce space under its digital
platform. It should be noted that, in financial year 2014-15, 63
per cent of HDFC Bank’s all transactions were conducted
through various digital channels.

8. RBI allows NRIs to subscribe ______


a. Bank Accounts
b. Draft
c. Chit funds
d. Cheat Funds

Answer

www.bankingpdf.com
c. Chit Funds
* Reserve Bank of Indian has allowed non-resident Indians
(NRIs) to invest in chit fund to encourage flow of capital into
the country. Decision in this regard was taken after RBI had
revised the extant guidelines for subscription to the chit funds
in consultation with the government. Henceforth, NRIs on non-
repatriation basis will be allowed to invest in chit fund without
any ceiling. However, the subscription to the chit funds by NRIs
will be mandatorily brought in through normal banking channel,
including through an account maintained with a bank in India.
The state government may permit any chit fund to accept
subscription from NRIs on non-repatriation basis. It will be
permitted by the registrar of chits or an officer authorized by
the state government in accordance with the provisions of the
Chit Fund Act, 1982. Earlier in May 2000, RBI had barred NRIs
from investing in a company or firms engaged in the business of
chit fund. Apart from this decision, RBI also extended the
scheme allowing airline companies to raise external commercial
borrowings (ECB) for working capital as a permissible end-use
under the approval route. Now, the ECB scheme will continue
till March 31, 2016.

9. Mahendra Kumar Sharma appointed as non-executive Chairman


of ____ Bank
a. HDFC
b. PNB
c. SBI
d. ICICI

Answer

d. ICICI
* Mahendra Kumar Sharma was appointed as non-executive
chairman of India’s largest private sector lender Industrial

www.bankingpdf.com
Credit and Investment Corporation of India (ICICI) Bank. He
will replace K V Kamath who had resigned from top most in
order to join as President of the Shanghai based New
Development Bank established by the BRICS nations. Presently,
Sharma is an independent director of two subsidiaries of ICICI
Bank viz ICICI Lombard General Insurance Company and ICICI
Prudential Asset Management Company. Earlier from 2003 to
2011, he also had served as an independent director on the
board of ICICI Bank. Prior to joining ICICI Bank, Sharma was
vice-president of Hindustan Unilever (HUL) and was associated
with HUL from 1974 to 2007. He has a strong understanding of
legal and corporate governance matters and had joined HUL in
1974 as a legal manager.

10. Kotak Mahindra, ____ Bank ink MoU for cross-border


business
a. ING
b. SBI
c. PNB
d. RBI

Answer

a. ING
* Kotak Mahindra Bank and ING Bank have inked a
memorandum of understanding (MoU) for exploring
opportunities arising from cross-border business, investment
and trade flows across various regions, including Europe. It was
signed between the two parties and covers an array of co-
operation in various areas such as trade remittances, finance. It
also includes services related to non-resident Indians,
corporate relationships, funding to counterparties’, fee-based
products, direct banking and digital strategy and cross-border
debt capital markets, mergers and acquisitions. Under the

www.bankingpdf.com
ambit of MoU, ING Bank will give its clients from The
Netherlands and the other 39 countries commercial banking
access to Kotak’s services in the vast Indian market. At the
same time it will also offer the combined clients in India the
opportunities of ING’s international network. It should be noted
that Kotak Mahindra and associates are significant shareholders
in Business Standard Private Ltd. ING Bank holds 6.5 per cent
stake in Kotak Mahindra Bank which is India’s fourth largest
private sector lender.

850 - 860 Questions :

1. Which of the following is NOT a correct statement?


a. Interest rates on unsecured loans is higher than the secured loans
b. Mortgage Loan is a kind of secured loan
c. Credit Card is a kind of secured loan
d. Unsecured loans are given out on the basis of credit worthiness of
the borrowers

Answer

c. Credit Card is a kind of secured loan


* In unsecured loans, the borrowers assets are not pledged as
collateral. Examples of such loans are personal loans, education
loans, credit cards etc. They are given out on the basis of credit
worthiness of the borrowers.

2. “No Risk, No reward”. This quote is most closely associated


with__?
a. Risk Diversification
b. Collateral Damage

www.bankingpdf.com
c. Risk-return trade-off
d. None of the above

Answer

c. Risk-return trade-off
* The principle of Risk-return trade-off says that higher the
potential return, higher is the risk. Low risk is associated with
low potential returns, whereas high risk is associated with high
potential returns.

3. In which among the following accounts, no cheque book is


issued?
a. Minor Account
b. Joint Account
c. Illiterate Account
d. Non Resident Account

Answer

Illiterate Account
* Illiterate accounts are opened on discretion of the banks if
the person personally goes to the bank along with a witness
already known to the bank and the depositor. No cheque books
are issued for such accounts. Any withdrawal is done by a
thumb impression of the depositor in presence of the bank
officer who is able to verify the identity

4. What is the priority sector loan target has been given to the
foreign banks operating in India in terms of agriculture loans?
a. 32%
b. 15%
c. 21%
d. No specific target

www.bankingpdf.com
Answer

No specific target
* In total loans, their target is fixed at 32%. There is no
specific target for agricultural loans.

5. Which among the following are often described as Double


Financial Repression for Banks in India?
a. Priority Sector Loans and Cash Reserve Ratio
b. Priority Sector Loans and Statutory Liquidity Ratio
c. Cash Reserve Ratio and Statutory Liquidity Ratio
d. Statutory Liquidity Ratio and Margin Requirements

Answer

Priority Sector Loans and Statutory Liquidity Ratio


* Priority Sector lending in India has been made a salient
feature of the banking in India mainly due to the social and
economic objectives that underlie PSL. However, banks are also
required to keep certain amount to maintain Statutory Liquidity
Ratio ( SLR) and from the remaining disposable amount, 40 per
cent is dedicated for the priority sector. Thus, large fraction of
banks’ resources cause the so called “Double Repression” on
the banking system.

6. In banking language, which among the following is called


Contingent Liability of the banks?
a. Fund based lending
b. Non fund based lending
c. Priority sector lending
d. Statutory pre-emptions

Answer

www.bankingpdf.com
Non fund based lending
* In Non-fund based lending, bank does not make any funds
outlay but only gives assurance. The “letter of credit” and
“bank guarantees” fall into the category of non-funding loans.
The non-funding loan can be converted to a fund-based advance
if the client fails to fulfil the term of contract with the
counterparty. In banking language, the non-funding advances
are called Contingent Liability of the banks.

7. In terms of agricultural loans in India, the short term credit


refers to the loans made for a period less than __:
a. 1 year
b. 15 Months
c. 18 Months
d. 24 Months

Answer

b. 15 Months
* In case of agriculture loans, there are three types of loans viz.
Short term (tenure 5 years).

8. To call a loan NPA, the interest or instalment of principal should


remain overdue for a minimum period of more than __?
a. 60 Days
b. 90 Days
c. 120 Days
d. 180 Days

Answer

90 Days
* According to RBI, terms loans on which interest or instalment
of principal remain overdue for a period of more than 90 days

www.bankingpdf.com
from the end of a particular quarter is called a Non- performing
Asset.

9. What is the loan limit for education under Priority Sector


Lending (PSL) for studying in India and abroad respectively?
a. Rs. 20 lakh, Rs. 10 Lakh
b. Rs. 10 lakh, Rs. 20 Lakh
c. Rs. 25 lakh, Rs. 25 Lakh
d. Rs. 20 lakh, Rs. 25 Lakh

Answer

Rs. 10 lakh, Rs. 20 Lakh


* Under Priority Sector Lending (PSL), the loans and advances
granted to only individuals for educational purposes up to Rs.10
lakh for studies in India and Rs. 20 lakh for studies abroad.

10. Who monitors Priority Sector Lending (PSL) in commercial


banks?
a. Reserve Bank of India (RBI)
b. Small Industries Development Bank of India (SIDBI)
c. National Bank for Agriculture and Rural Development (NABARD)
d. Government of India (GOI)

Answer

Reserve Bank of India (RBI)


* The priority sector lending by commercial banks is monitored
by Reserve Bank of India (RBI) through periodical returns
received from them.

www.bankingpdf.com
860 - 870 Questions :

1. The largest bank Imperial bank was nationalized in 1955 and


rechristened as State Bank of India on the recommendation of
which committee?
A. Rangarajan Committee
B. Chelliah Committee
C. Rekhi Committee
D. Gorewala Committee

Answer & Explanation

D. Gorewala Committee
Explanation: Bank of Madras merged into the other two in British
India i.e. Bank of Calcutta and Bank of Bombay to form the Imperial
Bank of India which became the State Bank of India.

2. SEBI was established in the year


A. 1994
B. 1990
C. 1992
D. 1982

Answer & Explanation

C. 1992
Explanation: SEBI(Securities and Exchange Board of India) was set
up with the main purpose of keeping a check on malpractices and
protect the interest of investors.

3. Which public sector bank of India is the first fully Indian bank,
established by the Indian capital?
A. Bank of Baroda
B. State Bank of India

www.bankingpdf.com
C. Central Bank of India
D. Punjab National Bank

Answer & Explanation

D. Punjab National Bank


Explanation: The first bank managed by Indians was Punjab National
Bank in 1895(Lahore) and is one of the largest banks in India. PNB has
the distinction of being the first Indian bank to have been started
solely with Indian capital that has survived to the present.

4. When RBI sells government securities its meaning is?


A. Liquidity in banking increases
B. Liquidity gets diminished
C. Liquidity remains unchanged
D. None of these

Answer & Explanation

B. Liquidity gets diminished


Explanation: A Government security is a tradable instrument issued
by the Central or the State Governments. It recognizes the
Government’s debt commitment. Such securities are short term
(treasury bills, with original maturities of less than one year) or long
term (Government bonds or dated securities with original maturity of
one year or more).

5. With a view to facilitate payment of balance in the deposit


account to the person named by the depositor without any
hassles in the event of death of account holder, the following
facility was introduced for bank account in our country.
A. Guarantee
B. Registration
C. Nomination
D. Will

www.bankingpdf.com
Answer & Explanation

C. Nomination
Explanation:The benefit of nomination is that in the event of death of
an account holder or locker holder, the Bank can release the account
proceeds or contents of the locker to the nominee without insisting
upon a Succession Certificate, Letter of Administration or Court Order.
When you open a savings bank account, or make a mutual fund
investment, there is a space which asks for the nominee details.
Nomination for joint holders is permitted, however in the event of
death of any of the holders the benefits will be transmitted to the
surviving holder’s name.

6. Which among the following statement is incorrect about SEBI?


A. Capital market regulator
B. Mutual fund regulator
C. Regulates the credit rating agencies in India
D. None

Answer & Explanation

D. None
Explanation: All are true

7. Which of the following rates are decided by the RBI called as


“Policy Rate”?
A. Cash reserve ratio
B. Lending rate
C. Bank rate
D. Deposit rate

Answer & Explanation

A. Cash reserve ratio


Explanation: CRR or cash reserve ratio is the minimum percentage of

www.bankingpdf.com
a bank’s deposits to be held in the form of cash. When a bank’s
deposits increase by Rs. 100 crore, and considering the present cash
reserve ratio of 3%, bank will have to hold additional Rs. 3 crore with
RBI and will be able to use only Rs. 97 crore for investments and
lending.

8. Non-Banking Financial Companies (NBFCs) are financial


institution that
A. holds a banking license
B. does not hold a banking license
C. are government undertaking institutions
D. None of these

Answer & Explanation

B. does not hold a banking license


Explanation: Housing Finance Companies, Merchant Banking
Companies, Stock Exchanges, Companies engaged in the business of
stock-broking, Venture Capital Fund Companies, Nidhi Companies,
Insurance companies and Chit Fund Companies are NBFCs but they
have been exempted from the requirement of registration under
Section 45-IA of the RBI Act, 1934.

9. In comparison with Liquidity Adjustment facility (LAF),


Marginal Standing Facility (MSF) has
A. Higher Rate of Interest
B. Lower Rate of Interest
C. No difference in interest rate
D. None of these

Answer & Explanation

A. Higher Rate of Interest


Explanation: Marginal standing facility is a window for banks to
borrow from Reserve Bank of India in emergency situation when inter-

www.bankingpdf.com
bank liquidity dries up completely. Banks borrow from the central bank
by pledging government securities at a rate higher than the repo rate
under liquidity adjustment facility or LAF in short.

10. Indian Depository Receipt (IDR) is the


A. Proof of an Indian company’s share
B. Is an unsecured money market instrument
C. Proof of ownership of foreign company’s share
D. Proof of earnings or profits of Indian Companies

Answer & Explanation

C. Proof of ownership of foreign company’s share


Explanation: These receipts can be listed in India and traded in
rupees. For instance foreign investors in the US-listed American
Depository Receipts of Infosys and Wipro get receipts against
ownership of shares held by an Indian custodian, an IDR is proof of
ownership of foreign company’s shares.

870 - 880 Questions :

1. Which of the following is not meant for the purpose of investment?


A. Mutual funds
B. Infrastructure funds
C. National Saving certificate
D. Letter of credit
E. None of these

Answer & Explanation

D. Letter of credit
Explanation: It is a document issued by a third party that guarantees

www.bankingpdf.com
payment for goods/services when the seller provides acceptable
documentation and are issued by banks or other financial institutions.

2. First Indian bank to open overseas branch?


A. Bank of India
B. Punjab National Bank
C. Bank of Baroda
D. Bank of India
E. None of these

Answer & Explanation

D. Bank of India
Explanation: Bank of India HQ- Mumbai, CEO & MD- M O Rego.

3. ECS stand for


A. External Commercial System
B. Electronic Clearing System
C. Electronic Commercial System
D. Electrical Clearing Service
E. Electronic Clearing Service

Answer & Explanation

E. Electronic Clearing Service


Explanation: It allows companies, Government departments and
institutions which have to make bulk payments in the form of dividend
warrants and interest payments on bonds to investors to transfer the
proceeds electronically to customers’ bank accounts.

4. Conversion of physical form of share into electronic form is called


A. Demat
B. Exchange
C. Materialization

www.bankingpdf.com
D. None of these
E. Clearance

Answer & Explanation

A. Demat
Explanation: Demat stands for Dematerialisation and is the process of
converting physical financial instruments such as share certificates,
mutual fund investments, and bonds into electronic form.

5. Which of the following is not correctly matched?


A. ICICI-1956
B. IDBI- 1964
C. IFCI- 1948
D. SIDBI- 1988
E. None of these

Answer & Explanation

D. SIDBI- 1988
Explanation: Small Industries Development Bank of India is an
independent financial institution to aid the growth and development of
micro, small and medium-scale enterprises in India and is founded in
April 2, 1990. CEO of SIDBI is Dr. Kshatrapati Shivaji

6. FINO stands for


A. Final Inclusion Network & operation
B. Finance Investment Network operation
C. Financial Inclusion Network Operation
D. Foreign Investment Network Operation
E. None of these

Answer & Explanation

C. Financial Inclusion Network Operation


Explanation: Financial Inclusion Network & Operation

www.bankingpdf.com
7. Nationalization of banks took place in India in how many phases?
A. 2
B. 1
C. 3
D. 4
E. None of these

Answer & Explanation

A. 2
Explanation: In 1969 and 1980

8. A closed economy is
A. Only Exports take place
B. Only Imports take place
C. Imports and exports take place
D. Neither Imports nor Exports take place
E. None of these

Answer & Explanation

D. Neither Imports nor Exports take place


Explanation: A closed economy means no imports are brought in and
no exports are sent out so as to provide consumers with everything
that they need from within the economy’s borders.

9. Who carry out open market operations?


A. SEBI
B. SIDBI
C. RBI
D. NABARD
E. None of these

Answer & Explanation

www.bankingpdf.com
C. RBI
Explanation: RBI carries out open market operations. Open market
operation is in which a country will conduct trade with outside regions.

10. Purpose of setting up Narsimhan committee-2?


A. Efficiency and productivity
B. Banking reform process
C. Export of IT sector
D. Fiscal Reform process
E. None of these

Answer & Explanation

B. Banking reform process


Explanation: In 1998 to analyze India’s banking sector and
recommending legislation and regulations to make it more effective,
competitive and efficient.issued by banks or other financial institutions.

890 - 900 Questions :

1. Export of services like shipping and insurance which can earn foreign
exchange without the transfer of goods from one country to another is
known as
A. Invisible Export
B. Visible Export
C. Open export
D. Closed export

Answer & Explanation


A. Invisible Export
Explanation:Invisible Export are like tourism, banking, foreign investments,
private transfers, and insurance. So basically, it’s the trading of services dealing
with the exchange of currency from one nation to another.

www.bankingpdf.com
2. By opening and investing in the Tax Saver Deposit Account Scheme in
a bank, a customer would get benefit under___________
A. Excise Duty
B. Customs Duty
C. Sales Tax
D. Income Tax
Answer & Explanation
D. Income Tax
Explanation: Under section 80C of Income Tax Act,1991 By opening and
investing in the Tax Saver Deposit Account Scheme in a bank, a customer would
get benefit under Income Tax
3. This organisation is made for empowering Micro, Small and Medium
enterprises in India?
A. ECGC
B. RBI
C. RRB
D. SIDBI
Answer & Explanation
D. SIDBI
Explanation: It is aimed to aid the growth and development of MSME in India.
4. Bancassurance can be sold to
A. Insurance Agents
B. Insurance companies
C. Banks
D. All existing and prospective bank customers
Answer & Explanation
D. All existing and prospective bank customers
Explanation: Bancassurance means selling insurance product through banks.
Bank and insurance company come up in a partnership where the bank sells the
tied insurance company’s insurance products to its clients.
5. Savings account with zero balance can be opened for
A. Students
B. BPL
C. With respect to Bank’s policies
D. Under Financial Inclusion scheme
Answer & Explanation
D. Under Financial Inclusion scheme
Explanation:Zero balance Saving Account – so your monthly average balance
requirement is nil
6. This is the amount of capital a bank or other financial institution has
to hold as required by its financial regulator.
A. CRR
B. CAR

www.bankingpdf.com
C. CRA
D. None of these
Answer & Explanation
B. CAR
Explanation:Capital Adequacy Ratio (CAR)is a bank’s ability to absorb losses
by calculating the ratio of capital to risk.
7. The cheque having a date subsequent to the date on which it is
drawn is known as
A. Post-dated cheque
B. Pre-dated cheque
C. Stale Cheque
D. Anti-dated cheque
Answer & Explanation
A. Post-dated cheque
Explanation:A post-dated cheque is a cheque written by the drawer for a date
in the future.
8. These are financial instruments used by the investors that are not
registered with the SEBI to invest in Indian securities.
A. Participatory notes
B. Promissory Notes
C. Treasury Bills
D. None of these
Answer & Explanation
A. Participatory notes
Explanation:It contains an unconditional promise to pay a certain sum to the
order of a specifically named person or to bearer
9. It is the extension of very small loans to the unemployed, to poor
entrepreneurs and to others living in poverty who are not considered
bankable.
A. Macro Credit
B. Medium Credit
C. Micro Credit
D. None of these
Answer & Explanation
C. Micro Credit
Explanation:A small financial loan made to poverty-stricken individuals seeking
to start their own business.
10. These are the lowest risk category instruments for the short term.
RBI issues them at a prefixed day and for a fixed amount.
A. Commercial Bills
B. Certificate of deposits
C. Letter of Credit
D. Treasury Bills

www.bankingpdf.com
Answer & Explanation
D. Treasury Bills
Explanation:RBI issues T-Bills for three different maturities: 91 days, 182 days
and 364 days. The 91 day T-Bills are issued on weekly auction basis while 182
day T-Bill auction is held on Wednesday preceding Non-reporting Friday and 364
day T-Bill auction on Wednesday preceding the Reporting Friday.

910 - 920 Questions :

1. The initial share capital for RBI was _________


A. Rs.5 lakh
B. Rs.5 crores
C. Rs.50 crores
D. Rs.1 crore

Answer & Explanation

B. Rs.5 crores
Explanation: The Reserve Bank of India was established on April 1,
1935 with the provisions of the Reserve Bank of India Act, 1934. The
Central Office of the Reserve Bank was initially established in Calcutta
but was permanently moved to Mumbai in 1937. It was originally
privately owned, since nationalisation in 1949, the Reserve Bank is
fully owned by the Government of India.

2. It offers buyers credit and lines of credit to foreign


governments and banks.
A. PNB
B. SBI
C. EXIM Bank
D. IDBI

Answer & Explanation

www.bankingpdf.com
C. EXIM Bank
Explanation:The Export-Import Bank of India was set up in 1982 with
main objective to ensure an approach in solving the allied problems
encountered by exporters in India.Recently,Export-Import (Exim) Bank
has organised a training programme for women in Rudraprayag district
of Uttarakhand to provide sustainable livelihoods to affected families
after 2013 floods.

3. The issuance of direction to housing finance institutions to


ensure their growth on sound lines.
A. HDFC
B. NHB
C. ICICI
D. SBI

Answer & Explanation

B.NHB
Explanation:National Housing Bank (NHB), a wholly owned subsidiary
of Reserve Bank of India (RBI), was set up in 1987. It is an apex
financial institution for housing and commenced its operations in 1988
with an objective to promote housing finance institutions both at local
and regional levels and to provide financial and other support incidental
to such institutions. CEO:Sriram Kalyanaraman

4. The rate of inflation increases when the purchasing power of


money___________
A. Increases
B. Decreases
C. Stable
D. None of these

Answer & Explanation

www.bankingpdf.com
B.Decreases
Explanation:Price inflation decreases people’s ability to pay for goods.
If an employee’s wages remain steady, but the cost of goods increases,
then the employee can afford less goods. As wage inflation occurs,
people will be able to buy more products. Misconception is that when
wages rise, prices also rise and there is little support that wage
inflations cause price inflation.

5. Full form of QIB is


A. Quantitative Institution Board
B. Qualified Inclusive Board
C. Qualified Institutional Buyers
D. Quantified Infrastructure Board

Answer & Explanation

C.Qualified Institutional Buyers


Explanation:It is a purchaser of securities that is considered
financially sophisticated and is legally identified by securities market
regulators to need less protection from issuers than most public
investors.

6. Insurance cover for bank deposits in India is provided by


A. LIC
B. GIIC
C. NICL
D. DICGC

Answer & Explanation

D. DICGC
Explanation:Deposit Insurance is to boost the faith of the public in the
banking system, and provide protection against the loss of deposits to
a significant extent. In India, the bank deposits are covered under the
insurance scheme provided by Deposit Insurance and Credit Guarantee

www.bankingpdf.com
Corporation (DICGC), a wholly owned subsidiary of the Reserve Bank
of India.

7. CRR and SLR are related to


A. Capital Market
B. Commodities Market
C. Money Market
D. None of these

Answer & Explanation

D. None of these
Explanation:Banks has to keep a certain percentage of its total
deposits with RBI as cash reserves that is CRR. Amount of liquid assets
such as precious metals(Gold) or other approved securities, that a
financial institution must maintain as reserves other than the cash is
SLR.

8. The following is not a primary function of a bank


A. Facilitating import of goods
B. Remittance facility
C. Safe custody of articles
D. Foreign Exchange

Answer & Explanation

A.Facilitating import of goods


Explanation: Primary functions: Accepting Deposits, Making
Advances,credit creation.
Secondary functions:Funds transfer, Cheques collection, Periodic
payments/collection, Portfolio management, Issue of draft, letter of
credit etc.,Locker facility, Underwriting of shares, Dealing in foreign
exchanges, Project reports, Social welfare programs.

www.bankingpdf.com
9. Full form of OLTAS
A. Online Tax Accounting System
B. Office Tax Allowing System
C. Onboard Local Transaction System
D. None of these

Answer & Explanation

A.Online Tax Accounting System


Explanation:OLTAS On-line Tax Accounting System (OLTAS) for Direct
Taxes) introduced in April, 2004 for collection, accounting and
reporting of the receipts and payments of Direct Taxes on-line through
a network of bank branches.

10. NBFC deposits are _____ and not backed by security.


A. Insured
B. Uninsured
C. Secured
D. None of these

Answer & Explanation

B.Uninsured
Explanation:NBFCs lend and make investments and hence their
activities are similar to that of banks however there are a few
differences like NBFC cannot accept demand deposits, do not form part
of the payment and settlement system and cannot issue cheques
drawn on itself, deposit insurance facility of Deposit Insurance and
Credit Guarantee Corporation is not available to depositors of NBFCs,
unlike in case of banks.

www.bankingpdf.com
920 - 930 Questions :

1. Through which of the following sources domestic funds are


raised by companies?
(a) IPO only
(b) FPO only
(c) Commercial papers only
(d) Both IPO and FPO
(e) All IPO, FPO and commercial papers

Answer

(a) IPO only

2. Out of FDI and FII, which is considered more permanent


(stable)?
(a) FII is considered more stable than FDI.
(b) Both are equally stable.
(c) Both are equally unstable.
(d) FDI is considered more stable than FII.
(e) None of these

Answer

(d) FDI is considered more stable than FII.

3. In the time of monetary shortage, RBI can buy commercial bills


from the market. By using this method what does RBI provide
to the market?
(a) It sucks credit from the market.
(b) It injects credit into the market.
(c) There is no effect on the credit situation in the market.
(d) None of these

Answer

www.bankingpdf.com
(b) It injects credit into the market.

4. Expand LIBOR?
(a) London Inter Bank Offered Rate
(b) London Inter Bank Official Rate
(c) London Inter Bank Offered Ratio
(d) London International Bank Offered Rate
(e) None of these

Answer

(a) London Inter Bank Offered Rate

5. When was IDBI delinked from the RBI and taken over by
Government of India?
(a) 1976
(b) 1977
(c) 1978
(d) 1980
(e) None of these

Answer

(a) 1976

6. If the price rises by not more than 3% per annum, what type of
inflation it is?
(a) Walking inflation
(b) Running inflation
(c) Creeping inflation
(d) Galloping inflation
(e) None of these

Answer

(c) Creeping inflation

www.bankingpdf.com
7. If the price rises by more than 20% and less than 1000% per
annum, what type of inflation it is?
(a) Galloping inflation
(b) Creeping inflation
(c) Stagflation
(d) Hyperinflation
(e) None of these

Answer

(a) Galloping inflation

8. If the Cash Reserve Ratio (CRR) is lowered by the RBI, its


impact on the credit creation will be to
(a) Decrease it
(b) No effect
(c) Increase it
(d) None of these

Answer

(c) Increase it

9. Who was the first Indian Governor of RBI?


(a) Yash Pal Singh
(b) Hemant Rao
(c) CD Deshmukh
(d) Jaipal Singh
(e) None of these

Answer

(c) CD Deshmukh

10. Expand CRAR in terms of banking sector.


(a) Capital- to- Return Asset Ratio

www.bankingpdf.com
(b) Capital –to-Risk Asset Ratio
(c) Capital –to-Risk Asset Range
(d) Core Risk Asset Return
(e) None of these

Answer

(b) Capital –to-Risk Asset Ratio

930 - 940 Questions :

1. Which of the following is not shared by the centre and the


states?
(a) Income tax
(b) Corporation tax
(c) Sales tax
(d) None of these
(e) All of these

Answer

(c) Sales tax

2. Credit is a
(a) Stock concept
(b) Flow concept
(c) A stock-flow concept
(d) A holding concept
(e) None of these

Answer

(c) A stock-flow concept

www.bankingpdf.com
3. The place where bankers meet and settle their mutual claims
and accounts is known as
(a) Clearing house
(b) Dumping ground
(c) Collection and clearing centre
(d) Treasury
(e) None of these

Answer

(a) Clearing house

4. Economic planning is an essential feature of


(a) Dual economy
(b) Mixed economy
(c) Social economy
(d) Capitalist economy
(e) None of these

Answer

(c) Social economy

5. Which of the following is considered as the founding father of


World Bank?
(a) Harry Dexter
(b) Lord Keynes
(c) LB Johnson
(d) Both (a) and (b)
(e) Both (a) and (c)

Answer

(d) Both (a) and (b)

www.bankingpdf.com
6. Who compiles Economic Survey report in India?
(a) RBI
(b) Ministry of Finance
(c) SEBI
(d) IRDA
(e) (a) and (b) combined

Answer

(b) Ministry of Finance

7. SEBI is a
(a) Constitutional body
(b) Statutory body
(c) Advisory body
(d) Non-statutory body
(e) None of these

Answer

(b) Statutory body

8. Who nominates directors on the central board of RBI?


(a) Governor of RBI
(b) Government of India
(c) A Panel of all commercial banks
(d) Empowered group of ministers
(e) None of these

Answer

(b) Government of India

9. Which of the following remains at the top of organizational


structure of RBI?
(a) Governor

www.bankingpdf.com
(b) Executive directors
(c) Central board of directors
(d) Chief general manager
(e) Principal chief general manager

Answer

(c) Central board of directors

10. Teaser loan are generally given for


(a) Home loans
(b) Car loans
(c) Personal loans
(d) Can be given on any loan
(e) None of these

Answer

(a) Home loans

940 - 950 Questions :

1. What does ‘Kite Flying’ refers to in the banking terminology?


(A) The practice of depositing and withdrawing frequently in a bank
account.
(B) Opening of multiple accounts with different names by any
individual
(C) Doing unauthorised business
(D) Permitting drawls against uncleared cheques
(E) None of these

Answer

(D) Permitting drawls against uncleared cheques

www.bankingpdf.com
2. Under which of the following type of account is a specified
amount deposited every month for a specified period, says, 12,
24, 36 and 60 months?
(a) Fixed Deposit Account
(b) Saving Bank Account
(c) Current account
(d) Recurring Account

Answer

(d) Recurring Account

3. RBI was established on ___________.


(a) April 1, 1925
(b) April 1, 1935
(c) April 1, 1945
(d) April 1, 1955

Answer

(b) April 1, 1935

4. Under which act does RBI issue directives to banks?


(a) PMLA Act, 2002
(b) RBI Act, 1934
(c) DICGC Act, 1961
(d) Banking Regulation Act, 1949

Answer

(d) Banking Regulation Act, 1949

5. When the Reserve Bank of India (RBI) is the lender of last


resort, what does it mean?
(a) RBI advances money to public whenever there is any emergency
(b) Commercial banks give fund to the RBI

www.bankingpdf.com
(c) RBI advances necessary credit against eligible securities
(d) All of the above
(e) None of the above

Answer

(c) RBI advances necessary credit against eligible


securities

6. Which of the following conditions must be fulfilled before a


bank is included in the Second Schedule to the Reserve Bank of
India Act?
(a) It must be a State co-operative bank or a company as defined in
the Companies Act, 1956 or an institution notified by the Central
Government in this behalf or a corporation or a company incorporated
by or under any law in force in any place outside In
(b) It must satisfy the Reserve Bank of India that its affairs are not
being conducted in a manner detrimental to the interests of the
depositors
(c) It must have a paid-up capital and reserves of an aggregate value
of not less than Rs. 5 lakh
(d) Only (a)
(e) All of the above

Answer

(e) All of the above

7. What is macro-economics?
(a) macro economics deals with economic activities of the level of an
economy as a whole
(b) it deals with only a segment of an economy
(c) it deals with both 1 and 2
(d) none of these

www.bankingpdf.com
Answer

8. Which of the following banks was first to establish merchant


banking business in India?
(a) ABN Amro Bank
(b) Citibank
(c) HDFC Bank
(d) Standard chartered
(e) Grindlays Bank

Answer

(e) Grindlays Bank

9. Free Trade Refers to


(a) Free movement of Goods from one country to other.
(b) Movement of Goods from one Country to another free of cost.
(c) Unrestricted exchange of Goods and Services.
(d) Tool free of Duty

Answer

(c) Unrestricted exchange of Goods and Services.

10. ‘Swabhiman’, the financial inclusion scheme, comes under


the purview of which ministry?
(a) Ministry of Commerce
(b) Ministry of Home Affairs
(c) Ministry of Finance
(d) Ministry of External Affairs

Answer

(c) Ministry of Finance

www.bankingpdf.com
950 - 960 Questions :

1. The tax payers data flows directly from banks to Tax


Information Network (TIN). Who maintains TIN?
(a) RBI
(b) National Securities Depository Ltd.
(c) National Securities Defense Ltd.
(d) Enforcement Directorate
(e) None of these

Answer

(b) National Securities Depository Ltd.

2. What is the meaning of ‘I promise to pay’ clause written on


notes?
(a) The bank’s obligation to pay the value of banknote does not arise
out of a contract but out of statutory provisions.
(b) The bank’s obligation to pay the value of banknote does arise out
of a contract but not out of statutory provisions.
(c) This obligation is neither due to a contract nor a statutory
provision.
(d) It means that to pay you the value, there is sovereign guaranty.
(e) None of these.

Answer

(a) The bank’s obligation to pay the value of banknote does not
arise out of a contract but out of statutory provisions.

3. RBI issued its first banknote in Jan 1938 bearing the portrait of
George IV. What was its denomination?
(a) Rs. 5
(b) Rs. 10
(c) Rs. 1

www.bankingpdf.com
(d) Rs. 2
(e) None of these

Answer

(a) Rs. 5

4. Expand CIN with respect to OLTAS.


(a) Challan Identification Number
(b) Challan Identification Notary
(c) Channel Identification Number
(d) Challan Inventory Number
(e) None of these

Answer

(a) Challan Identification Number

5. When were banknotes in the Mahatma Gandhi Series


introduced?
(a) 1991
(b) 1995
(c) 1996
(d) 2000
(e) None of these

Answer

(c) 1996

6. Banknotes can be issued in any denominations but coins can be


issued up to what denomination?
(a) Rs. 5000
(b) Rs. 1000
(c) Rs. 500

www.bankingpdf.com
(d) Rs. 100
(e) None of these

Answer

(b) Rs. 1000

7. Consider these statements:-


Statement I – One rupee note is the liability of GOI.
Statement II – Two rupee note is the liability of RBI.
(a) Both statements are true.
(b) Both are false.
(c) Statement I is true, whereas II is false.
(d) Statement I is false, whereas II is true.
(e) None of these.

Answer

(a) Both statements are true.

8. Consider these statements:-


Statement I – RBI decides volume and value of banknotes to be
printed.
Statement II – RBI decides volume and value of coins to be
minted.
(a) Both are true.
(b) Both are false.
(c) Statement I is true, but statement II is false.
(d) Statement I is false, whereas II is true.
(e) None of these.

Answer

(c) Statement I is true, but statement II is false.

www.bankingpdf.com
9. To enforce law and order Government of India has many
agencies. SFIO is one of them. Expand it.
(a) Serious Fraud Interrogation Office
(b) Serious Fraud Investigation Office
(c) Serious Feud Interrogation Office
(d) Serious Fraud Interrogation Ordinance
(e) None of these.

Answer

(b) Serious Fraud Investigation Office

10. Consider these statements:-


Statement I – Other than Rs. 1000 note, the security threads of
the banknotes contain the word ‘Bharat’ in Devanagari script
and ‘RBI’ alternatively.
Statement II – In Rs. 1000 note, the security thread contains
the word ‘Bharat’ in Devanagari script, ‘1000’, and ‘RBI”.
(a) Both are false.
(b) Both are true.
(c) Statement I is true but II is false.
(d) Statement I is false but II is true.
(e) None of these

Answer

(b) Both are true.

960 - 970 Questions :

1. If the RBI wants to infuse credit in Banking system in our


country, which of the followings can be done?
(a) Decreasing CRR

www.bankingpdf.com
(b) Decreasing SLR
(c) Both (a) and (b)
(d) None of the above

Answer

(c) Both (a) and (b)

2. Buying and selling of eligible securities by RBI in the money


market can be termed as
(a) Open Economy Operations
(b) Open Market Operations
(c) Credit Control Measures
(d) Credit Creation Measures
(e) None of these

Answer

(b) Open Market Operations

3. Which agency in India provides refinance of export credit?


(a) SIDBI
(b) NABARD
(c) RBI
(d) EXIM Bank
(e) Government of India

Answer

(d) EXIM Bank

4. When the Government of India cannot raise enough financial


resources through taxation, it finances its expenditure through
various means like borrowing from market, running down its
balances with RBI etc. This can be exactly called as
(a) Fiscal Deficit Financing

www.bankingpdf.com
(b) Deficit Financing
(c) Capital Infusion
(d) Restructuring
(e) None of these

Answer

(b) Deficit Financing

5. Expand MAT related to tax?


(a) Minimum Additional Tax
(b) Maximum Alternative Tax
(c) Maximum Additional Tax
(d) Minimum Alternative Tax
(e) None of these

Answer

(d) Minimum Alternative Tax

6. Expand FRBM.
(a) Financial Responsibility and Budget Management
(b) Fiscal Responsibility and Budget Maintenance
(c) Fiscal Responsibility and Budget Management
(d) Fiscal Risk and Budget Management
(e) None of these

Answer

(c) Fiscal Responsibility and Budget Management

7. Deficit financing can (these are also demerits of it)


(a) Cause inflation
(b) Bring rise in fiscal deficit
(c) Bring rise in credit creation in banks

www.bankingpdf.com
(d) All above
(e) None of the above

Answer

(d) All above

8. If the rupees depreciate, what is the effect on the exporters?


(a) They are unaffected
(b) They are in loss
(c) They are in profit
(d) They get credit crunch
(e) None of these

Answer

(c) They are in profit

9. What is Reserve Money (RM)?


(a) Currency in circulation with the public
(b) Deposits of some people with RBI
(c) Cash reserves of the banks
(d) Total of the above three options
(e) None of the above

Answer

(d) Total of the above three options

10. What is the objective of introduction of ‘Marginal Standing


Facility’ by RBI?
(a) To contain volatility in the overnight inter-bank rates
(b) To contain volatility in MIBOR
(c) To contain volatility in LIBOR
(d)To match with standard Tier 1 capital Ratio
(e) None of these

www.bankingpdf.com
Answer

(a) To contain volatility in the overnight inter-bank rates

970 - 980 Questions :

1. Which of the following can be called as ‘National Income’?


(a) Net National Income at factor cost
(b) Gross National Income at factor cost
(c) Net Domestic Product at factor cost
(d) Gross Domestic Product at factor cost
(e) None of these

Answer

(a) Net National Income at factor cost

2. What is ‘STT’?
(a) Standard Transaction Tax
(b) Securities transaction tax
(c) Securities transfer tax
(d) Standard transfer tax
(e) None of these

Answer

(b) Securities transaction tax

3. Which of the followings can be used for Current Account Deficit


financing?
(a) FII only
(b) FDI only
(c) ECB only

www.bankingpdf.com
(d) Both FII and FDI only
(e) All FII, FDI and ECB

Answer

(e) All FII, FDI and ECB

4. Expand FIPP –
(a) Foreign Investment Promotion Brochure
(b) Foreign Investment Production Board
(c) Foreign Investment Promotion Board
(d) Foreign Direct Investment Promotion Board
(e) None of these

Answer

(c) Foreign Investment Promotion Board

5. Bureau of Indian Standards (BIS) has its headquarters in


(a) Mumbai
(b) Kolkata
(c) Bhubaneswar
(d) New Delhi
(e) None of these

Answer

(d) New Delhi

6. Who regulates chit funds in India?


(a) Government of India
(b) State Governments
(c) Not regulated at all
(d) Concerned Districts
(e) None of these

www.bankingpdf.com
Answer

(b) State Governments

7. If the Statuary Liquidity Ratio (SLR) is increased, what will be


the effect on Bank’s credit situation?
(a) It will be reduced
(b) It will be increased
(c) It will remain unaffected
(d) None of these

Answer

(a) It will be reduced

8. You might have heard about PMI data. What is PMI?


(a) Prudential Managers Index
(b) Purchasing Managers Indian
(c) Purchasing Managers Index
(d) Prudential Managers Indian
(e) None of these

Answer

(c) Purchasing Managers Index

9. Banking sector comes under which sector?


(a) Services sector
(b) Manufacturing sector
(c) Realty sector
(d) Industrial sector
(e) None of these

Answer

(a) Services sector

www.bankingpdf.com
10. Which of the following agreement/accord is related to
baking reforms?
(a) Vienna agreement
(b) Basel accord
(c) Nagoya Protocol
(d) Cartagena Protocol
(e) None of these

Answer

(b) Basel accord

980 - 990 Questions :

1. Which of the following deposits cannot be withdrawn for a


preset fixed period of time?
(a) Current
(b) Term
(c) Saving
(d) Recurring

Answer

(e) None pf these

2. Which of the following is an example of world-wide financial


messaging network?
(a) NEFT
(b) CHIPS
(c) SFMS
(d) SWIFT
(e) None of these

Answer

www.bankingpdf.com
(d) SWIFT

3. Which of the following techniques, Indian currency notes do not


use?
(a) Security threads
(b) Optically variable ink
(c) Micro lettering
(d) Intaglio printing
(e) All of the above are used.

Answer

(e) All of the above are used.

4. For issuing notes, RBI is required to hold the minimum reserves


of ______ crores Rs. in Gold.
(a) 85
(b) 125
(c) 115
(d) 100
(e) None of these

Answer

(c) 115

5. Which of the following is not a money market instrument?


(a) T-Bills
(b) Commercial paper
(c) Certificate of deposits
(d) Mutual funds
(e) All of the above

Answer

(d) Mutual funds

www.bankingpdf.com
6. Which of the following is not a credit rating agency?
(a) CRISIL
(b) Moody
(c) S & P
(d) Fitch
(e) All of the above are credit rating agency

Answer

(e) All of the above are credit rating agency

7. When a customer deposits a sum with bank, the bank becomes


his _______.
(a) Creditor
(b) Debtors
(c) Trustee
(d) Convener
(e) None of these

Answer

(b) Debtors

8. Agriculture sector contributes ______% in GDP.( Approx)


(a) Below 1%
(b) Above 50% but below 70%
(c) Below 10% but more than 5%
(d) Near about 20%
(e) None of these

Answer

(d) Near about 20%

9. Which of the following is related to banking industry?


(a) CAPART

www.bankingpdf.com
(b) PURA
(c) CMOS
(d) LIEN
(e) All of the above

Answer

(d) LIEN

10. Commercial banks money is divided into M1-M3, whereas


central bank money is known as
(a) MBZ
(b) MBM
(c) MB
(d) MCB
(e) None of these

Answer

(c) MB

990 - 1000 Questions :

1. Which of the following is not a credit rating agency?


(a) WADA
(b) Fitch
(c) CRISIL
(d) Moody
(e) S & P

Answer

(a) WADA

www.bankingpdf.com
2. Who used to be the chairman of the Board for Financial
Supervision (BFS) in RBI?
(a) RBI dupty governor
(b) RBI governor
(c) RBI chief general manager
(d) Finance Minister
(e) None of these

Answer

(b) RBI governor

3. Which of the following scheme is related to Financial Inclusion?


(a) AADHAR
(b) AAY
(c) Swabhiman
(d) Swavalamban
(e) All of the above

Answer

(c) Swabhiman

4. What is MZM in money market?


(a) Money with highest return
(b) Money with zero maturity
(c) Money with sovereign guarantee
(d) Monetary base or total currency
(e) None of these

Answer

(b) Money with zero maturity

5. Among M1, M2, and M3 (Different criteria of money), which is


used to forecast inflation data of a country? However it is not

www.bankingpdf.com
used by Indian economy.
(a) M3
(b) M1
(c) M2
(d) All of the above
(e) None of the above

Answer

(c) M2

6. The Tier-1 capital of banks does not include


(a) Paid-up capital
(b) Revaluation reserves
(c) Statutory reserves
(d) Investment fluctuation reserves
(e) None of these

Answer

(b) Revaluation reserves

7. When was the Board for Financial Supervision (BFS)


constituted, which undertakes integrated supervision of
different sectors of the financial system?
(a) 1991
(b) 1992
(c) 1993
(d) 1994
(e) None of these

Answer

(d) 1994

www.bankingpdf.com
8. Where is National Institute of Bank Management (NIBM)
located?
(a) Nasik
(b) Mumbai
(c) Pune
(d) New Delhi
(e) None of these

Answer

(c) Pune

9. Which of the following is a fully-owned subsidiary of RBI?


(a) NHB
(b) DICGC
(c) BRBNMPL
(d) All of the above
(e) Only (a) and (b)

Answer

(d) All of the above

10. Expand DICGC.


(a) Deposit insurance and credit guarantee corporation
(b) Deposit investment and credit guarantee corporation
(c) Deposit investment and credit governing corporation
(d) Deposit insurance and credit governing corporation
(e) None of these

Answer

(a) Deposit insurance and credit guarantee corporation

www.bankingpdf.com
1000 – 1015 Questions :

1. Who among the following is the head of the committee, which


is recently set up to suggest measures for cost-cutting and
optional utilization of assets of Air India?
(1) Suresh Malhotra
(2) Ravindra H. Dholakia
(3) Nagendra Kumar
(4) Vijay Sinha
(5) None of these

Answer

(2) Ravindra H. Dholakia

2. Who is the head of the committee appointed for Capital Account


Convertibility in Banks?
(1) M.B. Shah
(2) Mukul Mudgal
(3) M. Damodaran
(4) S.S. Tarapore
(5) None of these

Answer

(4) S.S. Tarapore

3. Who among the following is the head of the three member


committee of jurists, constituted to give recommendations on
amending laws to provide speedier justice and enhanced
punishment in sexual assault cases?
(1) Leila Seth
(2) J.S. Verma

www.bankingpdf.com
(3) Gopal Subramaniam
(4) S.P. Barucha
(5) None of these

Answer

(2) J.S. Verma

4. Which among the following committee is working on the


revision of the Whole sale Price Index?
(1) Saumitra Chaudhuri committee
(2) Mihir Shah committee
(3) Narendra Jadhav committee
(4) Syeda Hameed committee
(5) None of these

Answer

(1) Saumitra Chaudhuri committee

5. The Concept of ‘Universal Banking’ was implemented in India


on the recommendations of:
(1) Abid Hussai Committee
(2) R H Khan Committee
(3) S Padmanabhan Committee
(4) YH Malegam Committee
(5) None of these

Answer

(2) R H Khan Committee

6. Which among the following committee has looked inot the


maladies affecting the proper functioning of the public
distribution systems (PDS) in India?
(1) Suresh Tendulkar Committee

www.bankingpdf.com
(2) Kaushik Basu Committee
(3) Wadhwa Committee
(4) Rangarajan Committee
(5) None of these

Answer

(3) Wadhwa Committee

7. Who among the following is the head of the one-man inquire


committee appointed by the government to inquire into reports
on the US lobbying activities of Wal-Mart for access to India
market?
(1) V.N. Khare
(2) Y.K. Sabharwal
(3) Mukul Mudgal
(4) R.C. Lahoti
(5) None of these

Answer

(1) V.N. Khare

8. The foundation for induction of computer technology in the


Indian banking system was laid with the recommendations of
which of the following committees?
(1) Abid Hussain Committee
(2) Deepak Parekh Committee
(3) Rangarajan Committee
(4) Narsimham Committee
(5) Tarapore Committee

Answer

(3) Rangarajan Committee

www.bankingpdf.com
9. Which among the following committee has suggested a
common national; examination with weight age to the State
Board results normalized on the basis of percentile formula for
admission to engineering institutions?
(1) Suresh Tandon Committee
(2) T. Ramasami Committee
(3) Rajendra Kumar Committee
(4) Dharam Paul Committee
(5) M. Damodran Committee

Answer

(2) T. Ramasami Committee

10. Which among the following committee is related with the


issues and concerns in the Non-banking financial Companies
(NBFCs) sector?
(1) Ragu Ram Committee
(2) Usha Thorat Comiittee
(3) Shome Committee
(4) Tarapore Committee
(5) None of these

Answer

(2) Usha Thorat Comiittee

11. In the context of the 12th five year plan, Planning


Commission has set up committee to recommend detail
mythology for identification of BPL families in Urban areas?
(1) R.Puri Committee
(2) Tendulkar Committee
(3) S.R. Hashim Committee
(4) Chaturvedi Committee
(5) None of these

www.bankingpdf.com
Answer

(3) S.R. Hashim Committee

12. Who among the following is the head of the expert


committee on Streamlining Short-term Co-operative Structure
in India?
(1) MB Shah Committee
(2) P Kotaiah
(3) Prakash Bakshi
(4) R K Singh
(5) None of these

Answer

(3) Prakash Bakshi

13. Who among the following heads the RBI Working group on
boosting Export Finance?
(1) Govind Kumar Menon
(2) Mukul Mudgal
(3) Gopal Kundu
(4) G Padmanabhan
(5) None of these

Answer

(4) G Padmanabhan

14. Who among the following is appointed by the Prime Minister


Manmohan Singh as the Chairman of the committee on General
Anti Avoidance Rules (GAAR)?
(1) Ravindra H. Dholakia
(2) Suresh Malhotra
(3) D.K. Jain

www.bankingpdf.com
(4) Parthasarathi Shome
(5) None of these

Answer

(4) Parthasarathi Shome

15. Who among the following is the head of the committee


formed by SEBI to frame a single set of guidelines for all types
of foreign investors?
(1) Prabhat Kumar
(2) T.R. Prasad
(3) K.M. Chandrashekhar
(4) Kamal Pande
(5) None of these

Answer

(3) K.M. Chandrashekhar

www.bankingpdf.com
www.bankingpdf.com
www.bankingpdf.com
www.bankingpdf.com

You might also like